Exemplar 22.B - Nicotine Addiction, Chapter 10 - Substance Abuse, NCLEX STUDY QUESTIONS-Drug abuse, nicotine nclex, Alcohol Abuse NCLEX Style Practice Ques, Substance Abuse and Addictive Disorders (ATI Chapter 17), Chp 9: Substance Abuse, Substance A...

Lakukan tugas rumah & ujian kamu dengan baik sekarang menggunakan Quizwiz!

The nurse recognizes that patients who use nicotine are at risk for what conditions?

cataracts and Abdominal aortic aneurysm

How many cigarettes are in a pack year?

20 cigarettes a day for a year.

Tachycardia

Abnormally fast heart rate

Perforated nasal septum

Where the cartilaginous membrane dividing the nostrils, develops a hole or fissure.

Which drug is NOT similar to the neurotransmitter serotonin? LSD mescaline psilocybin marijuana

marijuana

"The nurse recognizes that clients should be under medical supervision when attempting withdrawal from what drug? Inhalants Hallucinogens Marijuana Alcohol

" Alcohol" Objective: Describe the role of the nurse in delivering care to individuals who have substance abuse issues. Rationale: Symptoms of withdrawal can be particularly severe for clients physically dependent on alcohol and sedatives. Because of the severity of the symptoms, the process of withdrawal from these agents is best accomplished in a substance abuse treatment facility. Cognitive Level: Application Client Need: Physiological Integrity: Pharmacological and Parenteral Therapies Nursing Process: Planning "

4. The nurse is caring for a client who is experiencing alcohol withdrawal. The nurse would be most concerned if the client exhibited which of the following? 1. Hallucinations 2. Nervousness 3. Diaphoresis 4. Nausea

1. Hallucinations are a sign of late alcohol withdrawal. The nurse should stay with the client, have someone notify the physician, and institute seizure precautions. Nervousness, diaphoresis, and nausea are signs of early withdrawal. Lisko, Susan (2013-10-01). NCLEX-RN Questions and Answers Made Incredibly Easy (Nclexrn Questions & Answers Made Incredibly Easy) (Kindle Locations 15873-15875). Lippincott Williams & Wilkins. Kindle Edition.

52. A client is being discharged from an alcohol treatment program. The client's wife states, "I'm so afraid that when my husband leaves here, he'll relapse. How can I deal with this?" Which nursing statement would be most appropriate? 1) "Many family members of alcoholics find the Al-Anon support group to be helpful." 2) "You could try going out and having a few beers with him when he gets the urge to drink." 3) "Just make sure he doesn't drink at home. Find all of his hidden bottles and empty them." 4) "Tell your husband that if he drinks again, you will leave him."

1) "Many family members of alcoholics find the Al-Anon support group to be helpful." Al-Anon is a nonprofit organization that provides group support for the family and close friends of alcoholics.

A nurse is caring for a client who is experiencing amphetamine withdrawal. The nurse should assess the client for which of the following? 1. Disturbed sleep 2. Increased yawning 3. Psychomotor agitation 4. Inability to concentrate

1. It's common for a person withdrawing from amphetamines to experience disturbed sleep and unpleasant dreams. Increased yawning is seen with clients withdrawing from opioids. Psychomotor agitation is seen in cocaine withdrawal, and the inability to concentrate is seen in caffeine withdrawal. Lisko, Susan (2013-10-01). NCLEX-RN Questions and Answers Made Incredibly Easy (Nclexrn Questions & Answers Made Incredibly Easy) (Kindle Locations 16289-16291). Lippincott Williams & Wilkins. Kindle Edition.

A nurse suggests to a client struggling with alcohol addiction that keeping a journal may be helpful. The goal of this nursing intervention is to help the client do what? 1. Identify stressors and responses to them. 2. Understand the diagnosis. 3. Help others by reading the journal to them. 4. Develop an emergency plan for use in a crisis.

1. Keeping a journal enables the client to identify problems and patterns of coping. From this information, the difficulties the client faces can be addressed. A journal isn't necessarily kept to promote better understanding of the client's illness, but it helps the client understand himself better. Journals aren't read to other people unless the client wants to share a particular part. Journals aren't typically used for identifying an emergency plan for use in a crisis. Lisko, Susan (2013-10-01). NCLEX-RN Questions and Answers Made Incredibly Easy (Nclexrn Questions & Answers Made Incredibly Easy) (Kindle Locations 16107-16110). Lippincott Williams & Wilkins. Kindle Edition.

11. Upon admission for symptoms of alcohol withdrawal, a client states, I havent eaten in 3 days. Assessment reveals BP 170/100 mm Hg, P 110, R 28, and T 97F (36C) with dry skin, dry mucous membranes, and poor skin turgor. What should be the priority nursing diagnosis? A. Knowledge deficit B. Fluid volume excess C. Imbalanced nutrition: less than body requirements D. Ineffective individual coping

ANS: C The nurse should assess that the priority nursing diagnosis is imbalanced nutrition: less than body requirements. The client is exhibiting signs and symptoms of malnutrition as well as alcohol withdrawal. The nurse should consult a dietitian, restrict sodium intake to minimize fluid retention, and provide small, frequent feedings of nonirritating foods.

Upon admission for symptoms of alcohol withdrawal a client states, "I haven't eaten in 3 days." Assessment reveals BP 170/100 mm Hg, P 110, R 28, and T 97F (36C) with dry skin, dry mucous membranes, and poor skin turgor. What should be the priority nursing diagnosis? A. Knowledge deficit B. Fluid volume excess C. Imbalanced nutrition: less than body requirements D. Ineffective individual coping

ANS: C The nurse should assess that the priority nursing diagnosis is imbalanced nutrition: less than body requirements. The client is exhibiting signs and symptoms of malnutrition as well as alcohol withdrawal. The nurse should consult a dietitian, restrict sodium intake to minimize fluid retention, and provide small, frequent feedings of nonirritating foods.

During group therapy, a client diagnosed with chronic alcohol dependence states, "I would not have boozed it up if my wife hadn't been nagging me all the time to get a job. She never did think that I was good enough for her." How should a nurse interpret this statement? A. The client is using denial by avoiding responsibility. B. The client is using displacement by blaming his wife. C. The client is using rationalization to excuse his alcohol dependence. D. The client is using reaction formation by appealing to the group for sympathy.

ANS: C The nurse should interpret that the client is using rationalization to excuse his alcohol dependence. Rationalization is the defense mechanism by which people avoid taking responsibility for their actions by making excuses for the behavior.

15. During group therapy, a client diagnosed with alcohol use disorder states, I would not have boozed it up if my wife hadnt been nagging me all the time to get a job. She never did think that I was good enough for her. How should a nurse interpret this statement? A. The client is using denial by avoiding responsibility. B. The client is using displacement by blaming his wife. C. The client is using rationalization to excuse his alcohol dependence. D. The client is using reaction formation by appealing to the group for sympathy.

ANS: C The nurse should interpret that the client is using rationalization to excuse his alcohol use disorder. Rationalization is the defense mechanism by which people avoid taking responsibility for their actions by making excuses for the behavior.

3. On the first day of a clients alcohol detoxification, which nursing intervention should take priority? A. Strongly encourage the client to attend 90 Alcoholics Anonymous meetings in 90 days. B. Educate the client about the biopsychosocial consequences of alcohol abuse. C. Administer ordered chlordiazepoxide (Librium) in a dosage according to protocol. D. Administer vitamin B1 to prevent Wernicke-Korsakoff syndrome.

ANS: C The priority nursing intervention for this client should be to administer ordered chlordiazepoxide (Librium) in a dosage according to protocol. Chlordiazepoxide (Librium) is a benzodiazepine and is often used for substitution therapy in alcohol withdrawal. Substitution therapy may be required to reduce life-threatening effects of the rebound stimulation of the central nervous system that occurs during withdrawal.

14. Which medication orders should a nurse anticipate for a client who has a history of complicated withdrawal from benzodiazepines? A. Haloperidol (Haldol) and fluoxetine (Prozac) B. Carbamazepine (Tegretol) and donepezil (Aricept) C. Disulfiram (Antabuse) and lorazepam (Ativan) D. Chlordiazepoxide (Librium) and phenytoin (Dilantin)

ANS: D The nurse should anticipate that a physician would order chlordiazepoxide (Librium) and phenytoin (Dilantin) for a client who has a history of complicated withdrawal from benzodiazepines. It is common for long-lasting benzodiazepines to be prescribed for substitution therapy. Phenytoin (Dilantin) is an anticonvulsant that would be indicated for a client who has experienced a complicated withdrawal. Complicated withdrawals may progress to seizure activity.

A client is admitted to the emergency department with signs of drug use. The client reports ingesting Percocet and is currently experiencing respiratory depression. Based on this information, which prescription should the nurse anticipate for this client? A) Diazepam B) Haldol C) Vitamin B12 D) Naloxone

Answer: D Percocet (oxycodone and acetaminophen) is an opioid narcotic. Thus, the nurse should anticipate administration of naloxone (Narcan), because this medication is used in the treatment of narcotic overdose. Diazepam would be prescribed to manage the symptoms of substance withdrawal. Haldol would be administered to manage an overdose of phencyclidine (PCP). Vitamin B12 would be used to manage the neurologic symptoms that might accompany a nitrate overdose.

Which medication orders should a nurse anticipate for a client who has a history of complicated withdrawal from benzodiazepines? A. Haloperidol (Haldol) and fluoxetine (Prozac) B. Carbamazepine (Tegretol) and donepezil (Aricept) C. Disulfiram (Antabuse) and lorazepan (Ativan) D. Chlordiazepoxide (Librium) and phenytoin (Dilantin)

ANS: D The nurse should anticipate that a physician would order chlordiazepoxide (Librium) and phenytoin (Dilantin) for a client who has a history of complicated withdrawal from benzodiazepines. It is common for long-lasting benzodiazepines to be prescribed for substitution therapy. Phenytoin (Dilantin) is an anticonvulsant that would be indicated for a client who has experienced a complicated withdrawal. Complicated withdrawals may progress to seizure activity.

Which client statement indicates a knowledge deficit related to substance abuse? A. "Although it's legal, alcohol is one of the most widely abused drugs in our society." B. "Tolerance to heroin develops quickly." C. "Flashbacks from LSD use may reoccur spontaneously." D. "Marijuana is like smoking cigarettes. Everyone does it. It's essentially harmless."

ANS: D The nurse should determine that the client has a knowledge deficit related to substance abuse when the client compares marijuana to smoking cigarettes and claims it to be harmless. Cannabis is the second most widely abused drug in the United States.

4. Which client statement indicates a knowledge deficit related to substance use? A. Although its legal, alcohol is one of the most widely abused drugs in our society. B. Tolerance to heroin develops quickly. C. Flashbacks from LSD use may reoccur spontaneously. D. Marijuana is like smoking cigarettes. Everyone does it. Its essentially harmless.

ANS: D The nurse should determine that the client has a knowledge deficit related to substance use when the client compares marijuana to smoking cigarettes and claims it to be harmless. Both of these substances have potentially harmful effects. Cannabis is the second most widely abused drug in the United States.

16. A nurse is interviewing a client in an outpatient drug treatment clinic. To promote success in the recovery process, which outcome should the nurse expect the client to initially accomplish? A. The client will identify one person to turn to for support. B. The client will give up all old drinking buddies. C. The client will be able to verbalize the effects of alcohol on the body. D. The client will correlate life problems with alcohol use.

ANS: D To promote the recovery process the nurse should expect that the client would initially correlate life problems with alcohol use. Acceptance of the problem is the first step of the recovery process.

A nurse is interviewing a client in an outpatient substance-abuse clinic. To promote success in the recovery process, which outcome should the nurse expect the client to initially accomplish? A. The client will identify one person to turn to for support. B. The client will give up all old drinking buddies. C. The client will be able to verbalize the effects of alcohol on the body. D. The client will correlate life problems with alcohol use.

ANS: D To promote the recovery process the nurse should expect that the client would initially correlate life problems with alcohol use. Acceptance of the problem is the first step of the recovery process.

Hypothermia

Abnormally low body temp.

Bradycardia

Abnormally slow heart action.

A patient has been admitted with a diagnosis of cocaine poisoning. What does the nurse expect the patient's plan of care to include?

Administer IV diazepam. Perform a gastric lavage. Administer IV antipsychotic drugs.

An employee health nurse is providing care to a worker who was injured on the job. The client has a history of drug addiction and is currently enrolled in a 12-step recovery program. In order to determine whether the employee was impaired at the time of the accident, which diagnostic tool should the nurse use? A) Liver enzymes B) Stool guaiac C) Urine toxicology testing D) Hair testing

Answer: C Urine toxicology testing will determine whether the employee had drugs in his system during the shift in which the injury occurred. Hair testing can detect substance use for up to 90 days and is not an accurate tool to determine whether the employee was impaired at the time of the injury. Liver enzymes detect liver damage but are not specific to damage from substance abuse. A stool guaiac tests for blood.

The nurse is caring for a client who overdosed on LSD. Which intervention should the nurse​ provide? (Select all that​ apply.) Administer opioid antagonist as ordered. Reduce environmental stimuli. Have one person​ "talk down the​ client." Speak slowly and clearly with the client. Administer renal dialysis as prescribed.

Reduce environmental stimuli. Have one person​ "talk down the​ client." Speak slowly and clearly with the client. Renal dialysis is used for​ alcohol, barbiturate, or benzodiazepine​ overdose, and an opioid antagonist is used for​ heroin, meperidine,​ morphine, or methadone overdose. For LSD​ overdose, the nurse can expect a benzodiazepine or chloral hydrate to be used.

Which physiologic effects are likely to be observed in a patient after phentermine abuse?

Rhinorrhea Sleeplessness Decreased libido

Which of the following clients is exhibiting the effects of an alcohol use disorder? a) A 28-year-old woman who is experiencing the signs and symptoms of alcohol withdrawal b) A 22-year-old male student who now drinks nine or ten drinks in order to get the same effect that he used to get from drinking a six pack c) A 59-year-old woman whose recent CT scan reveals the presence of Wernicke's encephalopathy d) A 66-year-old man who has been admitted to the emergency department with apparent delirium tremens

b) A 22-year-old male student who now drinks nine or ten drinks in order to get the same effect that he used to get from drinking a six pack

When educating the public regarding substance-related disorders, the mental health nurse explains that the condition of drug dependence is a state defined as ... a) Physical effects resulting from the multiple episodes of substance use b) Chronic or recurrent drug intoxication, characterized by psychological and physical dependence c) Emotional dependence on a drug or a desire or compulsion to continue taking a drug d) Substance-seeking activities and pathological use patterns of the person using substances

b) Chronic or recurrent drug intoxication, characterized by psychological and physical dependence

The ingestion of mood-altering substances stimulates which neurotransmitter pathway in the limbic system to produce a "high" that is a pleasant experience? a) Norepinephrine b) Dopamine c) Serotonin d) Acetylcholine

b) Dopamine

When a client is working toward the prevention of an alcohol abuse relapse, the nurse is most therapeutic when a) Discussing the pros and the cons of alcohol abuse b) Helping the client identify positive coping mechanisms c) Planning strategies to prevent further relapses d) Monitoring the effects of treatment

b) Helping the client identify positive coping mechanisms

When conducting an adolescent drug awareness program, the nurse describes which of the following drugs as having the highest addiction potential? a) Marijuana and nicotine b) Heroin and morphine c) Cocaine and codeine d) Amphetamines and barbiturates

b) Heroin and morphine

All of the following are positive aspects regarding methadone for heroin addiction except ... a) It is controlled be a physician. b) It is available in IV form. c) It is available in tablet form. d) It is a legal medication.

b) It is available in IV form.

While working as a nurse in a hospital, you notice that a co-worker has unsteady gait and slurred speech. You suspect that your co-worker may be an impaired nurse. Which of the following would be the appropriate action to take? a) Ignore your co-worker's behavior until others notice the problem. b) Report your co-worker's behavior to your supervisor. c) Confront the co-worker with your suspicions. d) Ask your co-worker to perform a sobriety test.

b) Report your co-worker's behavior to your supervisor.

The nurse is counseling a group of clients recovering from substance abuse about the nature of denial. Which of the following interventions should the nurse teach the clients to use to help them gain insight into their denial? a) Teach them to have realistic expectations about themselves. b) Teach them to question why they feel threatened. c) Teach them meditation techniques. d) Teach them alternative coping strategies.

b) Teach them to question why they feel threatened.

A client drinks 24 oz of vodka every day, having gradually increased to this level over the past 2 to 3 years. The client continues to maintain a job and functions well in activities of daily living, relationships, and other aspects of social life. Which of the following terms should the nurse use to describe this drinking behavior? a) Intoxication b) Tolerance c) Withdrawal d) Abuse

b) Tolerance

Shaundra was admitted to the emergency department for intoxication with alcohol. She has an unsteady gait, myopathy, and neuropathy, and cannot remember past or recent events. When treated with thiamine, her symptoms greatly improve. Shaundra is most likely suffering from ... a) alcoholic dementia. b) Wernicke-Korsakoff syndrome. c) alcohol dependence with memory impairment. d) scurvy.

b) Wernicke-Korsakoff syndrome.

"Substance abuse" is a term that is used when ... a) a person uses drugs continuously despite negative consequences; the person does experience tolerance. b) a person uses alcohol or drugs repeatedly, but not compulsively or addictively; the person who is abusing substances will not experience withdrawal. c) a person uses any form of drug or alcohol. d) a person is addicted to drugs or alcohol and cannot stop using them.

b) a person uses alcohol or drugs repeatedly, but not compulsively or addictively; the person who is abusing substances will not experience withdrawal.

Studies evaluating the genetic bases of alcoholism have found that: biological offspring of alcoholics are about twice as likely to become alcoholics as people without such a family history. no genetic link appeared in any twin studies. children whose biological parents were alcoholics are much more likely to abuse alcohol as adults than are those whose biological parents were alcoholics. psychological factors are more important than genetic factors.

biological offspring of alcoholics are about twice as likely to become alcoholics as people without such a family history.

Alcohol use disorder is associated with memory problems, in particular, _____, periods of time during which the drinker cannot later remember what transpired while he or she was intoxicated. binges cravings blackouts hallucinations

blackouts

The nurse is conducting a health promotion class on drug awareness with a group of junior high school students. Which of the following teaching points should the nurse include in this session? a) "Heavy marijuana use has resulted in severe withdrawal symptoms in some people once they decide to quit." b) "Marijuana and heroin actually have a very similar potential for addiction." c) "Marijuana use can result in psychological dependence, which can have a very negative effect on your life." d) "Most people are unaware of the fact that marijuana is actually a very addictive drug."

c) "Marijuana use can result in psychological dependence, which can have a very negative effect on your life."

Supported by research results, the mental health nurse includes alcohol-related problem assessment questions when interviewing ... a) All adult females b) All males between the ages of 18 and 45 years of age c) All clients regardless of age and gender d) White males 45 to 65 years of age

c) All clients regardless of age and gender

The nurse is performing a history and physical examination on a client with chronic alcoholism. The client has a history of gastritis, esophagitis, elevated liver enzymes, cardiomyopathy, and pancreatitis. Which of these conditions are attributable to his history of alcohol abuse? a) Pancreatitis, esophagitis, gastritis, and elevated liver enzymes b) Pancreatitis and elevated liver enzymes c) All the conditions are attributable to the alcohol abuse d) Gastritis and elevated liver enzymes

c) All the conditions are attributable to the alcohol abuse

A client has been using paint thinner and glue to receive a "high." The nurse knows that these substances are ... a) Hypnotics b) Hallucinogens c) Inhalants d) Sedatives

c) Inhalants

When preparing to monitor a client scheduled for drug-managed substance detoxification, the nurse prepares the client for diagnostic testing to determine the functioning status primarily of the client's a) Lungs b) Stomach c) Liver d) Kidneys

c) Liver

A male in his 30s has been brought to the emergency department by emergency medical services with an apparent heroin overdose. In the immediate care of this client, what assessments should the nurse prioritize? a) Monitoring of fluid and electrolyte levels b) Musculoskeletal and integumentary assessments c) Neurological and respiratory assessments d) Psychosocial assessment and identification of support systems

c) Neurological and respiratory assessments

Which technique is a type of cognitive behavioral therapy? a) Music therapy b) Guided imagery c) Self-monitoring d) Distraction

c) Self-monitoring

Which of the following statements most accurately describes the etiology of substance-related disorders? a) The primary predictors of substance-related disorders are childhood trauma and parental abuse or neglect. b) Substance abuse is a learned behavior. c) Substance-related disorders result from the interplay of biologic, genetic, and psychosocial factors. d) Substance-related disorders are primarily a result of the presence of an individual's inherently addictive personality.

c) Substance-related disorders result from the interplay of biologic, genetic, and psychosocial factors.

The major difference between substance abuse and substance dependency is that a) people who abuse substances are often seen by their peers as "alcoholic." b) in substance dependence, the individual has shared with his or her significant others that there is definitely a problem. c) people who are dependent often lose their jobs or drop out of school. d) a person who is abusing does not have withdrawal symptoms, whereas the dependent person uses the substance despite significant functional difficulties and will have withdrawal.

d) a person who is abusing does not have withdrawal symptoms, whereas the dependent person uses the substance despite significant functional difficulties and will have withdrawal.

Safety is the nursing priority for a client who is at risk for alcohol withdrawal. A care plan for the client who is in withdrawal must include ... a) suicide precautions because suicide attempts are frequent during withdrawal. b) seizure precautions and vital signs. c) vital signs; medications as prescribed. d) observation for symptoms; vital signs; seizure and fall precautions; medications as ordered.

d) observation for symptoms; vital signs; seizure and fall precautions; medications as ordered.

John began drinking a six-pack of beer every day in his freshman year of college. By his sophomore year, he is drinking two six-packs to get the same effect. This phenomenon is best described as ... a) alcohol abuse b) dependence c) intoxication d) tolerance

d) tolerance

■■ Older adults who use substances are especially prone to falls and other injuries, memory loss, somatic reports (headaches), and changes in sleep patterns. ☐☐ Indications of alcohol use in the older adult may include a decrease in ability for self-care (functional status), urinary incontinence, and signs of dementia. ☐☐ Older adults may show effects of alcohol use at __ doses than younger adults. ☐☐ __ (the use of multiple medications), the potential interaction between substances and medication, in addition to age-related physiological changes, raises the likelihood of adverse effects, such as confusion and falls in older adult clients.

lower Polypharmacy

Treatments that target social factors are usually provided in groups. Which of these is NOT an example of group-based treatment? group therapy self-help groups methadone group family therapy

methadone group

A user's _____ to stop his or her substance abuse affects the ultimate success of that person's treatment. willpower intention dedication motivation

motivation

Which drug is NOT a depressant? nicotine opiates barbiturates alcohol

nicotine

Which disorder is NOT reported as being a common comorbid disorder with substance abuse? depression obsessive-compulsive disorder posttraumatic stress disorder attention-deficit/hyperactivity disorder

obsessive-compulsive disorder

According to the stages of change theory, an individual who does not admit that there is a problem and does not intend to change is considered to be in the _____ stage. precontemplation denial preparation maintenance

precontemplation

●● Care After Discharge ◯◯ Client Education ■■ Teach the client to recognize indications of __ and factors that contribute to relapse. ■■ Teach cognitive-behavioral techniques to help maintain sobriety and to create feelings of pleasure from activities other than using substances or from process addictions. ■■ Assist the client to develop communication skills to communicate with coworkers and family members while sober.

relapse

When the goal of treatment for substance abuse is abstinence: symptoms of withdrawal usually are minimal. harm reduction is a major part of the experience. relapse rates tend to be high. the success rate for curing the substance abuse is high

relapse rates tend to be high

A young man has been brought to the emergency department (ED) by police after exhibiting bizarre and violent behavior on the patio outside a restaurant. Upon questioning, the man's girlfriend admits that he had been smoking PCP with his friend. While in the ED, the man continues to exhibit signs of PCP-induced psychosis and has required physical restraints. What nursing outcome should the nurse prioritize in the care of this patient? a) The client will demonstrate appropriate social skills. b) The client will verbalize acceptance of responsibility for his behavior. c) The client will be physically safe and without injury. d) The client will establish a balance of rest, sleep, and activity.

c) The client will be physically safe and without injury.

Which of the following medications is used to prevent Wernicke-Korsakoff syndrome? a) Folic acid (Folate) b) Cyanocabalamin c) Thiamine (vitamin B1) d) Lorazepam (Ativan)

c) Thiamine (vitamin B1)

A client with chronic alcoholism has been found to have Korsakoff's psychosis. This irreversible complication is characterized by what? a) Inability to learn new skills and short-term memory loss b) Double vision and rapid eye movement c) Thiamine, or vitamin B1, deficiency d) Hypothalamic and mammillary body lesions

c) Thiamine, or vitamin B1, deficiency

A 51-year-old male has been admitted to the detoxification unit with acute symptoms of alcohol withdrawal. Nursing assessment is likely to reveal what? a) Anhidrosis, hypotonicity, and delusions b) Psychomotor hypoactivity, hypotension, and increased appetite c) Tremors, headache, flushed face, and hallucinations d) Hypomania, bradycardia, and generalized seizures

c) Tremors, headache, flushed face, and hallucinations

George is admitted to the detoxification unit on Sunday evening. His withdrawal symptoms from alcohol are most likely to begin, and he will likely need the most support, at what time? a) Friday evening b) Wednesday morning c) Tuesday evening d) Monday morning

c) Tuesday evening

●● Teamwork and Collaboration ◯◯ __ __, or __, means that an individual has both a mental illness, such as depression, as well as a problem with a substance use or addictive disorder. Both disorders need to be treated simultaneously and will require a team approach. ◯◯ Individual psychotherapies ■■ Cognitive behavioral therapies, such as relaxation techniques or cognitive reframing, can be used to decrease __ and __ __. ■■ Acceptance and commitment therapy (ACT) promotes acceptance of the client's experiences and promotes client commitment to positive behavior changes. ■■ Relapse prevention therapy (RPT) assists clients in identifying the __ for relapse and promotes behavioral self-control.

Dual diagnosis comorbidity anxiety change behavior potential

the nurse recalls that which cardiovascular effects are observed during cocaine toxicity?

Dysrhythmia Hypertension Myocardial infarction

"Substance abuse" is a term that is used when ... a) a person uses any form of drug or alcohol. b) a person is addicted to drugs or alcohol and cannot stop using them. c) a person uses alcohol or drugs repeatedly, but not compulsively or addictively; the person who is abusing substances will not experience withdrawal. d) a person uses drugs continuously despite negative consequences; the person does experience tolerance.

c) a person uses alcohol or drugs repeatedly, but not compulsively or addictively; the person who is abusing substances will not experience withdrawal.

The recommended first-line pharmacologic agents for managing severe alcohol withdrawal symptoms are a) serotonin reuptake inhibitors. b) major tranquilizers. c) benzodiazepines. d) atypical antipsychotics.

c) benzodiazepines.

●● Pharmacological therapy ◯◯ Medications ■■ Alcohol withdrawal - Diazepam (Valium), lorazepam (Ativan), __ (__), __ (__), __ (__) ■■ Alcohol abstinence - Disulfiram (Antabuse), naltrexone (Revia), acamprosate (Campral)

carbamazepine Tegretol clonidine Catapres chlordiazepoxide Librium

The nurse assesses a patient's tobacco use status. The patient does not smoke tobacco but is addicted to the use of snuff. Which should the nurse teach the patient is the most likely complication arising from the use of snuff?

cardiovascular disease

Stimulants are named for their effect on the _____ nervous system. autonomic sympathetic central parasympathetic

central

Amphetamines typically produce the same effects as _____, although these effects last longer. Ecstasy cocaine methamphetamines Ritalin

cocaine

It is common for people with _____ use disorder to also have _____ use disorder. cocaine; alcohol amphetamine; nicotine cocaine; Ecstasy marijuana; alcohol

cocaine; alcohol

Donovan and Jessor's study, which found that adolescents who later developed substance abuse were likely to exhibit problem behaviors such as drug and alcohol use, early sexual intercourse, and delinquent behavior, led to the _____ of substance abuse. gateway hypothesis diathesis‒stress model neuropsychosocial development model common liabilities model

common liabilities model

The model that explains how neurological, psychological, and social factors create a vulnerability to a variety of problematic behaviors, including substance abuse and dependence, is called the: gateway hypothesis. diathesis‒stress model. neuropsychosocial development model. common liabilities model.

common liabilities model.

What is addiction?

compulsive drug craving and use, despite adverse consequences

Which pattern BEST describes substance use such as that of Native American tribes that use peyote or psilocybin mushrooms (which, when eaten, produce vivid hallucinations) as part of sacred rituals? abstinence constrained common activity party use

constrained

According to the stages of change theory, in the _____ stage, the user admits that there is a problem but makes no actual behavioral changes; however, the user may consider taking action and making behavioral changes in the future. precontemplation contemplation preparation maintenance

contemplation

Alcohol use disorder is characterized by: craving, loss of control, physical dependence, and tolerance. tolerance and withdrawal. physical dependence, tolerance, and withdrawal. blackouts, tolerance, and withdrawal.

craving, loss of control, physical dependence, and tolerance.

A client can expect symptoms of alcohol withdrawal to begin how many hours after the last drink? a) 24 b) 2 c) 16 d) 8

d) 8

The mental health nurse recognizes that genetic intolerance of alcohol has been documented among ethnic a) Italians b) Africans c) Germans d) Asians

d) Asians

Disulfiram (Antabuse) has been prescribed for a client receiving treatment for alcoholism. Which of the following should be included in the client's plan of care? a) Limit alcohol consumption to a moderate level b) Have weekly blood alcohol levels drawn c) Do not drive heavy machinery d) Avoid all products containing alcohol

d) Avoid all products containing alcohol

Safe alcohol withdrawal usually is accomplished with the administration of which medication classification? a) Anticonvulsants b) Antidepressants c) Antipsychotics d) Benzodiazepines

d) Benzodiazepines

Which of the following is the most common defense mechanism used by a client diagnosed with a substance use problem? a) Repression b) Regression c) Suppression d) Denial

d) Denial

Clonidine (Catapres) is most effective for which of the following symptoms of opioid withdrawal? a) Anxiety b) Muscles aches c) Restlessness d) Diarrhea

d) Diarrhea

Suspicion that a nursing professional is impaired by a substance abuse problem is most supported by ... a) Spending a considerable amount of shift time off the unit b) Frequently calling off work for undefined illnesses c) Asking to be scheduled for weekend shifts as much as possible d) Having several clients complain that their pain medication is not working

d) Having several clients complain that their pain medication is not working

Which of the following is the best description of the term substance use disorders, according to the American Psychiatric Association? a) Indicates that the client does not experience tolerance and withdrawal symptoms b) Indicates that the client experiences tolerance and withdrawal symptoms c) Indicates that the client has a medical reason for taking the substance d) Is an umbrella term for substance abuse and substance dependence

d) Is an umbrella term for substance abuse and substance dependence

All of the following are positive aspects regarding methadone for heroin addiction except ... a) It is available in tablet form. b) It is a legal medication. c) It is controlled be a physician. d) It is available in IV form.

d) It is available in IV form.

A client admitted for acute alcohol intoxication begins to experience mild sweating, tachycardia, fever, and nausea and vomiting. Of the following, the drug treatment of choice would be what? a) Paxil b) Haldol c) Tegretol d) Librium

d) Librium

Which of the following medications is used to prevent alcohol withdrawal symptoms? a) Folic acid (Folate) b) Clonidine (Catapres) c) Naltrexone (ReVia) d) Lorazepam (Ativan)

d) Lorazepam (Ativan)

Which drug reverses opioid toxicity? a) Disulfiram (Antabuse) b) Methadone c) Clonidine (Catapres) d) Naloxone (Narcan)

d) Naloxone (Narcan)

Sandra was diagnosed with cocaine abuse at age 30. When she was 23, Sandra was diagnosed with major depressive episode, and she has continued to have depression off and on since then. Which of the following is the most correct statement? a) Sandra is probably using cocaine in combination with her antidepressants to try to get higher. b) Sandra will have a very poor prognosis for recovery if she discontinues cocaine because it will enhance her depressive symptoms. c) Sandra will need to have the cocaine abuse treated before anything can be done about her depression. d) Sandra is probably using cocaine to cope with her depressive symptoms, and she would be considered to have a dual diagnosis.

d) Sandra is probably using cocaine to cope with her depressive symptoms, and she would be considered to have a dual diagnosis.

A nurse who started recovering from alcohol abuse 3 months earlier is ready to return to work. When speaking with her therapist, she states she is nervous about how her co-workers will respond to her now that "they all know I'm a drunk." Which of the following diagnoses best targets the problem implicit in the nurse's remarks? a) Powerlessness related to perceived inability to change the opinions of others b) Ineffective coping related to long history of alcohol use c) Anxiety related to situational crisis d) Situational low self-esteem related to medical condition

d) Situational low self-esteem related to medical condition

A client tells the nurse that she went to a party and had several drinks. When she woke up the next morning, she could not remember driving home. What does the nurse suspect? a) The client was using other drugs in addition to the alcohol. b) The client has alcoholic amnestic disorder. c) The client has an underlying neurologic problem. d) The client had a blackout.

d) The client had a blackout.

A client with opioid addiction is prescribed methadone maintenance therapy. When explaining this treatment to the client, which of the following would the nurse need to keep in mind? a) Methadone is a not physiologically addictive. b) Methadone simulates the high of heroin. c) Methadone is a non-opioid drug. d) The drug helps to satisfy the craving for the opioid.

d) The drug helps to satisfy the craving for the opioid.

When assessing a client diagnosed with chronic alcoholism, the nurse addresses potential memory dysfunction by reviewing the client's serum level of ... a) Iron b) Vitamin E c) Vitamin C d) Thiamine

d) Thiamine

People addicted to alcohol and drugs have a rate of suicide that is what percentage higher than that of the general population? a) 20 b) 30 c) 40 d) 10

a) 20

A nurse in a clinic is caring for a client who has a history of alcohol abuse and reports bruising and frequent nosebleeds. For which of the following is the client at risk? a) Cirrhosis b) Diabetes c) Hepatitis A d) Malnutrition

A

A nurse is discussing cultural concepts in mental health nursing with nursing staff. When discussing Native American clients, the nurse should identify which of the following as an increased risk for this cultural group? a) Substance abuse b) Schizophrenia c) Personality disorders d) Eating disorders

A

A nurse is caring for a patient experiencing symptoms associated with morphine withdrawal. The patient has been abusing morphine injections for the past three years. To treat these symptoms, which drug(s) does the nurse expect the health care provider to prescribe?

Loperamide Acetaminophen

A patient that is receiving opioid therapy reports muscle pain, diarrhea, and sleeplessness. The nurse identifies that which medications are appropriate to be included in the patient's treatment plan?

Loperamide Acetaminophen

_____ refers to the set of symptoms that arise when a regular user decreases intake of the substance. Withdrawal Cessation Dependence Detoxification

Withdrawal

A nurse is conducting a seminar with a local parent group about substance use and abuse. The nurse determines that the seminar was successful when the group identifies which of the following as the substance most abused in the United States? a) Alcohol b) Heroin c) Marijuana d) Cocaine

a) Alcohol

A client who has used IV heroin every day for the past 10 years says, "I don't have a drug problem. I can quit whenever I want." Which of the following defense mechanisms is being used by the client? a) Denial b) Compensation c) Repression d) Rationalization

a) Denial

Tracy consumed alcohol almost daily while she was pregnant. Her newborn baby has growth deficiency and facial malformations. What is the name for the pattern of birth defects that can occur due to exposure to alcohol? a) Fetal alcohol syndrome b) Natal alcohol syndrome c) Fatal alcohol syndrome d) Prenatal alcohol syndrome

a) Fetal alcohol syndrome

Binge drinking, or heavy episodic drinking, occurs when a person drinks until his or her blood alcohol concentration reaches at least _____ percent in a 2-hour period. 0.05 0.07 0.08 1.00

0.08

A nurse is performing a psychosocial assessment of a client who has a history of alcohol abuse. Which of the following questions asked by the nurse indicates that the client may have a Substance Use Disorder? a) "Has alcohol use affected your performance at work?" b) "Have you received prior mental health treatment?" c) "Do you receive treatment for any mental health disorders?" d) "At what age did you begin drinking alcohol?"

A

A nurse is teaching a community education course about the physical complications related to substance abuse. Which of the following should the nurse identify as a primary cause of cirrhosis? a) Alcohol b) Caffeine c) Cocaine d) Inhalants

A

A nurse manager is preparing to confront a staff nurse who is abusing alcohol. Which of the following responses should the nurse manager expect? a) Denial b) Repression c) Rationalization d) Projection

A

Which is the priority nursing diagnosis for an individual experiencing alcohol withdrawal? A) Risk for injury B) Impaired thought C) Ineffective coping D) Ineffective denial

A

The nurse is assessing a client she suspects may have a drinking problem. She asks the client how much alcohol he drinks per day. The client replies, "The important point is that even if I have 10 drinks, I don't get drunk. I can hold my liquor." The client's reply suggests which of the following? a) He has become tolerant to alcohol. b) He is minimizing his intake. c) He has blackouts. d) He may drink frequently or a lot, but he doesn't have a problem.

a) He has become tolerant to alcohol.

After educating a group of students about caffeine, the instructor determines that the education was successful when the group identifies which of the following as indicating intoxication? Select all that apply. a) Insomnia b) Pale face c) Nervousness d) Urinary retention e) Restlessness f) Excitement

a) Insomnia c) Nervousness d) Urinary retention e) Restlessness f) Excitement

When discussing methadone treatment with a client, the nurse teaches what? a) It decreases the severity of heroin withdrawal symptoms. b) The cure rate is extremely high. c) It is a nonaddictive treatment. d) It takes 1 to 2 years to cure an opiate addict.

a) It decreases the severity of heroin withdrawal symptoms.

After a long history of intravenous heroin use, a client has expressed willingness to stop using heroin. The nurse would expect the client to receive which medication to decrease the severity of withdrawal? a) Methadone b) Gabapentin (Neurontin) c) Ondansetron (Zofran) d) Amphetamines

a) Methadone

When educating the public concerning marijuana use, the nurse stresses that heavy, regular use is likely to result in a) Psychological dependence with an increased risk for self-harm b) Accidental drug poisoning and tachycardia c) Both physiological and psychological dependence and an increased risk for respiratory problems d) Extraordinary pain tolerance and extreme hypertension

a) Psychological dependence with an increased risk for self-harm

Which of the following terms describes a situation that occurs when very small amounts of alcohol intoxicates the person after continued heavy drinking? a) Tolerance break b) Intoxication c) Tolerance d) Blackout

a) Tolerance break

"A client with a history of alcoholism is seen in the clinic, and plans are made to begin treatment with disulfiram (Antabuse). Before therapy is initiated, the nurse should first determine: If the client agrees to reduce alcohol intake. If a history of migraine headaches is present. If the client understands that the drug is addictive. If the client is pregnant.

" If the client is pregnant. Objective: Describe the role of the nurse in delivering care to individuals who have substance abuse issues. Rationale: Antabuse is taken to discourage relapses of alcohol intake. It is a pregnancy risk category X drug, and should never be taken if the client is pregnant. Alcohol should not be ingested while taking Antabuse; it causes severe headache, nausea, and vomiting. It is not addictive. Cognitive Level: Application Client Need: Physiological Integrity: Pharmacological and Parenteral Therapies Nursing Process: Assessment "

"Following a surgical procedure, the client states that he does not want to take narcotic pain analgesics because he is afraid he will become addicted to the drug. The response by the nurse is based on the knowledge that: A> Dependence on narcotics frequently occurs among postoperative clients. B> Female clients are more likely to become addicted. C> Addiction to prescription drugs is rare when they are used according to protocol. D> Addiction is rare if the client uses the narcotic for only 2-3 days.

"Addiction to prescription drugs is rare when they are used according to protocol. Objective: Explain the underlying causes of addiction. Rationale: Some clients and health professionals believe that therapeutic use of scheduled drugs creates large numbers of addicted clients. Prescription drugs rarely cause addiction when used according to accepted medical protocols. The risk of addiction for prescription medications is primarily a function of the dose and the length of therapy. Cognitive Level: Application Client Need: Physiological Integrity: Pharmacological and Parenteral Therapies Nursing Process: Analysis "

A pregnant woman​ states, "It is a myth that a glass of wine a day will harm the baby. People in other countries do it​ often, and their babies are​ fine." Which response by the nurse reflects​ evidence-based practice? ​"It is best to not drink while you are​ pregnant, but it will be fine when you are​ breastfeeding." ​"It is irresponsible to drink any amount of alcohol while you are​ pregnant; it will cause fetal abnormalities in all​ cases." ​"Women who drink one drink per week are at risk for delivering a stillborn baby. It is important to protect the​ well-being of your fetus at all​ times." ​"Alcohol use is linked to an increased risk of miscarriage and increased rates of prematurity and abruptio​ placentae."

"Alcohol use is linked to an increased risk of miscarriage and increased rates of prematurity and abruptio​ placentae."

"The nurse giving discharge teaching for a client prescribed alprazolam (Xanax) for treatment of anxiety should include what information? Take the drug only at bedtime. Monitor respiratory rate. Monitor blood pressure closely. Avoid combining the drug with alcohol.

"Avoid combining the drug with alcohol. Objective: Describe the role of the nurse in delivering care to individuals who have substance abuse issues. Rationale: Alprazolam (Xanax) is a benzodiazepine drug. Abuse of these drugs is uncommon; however, they frequently are mixed with alcohol to augment the drug experience. Overdose from this combination can be fatal. Cognitive Level: Application Client Need: Physiological Integrity: Pharmacological and Parenteral Therapies Nursing Process: Implementation "

"The nurse assesses a client suspected of a drug overdose. Because the client has a runny nose, crusty redness around the nostrils, and deterioration of the nasal cartilage, the nurse suspects the client has abused which drug? Crack Cocaine Ecstasy Heroin

"Cocaine" Objective: Explain the major characteristics of abuse, dependence, and tolerance resulting from the following substances: alcohol, nicotine, marijuana, hallucinogens, CNS stimulants, sedatives, and opioids. Rationale: Cocaine is the second most commonly used illicit drug in the United States. Routes of administration include snorting, smoking, and injecting. Users who snort cocaine develop a chronic runny nose, a crusty redness around the nostrils, and deterioration of the nasal cartilage. Overdose can result in dysrhythmias, convulsions, stroke, or death due to respiratory arrest. Cognitive Level: Application Client Need: Physiological Integrity: Pharmacological and Parenteral Therapies Nursing Process: Evaluation "

"The client states that he is going to quit smoking ""cold turkey."" The nurse teaches the client to expect which of the following symptoms during withdrawal from nicotine? Muscle spasms; hypertension; tremors Depression; hallucinations; anorexia Headaches; increased appetite; insomnia Excessive sweating; fatigue; seizures

"Headaches; increased appetite; insomnia Objective: Explain the major characteristics of abuse, dependence, and tolerance resulting from the following substances: alcohol, nicotine, marijuana, hallucinogens, CNS stimulants, sedatives, and opioids. Rationale: Symptoms of nicotine withdrawal include irritability, anxiety, restlessness, headaches, increased appetite, insomnia, inability to concentrate, and decrease in heart rate and blood pressure. Cognitive Level: Application Client Need: Physiological Integrity: Pharmacological and Parenteral Therapies Nursing Process: Implementation

"A client presents to the Emergency Department with euphoria, paranoia, and complaints of severe thirst and hunger. He insists that the nurse give him chocolate for his hunger. The nurse suspects substance abuse of what drug? Heroin Barbiturates Marijuana Crack

"Marijuana Objective: Explain the major characteristics of abuse, dependence, and tolerance resulting from the following substances: alcohol, nicotine, marijuana, hallucinogens, CNS stimulants, sedatives, and opioids. Rationale: Use of marijuana slows motor activity; decreases coordination; and causes disconnected thoughts, feelings of paranoia, and euphoria. It increases thirst and craving for food, especially chocolate and other candies. Cognitive Level: Analysis Client Need: Physiological Integrity: Pharmacological and Parenteral Therapies Nursing Process: Assessment "

"The student nurse asks the registered nurse to explain the difference between physical and psychological dependence on a drug. Which of the following responses by the nurse is accurate? 1> Physical dependence can occur quickly with repeated doses, especially with IV drugs. 2> Physical dependence is the usual factor in a client's relapse during addiction therapy. 3> Psychological dependence does not interfere with the client's lifestyle. 4> Psychological dependence results in the development of withdrawal symptoms if the substance is discontinued.

"Physical dependence can occur quickly with repeated doses, especially with IV drugs. Objective: Compare and contrast psychological and physical dependence. Rationale: Physical dependence refers to an altered physical condition caused by the nervous system's adapting to repeated substance use. Over time, the body's cells are tricked into believing that they are normal in the presence of the substance. Withdrawal symptoms occur when the substance is discontinued. Cognitive Level: Comprehension Client Need: Physiological Integrity: Pharmacological and Parenteral Therapies Nursing Process: Implementation "

"The client with a history of alcohol abuse is admitted to the hospital. The nursing care plan includes assessment of the client for which of the following symptoms indicative of alcohol withdrawal? Weakness; hypotension; violent yawning Tremors; hallucinations; delirium Mental depression; headaches; hunger Insomnia; nausea; bradycardia

"Tremors; hallucinations; delirium Objective: Describe the role of the nurse in delivering care to individuals who have substance abuse issues. Rationale: Clients experiencing alcohol withdrawal typically experience tremors, fatigue, anxiety, abdominal cramping, hallucinations, confusion, seizures, and delirium. Cognitive Level: Application Client Need: Physiological Integrity: Pharmacological and Parenteral Therapies Nursing Process: Assessment "

A patient presents to the clinic with a report of fatigue and difficulty concentrating. Which additional statement made by the patient would alert the healthcare provider to possible marijuana use? -"I feel nauseous and don't feel like eating." -"I feel anxious and have trouble sleeping." -"I've noticed that my eyes are red lately." -"I keep having really vivid and scary nightmares."

-"I've noticed that my eyes are red lately."

A patient reports smoking 10 cigarettes per day for 40 years. How will the healthcare provider document this patient's smoking habit in terms of pack years? -20 pack years -10 pack years -4 pack years -5 pack years

-5 pack years

A patient is brought to the emergency department (ED) by a friend. The patient is unresponsive and respirations are slow and shallow. Which of the following is the priority intervention? -Check the patient's blood glucose level -Administer naloxone, per protocol -Administer 100% oxygen per nasal cannula -Ask the friend if they were using illicit drugs

-Administer 100% oxygen per nasal cannula

Which of the following goals would the healthcare provider identify as realistic for a patient with a substance abuse problem? -Explore genetic anomalies associated with substance abuse -Use the substance only in moderation and in certain situations -Identify situations that trigger a desire to use the substance -Focus on how cravings can be eliminated by enhancing willpower

-Identify situations that trigger a desire to use the substance

A patient who overdosed on oxycodone is given naloxone. When assessing the patient, the healthcare provider would anticipate which of these clinical manifestations of opioid withdrawal? -Bradycardia and hyporthermia -Irritability and nausea -Hyperthermia and euphoria -Depressed respirations and somnolence

-Irritability and nausea

Emergency medical personnel bring an unconscious patient to the emergency department. The patient's pupils are pinpoint and respirations are depressed. Intoxication of which of the following substances could contribute to these clinical signs? -Methamphetamine -Methadone -Cocaine -Ecstasy

-Methadone

A patient is brought to the emergency department by a family member. The patient has been agitated for the past several hours and has alternated between grandiosity and expressing a desire to commit suicide. Upon examination, the patient is diaphoretic, hypertensive, and tachycardic. Intoxication with which of the following substances would contribute to these symptoms? -Methamphetamine -Benzodiazepine -Marijuana -Alcohol

-Methamphetamine

A patient is admitted to the medical unit after experiencing chest pain. Which of these additional findings would support a diagnosis of cocaine abuse? -Jaundice -Hypotension -Perforated nasal septum -Profuse diarrhea

-Perforated nasal septum

A patient who has a history of chronic back pain requires a higher dose of an opioid medication in order to achieve adequate pain relief. The healthcare provider suspects that these findings are a result of which of the following? -Dependence -Pseudoaddiction -Addiction -Tolerance

-Tolerance

Which of the following assessment findings in a patient's health history supports a diagnosis of substance dependence? -Numerous legal problems and interpersonal conflicts -Withdrawal symptoms when not using the substance -Impaired judgment and risk-taking behaviors -Continued tardiness and absenteeism from work

-Withdrawal symptoms when not using the substance

Some clinicians avoid the term _____, which is less exact than _____. substance dependence; intoxication substance dependence; substance abuse addiction; intoxication addiction; substance use disorder

addiction; substance use disorder

A client has received chlordiazepoxide (Librium) to control the symptoms of alcohol withdrawal. The chlordiazepoxide has been ordered as needed. The nurse assesses the client and determines an additional dose of medication is needed when the client displays which symptoms? Select all that apply. 1. Tachycardia 2. Mood swings 3. Elevated blood pressure and temperature 4. Piloerection 5. Tremors 6. Increasing anxiety

1, 3, 5, and 6. Benzodiazepines are usually administered based on elevations in heart rate, blood pressure, and temperature as well as on the presence of tremors and increasing anxiety. Mood swings are expected during the withdrawal period and are not an indication for further medication administration. Piloerection is not a symptom of alcohol withdrawal.

A client has received chlordiazepoxide (Librium) to control the symptoms of alcohol withdrawal. The chlordiazepoxide has been ordered as needed. The nurse assesses the client and determines an additional dose of medication is needed when the client displays which symptoms? Select all that apply. 1. Tachycardia 2. Mood swings 3. Elevated blood pressure and temperature 4. Piloerection 5. Tremors 6. Increasing anxiety

1, 3, 5, and 6. Benzodiazepines are usually administered based on elevations in heart rate, blood pressure, and temperature as well as on the presence of tremors and increasing anxiety. Mood swings are expected during the withdrawal period and are not an indication for further medication administration. Piloerection is not a symptom of alcohol withdrawal. Lisko, Susan (2013-10-01). NCLEX-RN Questions and Answers Made Incredibly Easy (Nclexrn Questions & Answers Made Incredibly Easy) (Kindle Locations 16639-16641). Lippincott Williams & Wilkins. Kindle Edition.

A client has received chlordiazepoxide (Librium) to control the symptoms of alcohol withdrawal. The chlordiazepoxide has been ordered as needed. The nurse assesses the client and determines an additional dose of medication is needed when the client displays which symptoms? Select all that apply. 1. Tachycardia 2. Mood swings 3. Elevated blood pressure and temperature 4. Piloerection 5. Tremors 6. Increasing anxiety

1, 3, 5, and 6. Benzodiazepines are usually administered based on elevations in heart rate, blood pressure, and temperature as well as on the presence of tremors and increasing anxiety. Mood swings are expected during the withdrawal period and are not an indication for further medication administration. Piloerection is not a symptom of alcohol withdrawal. Lisko, Susan (2013-10-01). NCLEX-RN Questions and Answers Made Incredibly Easy (Nclexrn Questions & Answers Made Incredibly Easy) (Kindle Locations 16639-16641). Lippincott Williams & Wilkins. Kindle Edition.

A nurse has developed a relationship with a client who has an addiction problem. The nurse determines that the therapeutic interaction is in the working stage when the client does what? Select all that apply. 1. The client addresses how the addiction has contributed to family distress. 2. The client reluctantly shares the family history of addiction. 3. The client verbalizes difficulty identifying personal strengths. 4. The client discusses financial problems related to the addiction. 5. The client expresses uncertainty about meeting with the nurse. 6. The client acknowledges the addiction's effects on the children.

1, 3, and 6. These statements are indicative of the nurse- client working phase, in which the client explores, evaluates, and determines solutions to identified problems. The remaining statements address what happens during the introductory phase of the nurse- client interaction.

When assessing a client with prolonged, chronic alcohol intake, the nurse would expect to find which of the following? 1. Enlarged liver 2. Nasal irritation 3. Muscle wasting 4. Limb paresthesia

1. A major effect of alcohol on the body is liver impairment, and an enlarged liver is a common physical finding. Nasal irritation is commonly seen with clients who snort cocaine. Muscle wasting and limb paresthesia don't tend to occur with clients who abuse alcohol. Lisko, Susan (2013-10-01). NCLEX-RN Questions and Answers Made Incredibly Easy (Nclexrn Questions & Answers Made Incredibly Easy) (Kindle Locations 15881-15883). Lippincott Williams & Wilkins. Kindle Edition.

A client can expect symptoms of alcohol withdrawal to begin how many hours after the last drink? a) 2 b) 8 c) 24 d) 16

b) 8

1. Family members of an alcoholic client ask the nurse to help them intervene. Which action is essential for a successful intervention? 1. All family members must tell the client they're powerless. 2. All family members must describe how the addiction affects them. 3. All family members must come up with their share of financial support. 4. All family members must become caregivers during the detoxification period.

1. 2. After the family is taught about addiction, they must write down examples of how the addiction has affected each of them and use this information during the intervention. It isn't necessary to tell the client the family is powerless. The family is empowered through this intervention experience. In many cases, a third-party payer will help with treatment costs. Doing an intervention doesn't make family members responsible for financial support or providing care and support during the detoxification period. Lisko, Susan (2013-10-01). NCLEX-RN Questions and Answers Made Incredibly Easy (Nclexrn Questions & Answers Made Incredibly Easy) (Kindle Locations 15840-15844). Lippincott Williams & Wilkins. Kindle Edition.

A client who smoked marijuana daily for 10 years tells a nurse, "I don't have any goals, and I just don't know what to do." What is the most appropriate nursing intervention for this client? 1. Focus the interaction. 2. Use nonverbal methods. 3. Use reflection techniques. 4. Ask open-ended questions.

1. A client with amotivational syndrome from chronic use of marijuana tends to talk in tangents and needs the nurse to focus the conversation. Nonverbal communication or reflection techniques wouldn't be useful as this client must focus and learn to identify and accomplish goals. Using only open-ended questions won't allow the client to focus and establish specific goals. Lisko, Susan (2013-10-01). NCLEX-RN Questions and Answers Made Incredibly Easy (Nclexrn Questions & Answers Made Incredibly Easy) (Kindle Locations 16420-16423). Lippincott Williams & Wilkins. Kindle Edition.

A client tells the nurse, "I have been drinking ever since they told me I had learning disabilities." How does the nurse interpret this response? 1. The client is self-medicating. 2. The client has an excuse to drink. 3. The client isn't a productive person. 4. The client will be unable to stop drinking.

1. A client with learning disabilities may experience frustration, depression, or overall feelings of low self-esteem and may self-medicate with alcohol. Many people with learning disabilities don't resort to alcohol but develop other coping skills to handle the disability. People with learning disabilities can be very productive. A person with a learning disability can successfully recover from alcohol addiction. CN: Psychosocial integrity; CNS: None; CL: Lisko, Susan (2013-10-01). NCLEX-RN Questions and Answers Made Incredibly Easy (Nclexrn Questions & Answers Made Incredibly Easy) (Kindle Locations 15982-15986). Lippincott Williams & Wilkins. Kindle Edition.

A client who uses cocaine finally admits he also abused other drugs to equalize the effect of cocaine. The nurse is aware that the client's drug history may include which substance? 1. Alcohol 2. Amphetamines 3. Caffeine 4. Phencyclidine

1. A cocaine addict will commonly use alcohol to decrease or equalize the stimulating effects of cocaine. Caffeine, phencyclidine, and amphetamines aren't used to equalize the stimulating effects of cocaine. Lisko, Susan (2013-10-01). NCLEX-RN Questions and Answers Made Incredibly Easy (Nclexrn Questions & Answers Made Incredibly Easy) (Kindle Locations 16373-16374). Lippincott Williams & Wilkins. Kindle Edition.

When assessing a client with prolonged, chronic alcohol intake, the nurse would expect to find which of the following? 1. Enlarged liver 2. Nasal irritation 3. Muscle wasting 4. Limb paresthesia

1. A major effect of alcohol on the body is liver impairment, and an enlarged liver is a common physical finding. Nasal irritation is commonly seen with clients who snort cocaine. Muscle wasting and limb paresthesia don't tend to occur with clients who abuse alcohol. Lisko, Susan (2013-10-01). NCLEX-RN Questions and Answers Made Incredibly Easy (Nclexrn Questions & Answers Made Incredibly Easy) (Kindle Locations 15881-15883). Lippincott Williams & Wilkins. Kindle Edition.

43. The nurse is assessing a client who is a substance abuser. The client states, "I use every day, but it rarely interferes with my work." The nurse determines that the client is using which defense mechanism? 1) Projection 2) Denial 3) Reaction formation 4) Displacement

2) Denial Denial is characterized by avoidance of disagreeable realities and unconscious refusal to acknowledge a thought, feeling, need, or desire. By stating that alcohol use rarely interferes with his or her work, the client is denying a substance abuse problem.

1. A nurse is planning a staff education program on substance use in older adults. Which of the following is appropriate for the nurse to include in the presentation? A. Older adults require higher doses of a substance to achieve a desired effect. B. Older adults commonly use rationalization to cope with a substance use disorder. C. Older adults are at a higher risk for substance use following retirement. D. Older adults develop substance use to mask signs of dementia.

1. A. INCORRECT: Requiring higher doses of a substance to achieve a desired effect is a result of the length and severity of substance use rather than age. B. INCORRECT: Denial, rather than rationalization, is a defense mechanism commonly used by substance users of all ages. C. CORRECT: Retirement, and other life change stressors, increases the risk for substance use in older adults, especially if there is a prior history of substance use. D. INCORRECT: Substance use in the older adult can result in signs of dementia.

7. A client who abuses alcohol tells a nurse, "Alcohol helps me sleep." What is the most appropriate response by the nurse? 1. " Alcohol doesn't help promote sleep." 2. " Continued alcohol use causes insomnia." 3. " One glass of alcohol at dinnertime can induce sleep." 4. " Sometimes, alcohol can make one drowsy enough to fall asleep."

1. Alcohol use may initially promote sleep, but with continued use, it causes insomnia. Evidence shows that alcohol doesn't facilitate sleep. One glass of alcohol at dinnertime won't induce sleep. The last option doesn't give information about how alcohol affects sleep. It makes the client think alcohol use to induce sleep is an appropriate strategy to try. Lisko, Susan (2013-10-01). NCLEX-RN Questions and Answers Made Incredibly Easy (Nclexrn Questions & Answers Made Incredibly Easy) (Kindle Locations 15901-15903). Lippincott Williams & Wilkins. Kindle Edition.

45. Which primary factor is critical in maintaining abstinence for the client diagnosed with alcohol use disorder? 1) Attendance at Alcoholics Anonymous (AA) meetings 2) Personal commitment to change 3) Family involvement 4) Compliance with pharmacological therapy

2) Personal commitment to change The first step in the recovery process necessitates that the client accept ownership of the problem and establish a behavioral change commitment to continued abstinence.

A client is receiving chlordiazepoxide (Librium) as needed for signs and symptoms of alcohol withdrawal. The nurse assesses the client and determines the need for medication when the client displays: 1. mild tremors, hypertension, tachycardia. 2. bradycardia, hyperthermia, sedation. 3. hypotension, decreased reflexes, drowsiness. 4. hypothermia, mild tremors, slurred speech.

1. Chlordiazepoxide is given during alcohol withdrawal. Symptoms that indicate a need for this drug include tremors, hypertension, tachycardia, and elevated body temperature. Bradycardia, sedation, hypotension, decreased reflexes, hypothermia, and slurred speech aren't symptoms of alcohol withdrawal. Lisko, Susan (2013-10-01). NCLEX-RN Questions and Answers Made Incredibly Easy (Nclexrn Questions & Answers Made Incredibly Easy) (Kindle Locations 15891-15893). Lippincott Williams & Wilkins. Kindle Edition.

A client is receiving chlordiazepoxide (Librium) as needed for signs and symptoms of alcohol withdrawal. The nurse assesses the client and determines the need for medication when the client displays: 1. mild tremors, hypertension, tachycardia. 2. bradycardia, hyperthermia, sedation. 3. hypotension, decreased reflexes, drowsiness. 4. hypothermia, mild tremors, slurred speech.

1. Chlordiazepoxide is given during alcohol withdrawal. Symptoms that indicate a need for this drug include tremors, hypertension, tachycardia, and elevated body temperature. Bradycardia, sedation, hypotension, decreased reflexes, hypothermia, and slurred speech aren't symptoms of alcohol withdrawal. Lisko, Susan (2013-10-01). NCLEX-RN Questions and Answers Made Incredibly Easy (Nclexrn Questions & Answers Made Incredibly Easy) (Kindle Locations 15891-15893). Lippincott Williams & Wilkins. Kindle Edition.

A client who has been drinking alcohol for 30 years asks a nurse if permanent damage has occurred to his immune system. What is the best response by the nurse? 1. " There is often less resistance to infections." 2. " Sometimes, the body's metabolism will increase." 3. " Put your energies into maintaining sobriety for now." 4. " Drinking puts you at high risk for disease later in life."

1. Chronic alcohol use depresses the immune system and causes increased susceptibility to infections. A nutritionally well-balanced diet that includes foods high in protein and B vitamins will help develop a strong immune system. The potential damage to the immune system doesn't increase the body's metabolism. The third option negates the client's concern and isn't an appropriate or caring response. Drinking alcohol may put the client at risk for immune system problems at any time in life. CN: Psychosocial integrity; CNS: None; CL: Analysis Lisko, Susan (2013-10-01). NCLEX-RN Questions and Answers Made Incredibly Easy (Nclexrn Questions & Answers Made Incredibly Easy) (Kindle Locations 16045-16049). Lippincott Williams & Wilkins. Kindle Edition.

A client who has been drinking alcohol for 30 years asks a nurse if permanent damage has occurred to his immune system. What is the best response by the nurse? 1. " There is often less resistance to infections." 2. " Sometimes, the body's metabolism will increase." 3. " Put your energies into maintaining sobriety for now." 4. " Drinking puts you at high risk for disease later in life."

1. Chronic alcohol use depresses the immune system and causes increased susceptibility to infections. A nutritionally well-balanced diet that includes foods high in protein and B vitamins will help develop a strong immune system. The potential damage to the immune system doesn't increase the body's metabolism. The third option negates the client's concern and isn't an appropriate or caring response. Drinking alcohol may put the client at risk for immune system problems at any time in life. CN: Psychosocial integrity; CNS: None; CL: Analysis Lisko, Susan (2013-10-01). NCLEX-RN Questions and Answers Made Incredibly Easy (Nclexrn Questions & Answers Made Incredibly Easy) (Kindle Locations 16045-16049). Lippincott Williams & Wilkins. Kindle Edition.

A polyaddicted client is hospitalized for withdrawal complications. What is the most important goal for this client? 1. The client will remain safe during the detoxification period. 2. The client will develop an accurate perception of his drug problem. 3. The client will abstain from mood-altering drugs. 4. The client will learn coping strategies to help him stop relying on drugs.

1. Client safety takes highest priority during detoxification. During this time, it's unrealistic to expect clients to perceive their drug problems accurately; typically, they experience cognitive impairment or deny their addiction. In the hospital, the client usually doesn't have access to drugs and should be drug free; the goal of abstaining from mood-altering drugs takes highest priority after discharge. Learning coping strategies is an appropriate goal immediately after withdrawal and when medical care is completed. Lisko, Susan (2013-10-01). NCLEX-RN Questions and Answers Made Incredibly Easy (Nclexrn Questions & Answers Made Incredibly Easy) (Kindle Locations 16603-16607). Lippincott Williams & Wilkins. Kindle Edition.

The specific effects of substance _____ depend on the substance and whether a person uses it only occasionally (e.g., getting drunk on Saturday night) or chronically (e.g., drinking to excess every night). use abuse intoxication dependence

intoxication

Whereas _____ refers to the direct results of using a substance, the criteria of a substance use disorder focus both on the indirect effects of repeated use, such as unmet obligations or risky behavior while using the substance (for instance, driving while under the influence), and the cravings and biological changes that can occur with repeated use or stopping use. dependence intoxication withdrawal toleration

intoxication

The nurse completed a health history on a client addicted to nicotine. Which info is needed when collecting info about past nicotine use? SATA A) Cost of nicotine use B) Frequency of nicotine use C) Brand of nicotine use D) Duration of nicotine use E) Amount of nicotine us

B, D, E Rationale: When obtaining a health​ history, it is important to collect information on past nicotine​ use, such as​ amount, frequency, and duration. The brand and the cost of nicotine used are not important factors when obtaining the health history. OK

17. During a family therapy session, an alcoholic client tells a family member, "You made it easy for me to use alcohol. You always made excuses for my behavior." What should the nurse encourage the family to do? 1. Give up enabling behaviors 2. Manage the client's self-care 3. Deal with negative behaviors 4. Evaluate the home environment

1. Enabling the behaviors of family members allows the client to continue the addiction by rationalizing, denying, or otherwise excusing the problem. Managing the client's self-care isn't an issue that needs to be addressed based on the client's statement. Dealing with negative behaviors and evaluating the home environment don't address the client's statement about the family's enabling behavior. Lisko, Susan (2013-10-01). NCLEX-RN Questions and Answers Made Incredibly Easy (Nclexrn Questions & Answers Made Incredibly Easy) (Kindle Locations 16002-16005). Lippincott Williams & Wilkins. Kindle Edition.

17. During a family therapy session, an alcoholic client tells a family member, "You made it easy for me to use alcohol. You always made excuses for my behavior." What should the nurse encourage the family to do? 1. Give up enabling behaviors 2. Manage the client's self-care 3. Deal with negative behaviors 4. Evaluate the home environment Lisko, Susan (2013-10-01). NCLEX-RN Questions and Answers Made Incredibly Easy (Nclexrn Questions & Answers Made Incredibly Easy) (Kindle Locations 15997-16002). Lippincott Williams & Wilkins. Kindle Edition.

1. Enabling the behaviors of family members allows the client to continue the addiction by rationalizing, denying, or otherwise excusing the problem. Managing the client's self-care isn't an issue that needs to be addressed based on the client's statement. Dealing with negative behaviors and evaluating the home environment don't address the client's statement about the family's enabling behavior. Lisko, Susan (2013-10-01). NCLEX-RN Questions and Answers Made Incredibly Easy (Nclexrn Questions & Answers Made Incredibly Easy) (Kindle Locations 16002-16005). Lippincott Williams & Wilkins. Kindle Edition.

Amphetamine use disorder can cause: reversible effects on perceptions and mental functioning. lethargy and suicidal thoughts. strokes. irreversible effects on memory and coordination.

irreversible effects on memory and coordination.

A nurse suggests to a client struggling with alcohol addiction that keeping a journal may be helpful. The goal of this nursing intervention is to help the client do what? 1. Identify stressors and responses to them. 2. Understand the diagnosis. 3. Help others by reading the journal to them. 4. Develop an emergency plan for use in a crisis. Lisko, Susan (2013-10-01). NCLEX-RN Questions and Answers Made Incredibly Easy (Nclexrn Questions & Answers Made Incredibly Easy) (Kindle Locations 16102-16106). Lippincott Williams & Wilkins. Kindle Edition.

1. Keeping a journal enables the client to identify problems and patterns of coping. From this information, the difficulties the client faces can be addressed. A journal isn't necessarily kept to promote better understanding of the client's illness, but it helps the client understand himself better. Journals aren't read to other people unless the client wants to share a particular part. Journals aren't typically used for identifying an emergency plan for use in a crisis. Lisko, Susan (2013-10-01). NCLEX-RN Questions and Answers Made Incredibly Easy (Nclexrn Questions & Answers Made Incredibly Easy) (Kindle Locations 16107-16110). Lippincott Williams & Wilkins. Kindle Edition.

The family of a client in rehabilitation following heroin withdrawal asks a nurse why the client is receiving naltrexone (ReVia). What is the best response by the nurse? 1. To help reverse withdrawal symptoms 2. To keep the client sedated during withdrawal 3. To take the place of detoxification with methadone 4. To decrease the client's memory of the withdrawal experience

1. Naltrexone is an opioid antagonist and helps the client stay drug free. Keeping the client sedated during withdrawal isn't the reason for giving this drug. The drug doesn't decrease the client's memory of the withdrawal experience and isn't used in place of detoxification with methadone.

A nurse is assessing a client with a history of substance abuse who has pinpoint pupils, a heart rate of 56 beats/ minute, a respiratory rate of 6 breaths/ minute, and temperature of 96.4 ° F. The nurse determines that which is the most likely cause of the client's symptoms? 1. Opioids 2. Amphetamines 3. Cannabis 4. Alcohol

1. Opioids, such as morphine and heroin, can cause pinpoint pupils and a reduced heart rate, respiratory rate, and body temperature with intoxication. Amphetamine intoxication can lead to tachycardia, euphoria, and irritability. Cannabis (marijuana) intoxication can cause slowed reflexes, lethargy, and tachycardia. Alcohol intoxication leads to slurred speech, unsteady gait, and uncoordination. Lisko, Susan (2013-10-01). NCLEX-RN Questions and Answers Made Incredibly Easy (Nclexrn Questions & Answers Made Incredibly Easy) (Kindle Locations 16345-16348). Lippincott Williams & Wilkins. Kindle Edition.

A nurse is assessing a client with a history of substance abuse who has pinpoint pupils, a heart rate of 56 beats/ minute, a respiratory rate of 6 breaths/ minute, and temperature of 96.4 ° F. The nurse determines that which is the most likely cause of the client's symptoms? 1. Opioids 2. Amphetamines 3. Cannabis 4. Alcohol

1. Opioids, such as morphine and heroin, can cause pinpoint pupils and a reduced heart rate, respiratory rate, and body temperature with intoxication. Amphetamine intoxication can lead to tachycardia, euphoria, and irritability. Cannabis (marijuana) intoxication can cause slowed reflexes, lethargy, and tachycardia. Alcohol intoxication leads to slurred speech, unsteady gait, and uncoordination. Lisko, Susan (2013-10-01). NCLEX-RN Questions and Answers Made Incredibly Easy (Nclexrn Questions & Answers Made Incredibly Easy) (Kindle Locations 16345-16348). Lippincott Williams & Wilkins. Kindle Edition.

46. A client has been admitted to the emergency department and states he just used cocaine. The nurse monitors the client for which condition? 1. Tachycardia 2. Hyperthermia 3. Hypotension 4. Bradypnea

1. Tachycardia is common because cocaine increases the heart's demand for oxygen. Cocaine doesn't cause hyperthermia (elevated temperature), hypotension (decreased blood pressure), or bradypnea (decreased respiratory rate). CN: Psychosocial integrity; Lisko, Susan (2013-10-01). NCLEX-RN Questions and Answers Made Incredibly Easy (Nclexrn Questions & Answers Made Incredibly Easy) (Kindle Locations 16298-16301). Lippincott Williams & Wilkins. Kindle Edition.

2. A nurse is assessing a client who has alcohol use disorder and is experiencing withdrawal. Which of the following is an expected finding? (Select all that apply.) A. Bradycardia B. Fine tremors of both hands C. Hypotension D. Vomiting E. Restlessness

2. A. INCORRECT: An expected finding of alcohol withdrawal is tachycardia rather than bradycardia. B. CORRECT: Fine tremors of both hands is an expected finding of alcohol withdrawal. C. INCORRECT: An expected finding of alcohol withdrawal is hypertension rather than hypotension. D. CORRECT: Vomiting is an expected finding of alcohol withdrawal. E. CORRECT: Restlessness is an expected finding of alcohol withdrawal.

A client addicted to alcohol is scheduled to begin individual therapy with the nurse. What is the most important nursing intervention for the client? 1. Learn to express feelings. 2. Establish new roles in the family. 3. Determine strategies for socializing. 4. Decrease preoccupation with physical health.

1. The client must address issues, learn ways to cope effectively with life stressors, and express his needs appropriately. After the client establishes sobriety, the possibility of taking on new roles can become a reality. Determining strategies for socializing isn't the priority intervention for an addicted client. Usually, these clients need to change former socializing habits. Clients addicted to alcohol don't tend to be preoccupied with physical health problems. Lisko, Susan (2013-10-01). NCLEX-RN Questions and Answers Made Incredibly Easy (Nclexrn Questions & Answers Made Incredibly Easy) (Kindle Locations 16181-16184). Lippincott Williams & Wilkins. Kindle Edition.

A client addicted to alcohol is scheduled to begin individual therapy with the nurse. What is the most important nursing intervention for the client? 1. Learn to express feelings. 2. Establish new roles in the family. 3. Determine strategies for socializing. 4. Decrease preoccupation with physical health. Lisko, Susan (2013-10-01). NCLEX-RN Questions and Answers Made Incredibly Easy (Nclexrn Questions & Answers Made Incredibly Easy) (Kindle Locations 16176-16180). Lippincott Williams & Wilkins. Kindle Edition.

1. The client must address issues, learn ways to cope effectively with life stressors, and express his needs appropriately. After the client establishes sobriety, the possibility of taking on new roles can become a reality. Determining strategies for socializing isn't the priority intervention for an addicted client. Usually, these clients need to change former socializing habits. Clients addicted to alcohol don't tend to be preoccupied with physical health problems. Lisko, Susan (2013-10-01). NCLEX-RN Questions and Answers Made Incredibly Easy (Nclexrn Questions & Answers Made Incredibly Easy) (Kindle Locations 16181-16184). Lippincott Williams & Wilkins. Kindle Edition.

What is the priority nursing intervention for a client recovering from cocaine addiction? 1. Help the client find ways to be happy and competent. 2. Foster the creative use of self in community activities. 3. Teach the client to handle stresses in the work setting. 4. Help the client acknowledge the current level of dependency.

1. The major component of a treatment program for a client with cocaine addiction is to have the client feel happy and competent. Cocaine addiction is difficult to treat because the drug actions reinforce its use. There are often perceived positive effects. Clients often credit the drug with giving them creative energy instead of looking within themselves. Fostering the creative use of self may inadvertently reinforce the client's drug use. Teaching the client to handle stresses is appropriate but isn't the most immediate nursing action. Examining the client's level of dependency isn't the immediate choice, as the client needs to work on remaining drug free. Lisko, Susan (2013-10-01). NCLEX-RN Questions and Answers Made Incredibly Easy (Nclexrn Questions & Answers Made Incredibly Easy) (Kindle Locations 16449-16454). Lippincott Williams & Wilkins. Kindle Edition.

A client who abused alcohol for more than 20 years is diagnosed with cirrhosis of the liver. The nurse determines that teaching about the disease has been successful when the client makes which statement? 1. " If I decide to stop drinking, I won't kill myself." 2. " If I watch my blood pressure, I should be okay." 3. " If I take vitamins, I can undo some liver damage." 4. " If I use nutritional supplements, I won't have problems."

1. This statement reflects the client's perception of the severity of the condition and the life-threatening complications that can result from continued use of alcohol. Aggressive treatment is required, not merely watching one's blood pressure. At this point in the illness, there is little likelihood that liver damage from cirrhosis can be altered. The fourth option denies the severity of the problem and negates the life-threatening complications common with a diagnosis of cirrhosis. Lisko, Susan (2013-10-01). NCLEX-RN Questions and Answers Made Incredibly Easy (Nclexrn Questions & Answers Made Incredibly Easy) (Kindle Locations 16225-16228). Lippincott Williams & Wilkins. Kindle Edition.

A client who abused alcohol for more than 20 years is diagnosed with cirrhosis of the liver. The nurse determines that teaching about the disease has been successful when the client makes which statement? 1. " If I decide to stop drinking, I won't kill myself." 2. " If I watch my blood pressure, I should be okay." 3. " If I take vitamins, I can undo some liver damage." 4. " If I use nutritional supplements, I won't have problems."

1. This statement reflects the client's perception of the severity of the condition and the life-threatening complications that can result from continued use of alcohol. Aggressive treatment is required, not merely watching one's blood pressure. At this point in the illness, there is little likelihood that liver damage from cirrhosis can be altered. The fourth option denies the severity of the problem and negates the life-threatening complications common with a diagnosis of cirrhosis. Lisko, Susan (2013-10-01). NCLEX-RN Questions and Answers Made Incredibly Easy (Nclexrn Questions & Answers Made Incredibly Easy) (Kindle Locations 16225-16228). Lippincott Williams & Wilkins. Kindle Edition.

16. A nurse is caring for a client undergoing treatment for acute alcohol dependence. The client tells the nurse, "I don't have a problem. My wife made me come here." Which defense mechanism does the nurse interpret the client's statement as representing? 1. Projection and suppression 2. Denial and rationalization 3. Rationalization and repression 4. Suppression and denial

2. The client is using denial and rationalization. Denial is the unconscious disclaimer of unacceptable thoughts, feelings, needs, or certain external factors. Rationalization is the unconscious effort to justify intolerable feelings, behaviors, and motives. The client isn't using projection, suppression, or repression. Lisko, Susan (2013-10-01). NCLEX-RN Questions and Answers Made Incredibly Easy (Nclexrn Questions & Answers Made Incredibly Easy) (Kindle Locations 15993-15995). Lippincott Williams & Wilkins. Kindle Edition.

A client tells the nurse that he used amphetamines to be productive at work. The nurse is aware that abrupt discontinuation of the drug will produce which symptom? 1. Severe anxiety 2. Increased yawning 3. Altered perceptions 4. Amotivational syndrome

1. When amphetamines are abruptly discontinued, the client may experience severe anxiety or agitation. Increased yawning is a symptom of opioid withdrawal. Altered perceptions occur when a client is withdrawing from hallucinogens. Amotivational syndrome is seen with clients using marijuana.

A client addicted to alcohol tells a nurse, "Making friends used to be hard for me." The nurse determines that client teaching about relationships has been successful when the client makes which statement? 1. " I've set limits on my behaviors toward others." 2. " I need to be judgmental of others." 3. " I won't become intimately involved with others." 4. " I can't bear to see myself hurt again in a relationship."

1. When the client can set personal limits and maintain boundaries, the ability to have successful interpersonal relationships can occur. Being judgmental is contraindicated if a client wants to have successful relationships. Setting arbitrary limits on relationships indicates the client needs to learn more interpersonal relationship skills. The universal truth about relationships is that they bring both joy and pain. The last statement indicates a need to learn more about relationships. Lisko, Susan (2013-10-01). NCLEX-RN Questions and Answers Made Incredibly Easy (Nclexrn Questions & Answers Made Incredibly Easy) (Kindle Locations 16213-16217). Lippincott Williams & Wilkins. Kindle Edition.

A client addicted to alcohol tells a nurse, "Making friends used to be hard for me." The nurse determines that client teaching about relationships has been successful when the client makes which statement? 1. " I've set limits on my behaviors toward others." 2. " I need to be judgmental of others." 3. " I won't become intimately involved with others." 4. " I can't bear to see myself hurt again in a relationship." Lisko, Susan (2013-10-01). NCLEX-RN Questions and Answers Made Incredibly Easy (Nclexrn Questions & Answers Made Incredibly Easy) (Kindle Locations 16208-16212). Lippincott Williams & Wilkins. Kindle Edition.

1. When the client can set personal limits and maintain boundaries, the ability to have successful interpersonal relationships can occur. Being judgmental is contraindicated if a client wants to have successful relationships. Setting arbitrary limits on relationships indicates the client needs to learn more interpersonal relationship skills. The universal truth about relationships is that they bring both joy and pain. The last statement indicates a need to learn more about relationships. Lisko, Susan (2013-10-01). NCLEX-RN Questions and Answers Made Incredibly Easy (Nclexrn Questions & Answers Made Incredibly Easy) (Kindle Locations 16213-16217). Lippincott Williams & Wilkins. Kindle Edition.

38. When teaching a client diagnosed with alcoholism about nutritional needs, which nutritional concept should the nurse emphasize? 1) Eat a high-protein, low-carbohydrate diet to promote lean body mass. 2) Increase sodium-rich foods to increase iodine levels. 3) Provide multivitamin supplements, including thiamine and folic acid. 4) Restrict fluid intake to decrease renal load.

3) Provide multivitamin supplements, including thiamine and folic acid. Vitamin B deficiencies contribute to the nervous system disorders seen in chronic alcohol abuse. Supplements of these vitamins are important to prevent complications. It is important that vitamin supplements include both thiamine (vitamin B1) and folic acid.

49. A client is diagnosed with stimulant use disorder: cocaine and antisocial personality disorder. The client eagerly participates in therapy and becomes charming and ingratiating to the primary nurse. Which best describes these client behaviors? 1) The client has not completed the cocaine withdrawal process. 2) The client is probably hiding something. 3) The client is exhibiting characteristics of antisocial personality disorder. 4) The client is exhibiting symptoms of cocaine dependence.

3) The client is exhibiting characteristics of antisocial personality disorder.

32. An impaired nurse is admitted to an inpatient substance abuse treatment facility. Which applies to his situation? 1) The nurse must relinquish his driver's license to the office of motor vehicles. 2) The nurse is mandated to comply with treatment and prescribed therapies. 3) The nurse is not mandated to meet specific requirements, because all civil rights are ensured. 4) The nurse must relinquish his registered nurse (RN) license to the state board of nursing.

3) The nurse is not mandated to meet specific requirements, because all civil rights are ensured. Although some variations occur from state to state, currently psychiatric clients maintain all of their civil rights. This nurse is not mandated by law to meet specific requirements, because all civil rights are ensured.

A nurse is caring for a client with congestive heart failure (CHF) who currently smokes cigarettes and has a 50 pack-year smoking history. When providing smoking cessation education to this client, which statements regarding the pathophysiology of nicotine use are appropriate? Select all that apply. A) "In low doses, nicotine stimulates nicotinic receptors in the brain to release dopamine." B) "In high doses, nicotine stimulates the parasympathetic system to release epinephrine, causing vasoconstriction." C) "Initially, nicotine increases mental alertness and cognitive ability." D) "Nicotine is a nonpsychoactive substance found in tobacco." E) "Gradual reduction of nicotine intake appears to be the best method of cessation."

A C

Hypertensive (High blood pressure)

A condition in which the force of the blood against the artery walls is too high.

'pseudo-addiction?'

A condition where a patient is experiencing severe pain, as a result of a chronic illness but the signs and symptoms of this are misunderstood. Carers and other healthcare professionals may interpret the patient's request for painkillers as a form of addiction.

Which of the following statements is true with regard to vaping? A) Vaping is a safe alternative to cigarette smoking. B) E-cigarettes are less popular among teenagers than cigarettes and other traditional tobacco products. C) Vaping has been linked to a devastating respiratory disease known as "popcorn lung." D) Throughout the United States, vaping is subject to the same regulations as cigarette smoking.

C

The nurse is planning care for a client with esophageal cancer caused by years of nicotine abuse. Which of the following would be the priority nursing diagnosis for this client? A) Decisional Conflict B) Social Isolation C) Disturbed Body Image D) Ineffective Airway Clearance

D

Which of these is NOT part of the DSM-5 criteria for diagnosing substance use disorder? Larger amounts of a substance are taken or over a longer period of time than intended. Substance use continues despite persistent or recurrent physical or psychological problems caused or exacerbated by the substance use. Important social, occupational, or recreational activities are given up or reduced because of substance use. A person has recurrent substance-related legal problems.

A person has recurrent substance-related legal problems.

A client asks the nurse about the effectiveness of hypnosis for smoking cessation. Which is the nurse's correct response? A) "Hypnosis works best when paired with a smoking cessation program." B) "Hypnosis is very effective." C) "Hypnosis never works for quitting." D) "Hypnosis sometimes works."

A) "Hypnosis works best when paired with a smoking cessation program." Rationale: Mixed results have been found in​ studies, perhaps due to the varying quality of the hypnotherapy.​ Therefore, it is recommended to pair hypnotherapy with other traditional smoking cessation programs.

26. A client diagnosed with alcohol use disorder joins a community 12-step program and states, My life is unmanageable. How should the nurse interpret this clients statement? A. The client is using minimization as an ego defense. B. The client is ready to sign an Alcoholics Anonymous contract for sobriety. C. The client has accomplished the first of 12 steps advocated by Alcoholics Anonymous. D. The client has met the requirements to be designated as an Alcoholics Anonymous sponsor

ANS: C The first step of the 12-step program advocated by Alcoholics Anonymous is that clients must admit powerlessness over alcohol and that their lives have become unmanageable.

4. A nurse is caring for a client who has alcohol use disorder. The client is no longer experiencing withdrawal manifestations. Which of the following medications should the nurse anticipate administering to assist the client with maintaining abstinence from alcohol? A. Chlordiazepoxide (Librium) B. Bupropion (Zyban) C. Disulfiram (Antuse) D. Carbamazepine (Tegretol)

4. A. INCORRECT: Chlordiazepoxide (Librium) is indicated for acute alcohol withdrawal rather than to maintain abstinence from alcohol. B. INCORRECT: Bupropion (Zyban) is indicated for nicotine withdrawal rather than to maintain abstinence from alcohol. C. CORRECT: Disulfiram (Antabuse) is administered to help clients maintain abstinence from alcohol. D. INCORRECT: Carbamazepine (Tegretol) is indicated for acute alcohol withdrawal rather than to maintain abstinence from alcohol.

A client with a history of alcohol abuse tells the nurse that he refuses to take his vitamins. What is the most appropriate response by the nurse? 1. " It's important to take vitamins to stop your craving." 2. " Prolonged use of alcohol can cause vitamin depletion." 3. " For every vitamin you take, you'll help your liver heal." 4. " By taking vitamins, you don't need to worry about your diet." Lisko, Susan (2013-10-01). NCLEX-RN Questions and Answers Made Incredibly Easy (Nclexrn Questions & Answers Made Incredibly Easy) (Kindle Locations 15957-15962). Lippincott Williams & Wilkins. Kindle Edition.

13. 2. Chronic alcoholism interferes with the metabolism of many vitamins. Vitamin supplements can prevent deficiencies from occurring. Taking vitamins won't stop a person from craving alcohol or help a damaged liver heal. A balanced diet is essential in addition to taking multivitamins. Lisko, Susan (2013-10-01). NCLEX-RN Questions and Answers Made Incredibly Easy (Nclexrn Questions & Answers Made Incredibly Easy) (Kindle Locations 15963-15965). Lippincott Williams & Wilkins. Kindle Edition.

14. The nurse determines further teaching about nutrition is necessary when an alcoholic client makes which statement? 1. " I should avoid foods high in fat." 2. " I should eat only one balanced meal per day." 3. " I should take vitamin and mineral supplements." 4. " I should eat large portions of food containing fiber."

14. 2. If the client eats only one adequate meal each day, there will be a deficit of essential nutrients. It's appropriate for the client to take vitamin and mineral supplements to prevent deficiency in these nutrients. Avoiding foods high in fat content and consuming large portions of foods containing fiber indicate the client has good knowledge about nutrition. Lisko, Susan (2013-10-01). NCLEX-RN Questions and Answers Made Incredibly Easy (Nclexrn Questions & Answers Made Incredibly Easy) (Kindle Locations 15972-15975). Lippincott Williams & Wilkins. Kindle Edition.

Which client statement demonstrates positive progress towards recovery from substance abuse? A) "I'm ready for discharge and feel better now." B) "I don't need to be here with these crazy people." C) ''I only used the pills to be able to sleep." "Taking those pills got out of control. It cost me my job, my marriage, and my children."

D

A client tells the nurse, "I have been drinking ever since they told me I had learning disabilities." How does the nurse interpret this response? 1. The client is self-medicating. 2. The client has an excuse to drink. 3. The client isn't a productive person. 4. The client will be unable to stop drinking.

15. 1. A client with learning disabilities may experience frustration, depression, or overall feelings of low self-esteem and may self-medicate with alcohol. Many people with learning disabilities don't resort to alcohol but develop other coping skills to handle the disability. People with learning disabilities can be very productive. A person with a learning disability can successfully recover from alcohol addiction. CN: Psychosocial integrity; CNS: None; CL: Lisko, Susan (2013-10-01). NCLEX-RN Questions and Answers Made Incredibly Easy (Nclexrn Questions & Answers Made Incredibly Easy) (Kindle Locations 15982-15986). Lippincott Williams & Wilkins. Kindle Edition.

35. Which would the nurse consider a priority intervention when planning care for a medically unstable client diagnosed with alcohol use disorder? 1) Simplifying the environment 2) Addressing physical needs 3) Providing opportunities for success experiences 4) Establishing a trusting interpersonal relationship

2) Addressing physical needs Physical problems must be addressed prior to meeting any psychosocial needs of a client who is medically unstable. According to Maslow's hierarchy of needs, physiological needs should be prioritized over all other needs.

48. A client who is unable to control binge drinking requires increased amounts of alcohol to achieve the same level of intoxication. The client is experiencing marital strife and legal problems. The client's behaviors meet the criteria for which DSM-5 diagnostic category? 1) Dual diagnosis 2) Alcohol use disorder 3) Neurocognitive disorder 4) Alcohol intoxication

2) Alcohol use disorder This client has developed tolerance, cannot control alcohol intake, and has continued use despite persistent problems related to drinking. These symptoms meet the criteria for the diagnosis of alcohol use disorder in the DSM-5.

33. Which client and family teaching is most important regarding the cause of substance addiction? 1) An individual's social and cultural environment can be implicated in the cause of substance addiction. 2) Biological, psychological, and sociocultural factors can all be implicated in the cause of substance addiction. 3) Evidence of a genetic link accounts for most cases of substance addiction. 4) Reinforcing properties of the substance encourage progression from use to addiction.

2) Biological, psychological, and sociocultural factors can all be implicated in the cause of substance addiction.

Which term would the nurse utilize to describe a syndrome that occurs after stopping the use of a drug to which one is addicted? A) Codependence B) Tolerance C) Guilt D) Withdrawal

D

A family tells the nurse that they are concerned about a family member who stopped using amphetamines 3 months ago and is now acting paranoid. What is the best response by the nurse? 1. " A person gets symptoms of paranoia with polysubstance abuse." 2. " When a person uses amphetamines, paranoid tendencies may continue for months." 3. " Sometimes, family dynamics and a high suspicion of continued drug use make a person paranoid." 4. " Amphetamine abusers may have severe anxiety and paranoid thinking." Lisko, Susan (2013-10-01). NCLEX-RN Questions and Answers Made Incredibly Easy (Nclexrn Questions & Answers Made Incredibly Easy) (Kindle Locations 16496-16501). Lippincott Williams & Wilkins. Kindle Edition.

2. After a client uses amphetamines, there may be long-term effects that exist for months after use. Two common effects are paranoia and ideas of reference. Even with polysubstance abuse, the paranoia comes from the chronic use of amphetamines. The third option blames the family when the paranoia comes from the drug use. Severe anxiety isn't typically manifested in paranoid thinking. Lisko, Susan (2013-10-01). NCLEX-RN Questions and Answers Made Incredibly Easy (Nclexrn Questions & Answers Made Incredibly Easy) (Kindle Locations 16502-16505). Lippincott Williams & Wilkins. Kindle Edition.

1. Family members of an alcoholic client ask the nurse to help them intervene. Which action is essential for a successful intervention? 1. All family members must tell the client they're powerless. 2. All family members must describe how the addiction affects them. 3. All family members must come up with their share of financial support. 4. All family members must become caregivers during the detoxification period.

2. After the family is taught about addiction, they must write down examples of how the addiction has affected each of them and use this information during the intervention. It isn't necessary to tell the client the family is powerless. The family is empowered through this intervention experience. In many cases, a third-party payer will help with treatment costs. Doing an intervention doesn't make family members responsible for financial support or providing care and support during the detoxification period. Lisko, Susan (2013-10-01). NCLEX-RN Questions and Answers Made Incredibly Easy (Nclexrn Questions & Answers Made Incredibly Easy) (Kindle Locations 15840-15844). Lippincott Williams & Wilkins. Kindle Edition.

The nurse is caring for a client struggling with alcohol dependence. It is most important for the nurse to do which of the following? 1. Speak briefly and directly. 2. Avoid blaming or preaching to the client. 3. Confront feelings and examples of perfectionism. 4. Determine if nonverbal communication will be more effective.

2. Blaming or preaching to the client causes negativity and prevents the client from hearing what the nurse has to say. Speaking briefly to the client may not allow time for adequate communication. Perfectionism doesn't tend to be an issue. Determining if nonverbal communication will be more effective is better suited for a client with cognitive impairment. Lisko, Susan (2013-10-01). NCLEX-RN Questions and Answers Made Incredibly Easy (Nclexrn Questions & Answers Made Incredibly Easy) (Kindle Locations 16075-16077). Lippincott Williams & Wilkins. Kindle Edition.

The nurse is caring for a client struggling with alcohol dependence. It is most important for the nurse to do which of the following? 1. Speak briefly and directly. 2. Avoid blaming or preaching to the client. 3. Confront feelings and examples of perfectionism. 4. Determine if nonverbal communication will be more effective. Lisko, Susan (2013-10-01). NCLEX-RN Questions and Answers Made Incredibly Easy (Nclexrn Questions & Answers Made Incredibly Easy) (Kindle Locations 16070-16074). Lippincott Williams & Wilkins. Kindle Edition.

2. Blaming or preaching to the client causes negativity and prevents the client from hearing what the nurse has to say. Speaking briefly to the client may not allow time for adequate communication. Perfectionism doesn't tend to be an issue. Determining if nonverbal communication will be more effective is better suited for a client with cognitive impairment. Lisko, Susan (2013-10-01). NCLEX-RN Questions and Answers Made Incredibly Easy (Nclexrn Questions & Answers Made Incredibly Easy) (Kindle Locations 16075-16077). Lippincott Williams & Wilkins. Kindle Edition.

The nurse is assessing a client who repeatedly abuses cocaine. It is important for the nurse to observe the client for which of the following? 1. Panic attacks 2. Bipolar cycling 3. Attention deficits 4. Expressive aphasia

2. Clients who frequently use cocaine will experience the rapid cycling effect of excitement and then severe depression. They don't tend to experience panic attacks, expressive aphasia, or attention deficits. Lisko, Susan (2013-10-01). NCLEX-RN Questions and Answers Made Incredibly Easy (Nclexrn Questions & Answers Made Incredibly Easy) (Kindle Locations 16364-16366). Lippincott Williams & Wilkins. Kindle Edition.

A client diagnosed with alcohol abuse joins a community 12-step program and states, "My life is unmanageable." How should the nurse interpret this client's statement? A. The client is using minimization as an ego defense. B. The client is ready to sign an Alcoholics Anonymous contract for sobriety. C. The client has accomplished the first of 12 steps advocated by Alcoholics Anonymous. D. The client has met the requirements to be designated as an Alcoholics Anonymous sponsor.

ANS: C The first step of the 12-step program advocated by Alcoholics Anonymous is that clients must admit powerlessness over alcohol and that their lives have become unmanageable.

A client requiring a mandatory​ court-ordered drug testing is seen in the clinic. On which body tissue should the nurse conduct this​ testing? (Select all that​ apply.) Hair DNA Urine Serum Saliva

Hair Urine Serum Saliva

A client who formerly used lysergic acid diethylamide (LSD) is seeking counseling. The nurse anticipates that the assessment of the client will include which finding? 1. Lack of trust 2. Panic attacks 3. Recurrent depression 4. Loss of ego boundaries

2. Clients who used LSD typically have a history of panic attacks or psychotic behavior. This is often referred to as a "bad trip." Loss of ego boundaries, recurrent depression, and lack of trust don't tend to be problems for this type of client. CN: Psychosocial integrity; CNS: None; CL: Lisko, Susan (2013-10-01). NCLEX-RN Questions and Answers Made Incredibly Easy (Nclexrn Questions & Answers Made Incredibly Easy) (Kindle Locations 16402-16405). Lippincott Williams & Wilkins. Kindle Edition.

A nurse holds the hand of a client who is withdrawing from alcohol. What is the nurse's rationale for this intervention? A. To assess for emotional strength B. To assess for Wernicke-Korsakoff syndrome C. To assess for tachycardia D. To assess for fine tremors

ANS: D The nurse is most likely assessing the client for fine tremors secondary to alcohol withdrawal. Withdrawal from alcohol can also cause headache, insomnia, transient hallucinations, depression, irritability, anxiety, elevated blood pressure, sweating, tachycardia, malaise, coarse tremors, and seizure activity.

A client with a history of alcohol abuse tells the nurse that he refuses to take his vitamins. What is the most appropriate response by the nurse? 1. " It's important to take vitamins to stop your craving." 2. " Prolonged use of alcohol can cause vitamin depletion." 3. " For every vitamin you take, you'll help your liver heal." 4. " By taking vitamins, you don't need to worry about your diet."

2. Chronic alcoholism interferes with the metabolism of many vitamins. Vitamin supplements can prevent deficiencies from occurring. Taking vitamins won't stop a person from craving alcohol or help a damaged liver heal. A balanced diet is essential in addition to taking multivitamins. Lisko, Susan (2013-10-01). NCLEX-RN Questions and Answers Made Incredibly Easy (Nclexrn Questions & Answers Made Incredibly Easy) (Kindle Locations 15963-15965). Lippincott Williams & Wilkins. Kindle Edition.

A nurse is caring for a client recovering from cocaine abuse. The priority intervention for this client would be? 1. Skin care 2. Suicide precautions 3. Frequent orientation 4. Nutrition consultation

2. Clients recovering from cocaine use are prone to "postcoke depression" and have a likelihood of becoming suicidal if they can't take the drug. Frequent orientation and skin care are routine nursing interventions but aren't the most immediate considerations for this client. Nutrition consultation isn't the most pressing intervention for this client. CN: Safe, effective care environment; CNS: Management Lisko, Susan (2013-10-01). NCLEX-RN Questions and Answers Made Incredibly Easy (Nclexrn Questions & Answers Made Incredibly Easy) (Kindle Locations 16355-16359). Lippincott Williams & Wilkins. Kindle Edition.

A client with a history of alcohol abuse has been diagnosed with nutritional deficits. What is the best intervention for the nurse to implement? 1. Encourage the client to eat a diet high in calories. 2. Help the client recognize and follow a balanced diet. 3. Have the client drink liquid protein supplements daily. 4. Have the client monitor the calories consumed each day.

2. Clients who abuse alcohol are usually malnourished and need help to follow a balanced diet. Increasing calories may cause the client to just eat empty calories. The client must be involved in the decision to supplement the daily dietary intake. The nurse can't force the client to drink liquid protein supplements. Having the client monitor calorie intake could be done only after the client recognizes the need to maintain a balanced diet. Calorie counts usually aren't needed in most recovering clients who begin to eat from the basic food groups. Lisko, Susan (2013-10-01). NCLEX-RN Questions and Answers Made Incredibly Easy (Nclexrn Questions & Answers Made Incredibly Easy) (Kindle Locations 15951-15955). Lippincott Williams & Wilkins. Kindle Edition.

A client with a history of alcohol abuse has been diagnosed with nutritional deficits. What is the best intervention for the nurse to implement? 1. Encourage the client to eat a diet high in calories. 2. Help the client recognize and follow a balanced diet. 3. Have the client drink liquid protein supplements daily. 4. Have the client monitor the calories consumed each day.

2. Clients who abuse alcohol are usually malnourished and need help to follow a balanced diet. Increasing calories may cause the client to just eat empty calories. The client must be involved in the decision to supplement the daily dietary intake. The nurse can't force the client to drink liquid protein supplements. Having the client monitor calorie intake could be done only after the client recognizes the need to maintain a balanced diet. Calorie counts usually aren't needed in most recovering clients who begin to eat from the basic food groups. Lisko, Susan (2013-10-01). NCLEX-RN Questions and Answers Made Incredibly Easy (Nclexrn Questions & Answers Made Incredibly Easy) (Kindle Locations 15951-15955). Lippincott Williams & Wilkins. Kindle Edition.

18. What is the most important short-term goal for a client with a knowledge deficit about the effects of alcohol on the body? 1. Test blood chemistries daily. 2. Verbalize the results of substance use. 3. Talk to a pharmacist about the substance. 4. Attend a weekly aerobic exercise program.

2. It's important for the client to talk about the health consequences of the continued use of alcohol. Testing blood chemistries daily gives the client minimal knowledge about the effects of alcohol on the body and isn't the most useful information in a teaching plan. A pharmacist isn't the appropriate health care professional to educate the client about the effects of alcohol use on the body. Although exercise is an important goal of self-care, it doesn't address the client's knowledge deficit about the effects of alcohol on the body. CN: Safe, effective care environment; Lisko, Susan (2013-10-01). NCLEX-RN Questions and Answers Made Incredibly Easy (Nclexrn Questions & Answers Made Incredibly Easy) (Kindle Locations 16012-16017). Lippincott Williams & Wilkins. Kindle Edition.

Which of the following is a common element of abuse experienced by the victim? A) Accidental injury B) Feelings of control C) Humiliation D) Manipulation

Answer: C Common elements of abuse include humiliation, intimidation, and physical injury. Injury associated with abuse is not accidental. Feelings of control and use of manipulation tactics are related to the perpetrator, not the victim.

The nurse completed a health history on a client addicted to nicotine. Which information is needed when collecting information about past nicotine​ use? (Select all that​ apply.)

​Rationale: When obtaining a health​ history, it is important to collect information on past nicotine​ use, such as​ amount, frequency, and duration. The brand and the cost of nicotine used are not important factors when obtaining the health history.

The nurse anticipates that a client undergoing nicotine withdrawal may make which statement? 1. " I sometimes feel like I'm seeing things." 2. " I feel lousy, and I'm grumpy with everybody." 3. " I can't believe I feel fine after just having stopped smoking." 4. " I'm always yawning now." .

2. During nicotine withdrawal, the client is typically irritable and nervous. Seeing things (hallucinations) isn't linked to nicotine withdrawal. A client going through nicotine withdrawal is unlikely to "feel fine." Yawning is associated with withdrawal from opioids, not nicotine. Lisko, Susan (2013-10-01). NCLEX-RN Questions and Answers Made Incredibly Easy (Nclexrn Questions & Answers Made Incredibly Easy) (Kindle Locations 16594-16596). Lippincott Williams & Wilkins. Kindle Edition.

The nurse is caring for a client with a history of chronic alcoholism and is aware that the client may be predisposed to which of the following? 1. Arteriosclerosis 2. Heart failure 3. Heart valve damage 4. Pericarditis

2. Heart failure is a severe cardiac consequence associated with long-term alcohol use. Arteriosclerosis, heart valve damage, and pericarditis aren't medical consequences of alcoholism. Lisko, Susan (2013-10-01). NCLEX-RN Questions and Answers Made Incredibly Easy (Nclexrn Questions & Answers Made Incredibly Easy) (Kindle Locations 15933-15934). Lippincott Williams & Wilkins. Kindle Edition.

The nurse is caring for a client with a history of chronic alcoholism and is aware that the client may be predisposed to which of the following? 1. Arteriosclerosis 2. Heart failure 3. Heart valve damage 4. Pericarditis

2. Heart failure is a severe cardiac consequence associated with long-term alcohol use. Arteriosclerosis, heart valve damage, and pericarditis aren't medical consequences of alcoholism. Lisko, Susan (2013-10-01). NCLEX-RN Questions and Answers Made Incredibly Easy (Nclexrn Questions & Answers Made Incredibly Easy) (Kindle Locations 15933-15934). Lippincott Williams & Wilkins. Kindle Edition.

14. The nurse determines further teaching about nutrition is necessary when an alcoholic client makes which statement? 1. " I should avoid foods high in fat." 2. " I should eat only one balanced meal per day." 3. " I should take vitamin and mineral supplements." 4. " I should eat large portions of food containing fiber."

2. If the client eats only one adequate meal each day, there will be a deficit of essential nutrients. It's appropriate for the client to take vitamin and mineral supplements to prevent deficiency in these nutrients. Avoiding foods high in fat content and consuming large portions of foods containing fiber indicate the client has good knowledge about nutrition. Lisko, Susan (2013-10-01). NCLEX-RN Questions and Answers Made Incredibly Easy (Nclexrn Questions & Answers Made Incredibly Easy) (Kindle Locations 15972-15975). Lippincott Williams & Wilkins. Kindle Edition.

Which therapy should the nurse recommend that demonstrates success in smoking​ cessation? (Select all that​ apply.)

​Rationale: Yoga and massage have been shown to reduce anxiety when withdrawing from nicotine. Acupuncture has been shown to improve the likelihood of quitting.​ Gum, with or without​ nicotine, can be used to give oral stimulation without cigarettes. No studies have been done to show phototherapy improves smoking cessation.

18. What is the most important short-term goal for a client with a knowledge deficit about the effects of alcohol on the body? 1. Test blood chemistries daily. 2. Verbalize the results of substance use. 3. Talk to a pharmacist about the substance. 4. Attend a weekly aerobic exercise program.

2. It's important for the client to talk about the health consequences of the continued use of alcohol. Testing blood chemistries daily gives the client minimal knowledge about the effects of alcohol on the body and isn't the most useful information in a teaching plan. A pharmacist isn't the appropriate health care professional to educate the client about the effects of alcohol use on the body. Although exercise is an important goal of self-care, it doesn't address the client's knowledge deficit about the effects of alcohol on the body. CN: Safe, effective care environment; Lisko, Susan (2013-10-01). NCLEX-RN Questions and Answers Made Incredibly Easy (Nclexrn Questions & Answers Made Incredibly Easy) (Kindle Locations 16012-16017). Lippincott Williams & Wilkins. Kindle Edition.

A nurse is caring for a client addicted to heroin who is experiencing withdrawal symptoms. The nurse is aware that the withdrawal symptoms may be affected by which factor? 1. Ego strength 2. Liver function 3. Seizure history 4. Kidney function

2. Liver function status is an important variable that can be used to indicate the severity of a client's drug withdrawal. Ego strength, seizure history, and kidney function aren't variables that can be used to predict the severity of withdrawal symptoms.

A client who is withdrawing from alcohol is being given lorazepam (Ativan). The client's family asks the nurse about the medication. What is the best response by the nurse? 1. " Short-term use of lorazepam can lead to dependence." 2. " The lorazepam will reduce the symptoms of withdrawal." 3. " The lorazepam will make him forget about symptoms of withdrawal." 4. " The lorazepam will also help with his heart disease."

2. Lorazepam is a short-acting benzodiazepine usually given for 1 week to help the client in alcohol withdrawal. Long-term (not short-term) use of lorazepam can lead to dependence. The medication isn't given to help forget the experience; it lessens the symptoms of withdrawal. It isn't used to treat coexisting cardiovascular problems.

8. A client who is withdrawing from alcohol is being given lorazepam (Ativan). The client's family asks the nurse about the medication. What is the best response by the nurse? 1. " Short-term use of lorazepam can lead to dependence." 2. " The lorazepam will reduce the symptoms of withdrawal." 3. " The lorazepam will make him forget about symptoms of withdrawal." 4. " The lorazepam will also help with his heart disease."

2. Lorazepam is a short-acting benzodiazepine usually given for 1 week to help the client in alcohol withdrawal. Long-term (not short-term) use of lorazepam can lead to dependence. The medication isn't given to help forget the experience; it lessens the symptoms of withdrawal. It isn't used to treat coexisting cardiovascular problems. Lisko, Susan (2013-10-01). NCLEX-RN Questions and Answers Made Incredibly Easy (Nclexrn Questions & Answers Made Incredibly Easy) (Kindle Locations 15912-15914). Lippincott Williams & Wilkins. Kindle Edition.

8. A client who is withdrawing from alcohol is being given lorazepam (Ativan). The client's family asks the nurse about the medication. What is the best response by the nurse? 1. " Short-term use of lorazepam can lead to dependence." 2. " The lorazepam will reduce the symptoms of withdrawal." 3. " The lorazepam will make him forget about symptoms of withdrawal." 4. " The lorazepam will also help with his heart disease."

2. Lorazepam is a short-acting benzodiazepine usually given for 1 week to help the client in alcohol withdrawal. Long-term (not short-term) use of lorazepam can lead to dependence. The medication isn't given to help forget the experience; it lessens the symptoms of withdrawal. It isn't used to treat coexisting cardiovascular problems. Lisko, Susan (2013-10-01). NCLEX-RN Questions and Answers Made Incredibly Easy (Nclexrn Questions & Answers Made Incredibly Easy) (Kindle Locations 15912-15914). Lippincott Williams & Wilkins. Kindle Edition.

A client tells a nurse, "I'm not going to have problems from smoking marijuana." What is the most appropriate response by the nurse? 1. " Evidence shows it can cause major health problems." 2. " Marijuana can cause reproductive problems later in life." 3. " Smoking marijuana isn't as dangerous as smoking cigarettes." 4. " Some people have minor or no reactions to smoking marijuana." .

2. Marijuana causes cardiac, respiratory, immune, and reproductive health problems. Most people who smoke marijuana don't have major health problems. All people who smoke marijuana have symptoms of intoxication. The residues from marijuana are more toxic than those from cigarettes. Lisko, Susan (2013-10-01). NCLEX-RN Questions and Answers Made Incredibly Easy (Nclexrn Questions & Answers Made Incredibly Easy) (Kindle Locations 16236-16238). Lippincott Williams & Wilkins. Kindle Edition.

What is the most important teaching information for the nurse to provide a client who abuses prescription drugs? 1. Herbal substitutes are safer to use. 2. Medication should be used only for the reason prescribed. 3. The client should consult a physician before using a drug. 4. Consider if family members influence the client to use drugs.

2. People often take prescribed drugs for reasons other than those intended, primarily to self-medicate or experience a sense of euphoria. The safety and efficacy of most herbal remedies haven't been established. Sometimes, over-the-counter medications are necessary for minor problems. There may be a family history of substance abuse, but it isn't a priority when planning nursing care. Lisko, Susan (2013-10-01). NCLEX-RN Questions and Answers Made Incredibly Easy (Nclexrn Questions & Answers Made Incredibly Easy) (Kindle Locations 16307-16310). Lippincott Williams & Wilkins. Kindle Edition.

The nurse is incorporating yoga into a smoking cessation class. Which rationale explains the value of yoga for this​ class?

​Rationale: Yoga reduces anxiety caused by the craving for nicotine during cessation. While it may build flexibility and raise spiritual​ awareness, that is not the purpose in this class. Yoga does not influence nicotinic receptors in the brain.

16. A nurse is caring for a client undergoing treatment for acute alcohol dependence. The client tells the nurse, "I don't have a problem. My wife made me come here." Which defense mechanism does the nurse interpret the client's statement as representing? 1. Projection and suppression 2. Denial and rationalization 3. Rationalization and repression 4. Suppression and denial Lisko, Susan (2013-10-01). NCLEX-RN Questions and Answers Made Incredibly Easy (Nclexrn Questions & Answers Made Incredibly Easy) (Kindle Locations 15987-15992). Lippincott Williams & Wilkins. Kindle Edition.

2. The client is using denial and rationalization. Denial is the unconscious disclaimer of unacceptable thoughts, feelings, needs, or certain external factors. Rationalization is the unconscious effort to justify intolerable feelings, behaviors, and motives. The client isn't using projection, suppression, or repression. Lisko, Susan (2013-10-01). NCLEX-RN Questions and Answers Made Incredibly Easy (Nclexrn Questions & Answers Made Incredibly Easy) (Kindle Locations 15993-15995). Lippincott Williams & Wilkins. Kindle Edition.

A client withdrawing from alcohol tells the nurse that he is worried about periodic hallucinations. What is the most appropriate intervention by the nurse? 1. Point out that the sensation doesn't exist. 2. Allow the client to talk about the experience. 3. Encourage the client to wash the body areas well. 4. Determine if the client has a cognitive impairment.

2. The client needs to talk about the periodic hallucinations to prevent them from becoming triggers to acting out behaviors and possible self-injury. The client's experience of sensory-perceptual alterations must be acknowledged; therefore, denying that the client's hallucinations exist isn't a helpful strategy. Determining if the client has a cognitive impairment and encouraging the client to wash the body areas well don't address the problem of periodic hallucinations. Lisko, Susan (2013-10-01). NCLEX-RN Questions and Answers Made Incredibly Easy (Nclexrn Questions & Answers Made Incredibly Easy) (Kindle Locations 16033-16036). Lippincott Williams & Wilkins. Kindle Edition.

A client withdrawing from alcohol tells the nurse that he is worried about periodic hallucinations. What is the most appropriate intervention by the nurse? 1. Point out that the sensation doesn't exist. 2. Allow the client to talk about the experience. 3. Encourage the client to wash the body areas well. 4. Determine if the client has a cognitive impairment.

2. The client needs to talk about the periodic hallucinations to prevent them from becoming triggers to acting out behaviors and possible self-injury. The client's experience of sensory-perceptual alterations must be acknowledged; therefore, denying that the client's hallucinations exist isn't a helpful strategy. Determining if the client has a cognitive impairment and encouraging the client to wash the body areas well don't address the problem of periodic hallucinations. Lisko, Susan (2013-10-01). NCLEX-RN Questions and Answers Made Incredibly Easy (Nclexrn Questions & Answers Made Incredibly Easy) (Kindle Locations 16033-16036). Lippincott Williams & Wilkins. Kindle Edition.

A client experiencing alcohol withdrawal tells the nurse she is upset about going through detoxification. What is the most important goal for this client? 1. The client will commit to a drug-free lifestyle. 2. The client will work with the nurse to remain safe. 3. The client will drink plenty of fluids on a daily basis. 4. The client will make a personal inventory of strengths.

2. The priority goal is for client safety. Although drinking enough fluids, identifying personal strengths, and committing to a drug-free lifestyle are important goals, the nurse's first priority must be to promote client safety. Lisko, Susan (2013-10-01). NCLEX-RN Questions and Answers Made Incredibly Easy (Nclexrn Questions & Answers Made Incredibly Easy) (Kindle Locations 16056-16058). Lippincott Williams & Wilkins. Kindle Edition.

A client experiencing alcohol withdrawal tells the nurse she is upset about going through detoxification. What is the most important goal for this client? 1. The client will commit to a drug-free lifestyle. 2. The client will work with the nurse to remain safe. 3. The client will drink plenty of fluids on a daily basis. 4. The client will make a personal inventory of strengths. Lisko, Susan (2013-10-01). NCLEX-RN Questions and Answers Made Incredibly Easy (Nclexrn Questions & Answers Made Incredibly Easy) (Kindle Locations 16050-16055). Lippincott Williams & Wilkins. Kindle Edition.

2. The priority goal is for client safety. Although drinking enough fluids, identifying personal strengths, and committing to a drug-free lifestyle are important goals, the nurse's first priority must be to promote client safety. Lisko, Susan (2013-10-01). NCLEX-RN Questions and Answers Made Incredibly Easy (Nclexrn Questions & Answers Made Incredibly Easy) (Kindle Locations 16056-16058). Lippincott Williams & Wilkins. Kindle Edition.

The nurse is performing an assessment of a client with a history of polysubstance abuse. What is the most important information for the nurse to obtain? 1. Oral administration of any drug 2. Time of last use of each drug 3. How the drug was obtained 4. The place the drug was used

2. The time of last use gives information about expected withdrawal symptoms of the drugs and what immediate treatment is necessary. How the drugs were obtained and the places the drugs were used aren't essential information for treatment, nor is administration. Lisko, Susan (2013-10-01). NCLEX-RN Questions and Answers Made Incredibly Easy (Nclexrn Questions & Answers Made Incredibly Easy) (Kindle Locations 16246-16247). Lippincott Williams & Wilkins. Kindle Edition.

31. A client recovering from alcohol abuse tells the nurse, "I get nothing out of Alcoholics Anonymous (AA) meetings." What is the best response by the nurse? 1. " What were you told about going to AA meetings?" 2. " What do you want to get out of the AA meetings?" 3. " When do you think you'll stop going to the meetings?" 4. " Do you think you can control what happens in a meeting?"

2. This response puts some of the responsibility for staying sober on the client and encourages the client to take a more active role. Asking what the client was told about AA meetings opens up a discussion that allows the client to continue to discuss disappointments rather than taking a proactive stand to support the value of AA meetings. The third option condones the client's desire to stop going to the meetings. The fourth option changes the issue from being responsible for staying sober to focusing on what the client can't control. Lisko, Susan (2013-10-01). NCLEX-RN Questions and Answers Made Incredibly Easy (Nclexrn Questions & Answers Made Incredibly Easy) (Kindle Locations 16149-16152). Lippincott Williams & Wilkins. Kindle Edition.

31. A client recovering from alcohol abuse tells the nurse, "I get nothing out of Alcoholics Anonymous (AA) meetings." What is the best response by the nurse? 1. " What were you told about going to AA meetings?" 2. " What do you want to get out of the AA meetings?" 3. " When do you think you'll stop going to the meetings?" 4. " Do you think you can control what happens in a meeting?"

2. This response puts some of the responsibility for staying sober on the client and encourages the client to take a more active role. Asking what the client was told about AA meetings opens up a discussion that allows the client to continue to discuss disappointments rather than taking a proactive stand to support the value of AA meetings. The third option condones the client's desire to stop going to the meetings. The fourth option changes the issue from being responsible for staying sober to focusing on what the client can't control. Lisko, Susan (2013-10-01). NCLEX-RN Questions and Answers Made Incredibly Easy (Nclexrn Questions & Answers Made Incredibly Easy) (Kindle Locations 16149-16152). Lippincott Williams & Wilkins. Kindle Edition.

56. The nurse determines that teaching about cocaine has been effective when the client makes which statement? 1. " I wasn't using cocaine to feel better about myself." 2. " I started using cocaine more and more until I couldn't stop." 3. " I'm not addicted to cocaine because I don't use it every day." 4. " I'm not going to be a chronic user; I only use it on holidays." Lisko, Susan (2013-10-01). NCLEX-RN Questions and Answers Made Incredibly Easy (Nclexrn Questions & Answers Made Incredibly Easy) (Kindle Locations 16386-16391). Lippincott Williams & Wilkins. Kindle Edition.

2. This statement reflects the trajectory or common pattern of cocaine use and indicates successful teaching. The first option reflects the client's denial. People gravitate to the drug and continue its use because it gives them a sense of well-being, competency, and power. Cocaine abusers tend to be binge users and can be drug free for days or weeks between use, but they still have a drug problem. The fourth option indicates the client is in denial about the drug's potential to become a habit. Effective teaching didn't occur. Lisko, Susan (2013-10-01). NCLEX-RN Questions and Answers Made Incredibly Easy (Nclexrn Questions & Answers Made Incredibly Easy) (Kindle Locations 16392-16395). Lippincott Williams & Wilkins. Kindle Edition.

An alcoholic client tells the nurse, "I feel so depressed about what I've done to my family that I feel like giving up." It is most important for the nurse to assess the client for which of the following? 1. Family support 2. A plan for self-harm 3. A sponsor for the client 4. Other ambivalent feelings

2. When a client talks about giving up, the nurse must explore the potential for suicidal behavior. Although questioning the client about family support, the availability of a sponsor, or ambivalent feelings is important, the priority action is to assess for suicide. Lisko, Susan (2013-10-01). NCLEX-RN Questions and Answers Made Incredibly Easy (Nclexrn Questions & Answers Made Incredibly Easy) (Kindle Locations 16024-16026). Lippincott Williams & Wilkins. Kindle Edition.

An alcoholic client tells the nurse, "I feel so depressed about what I've done to my family that I feel like giving up." It is most important for the nurse to assess the client for which of the following? 1. Family support 2. A plan for self-harm 3. A sponsor for the client 4. Other ambivalent feelings

2. When a client talks about giving up, the nurse must explore the potential for suicidal behavior. Although questioning the client about family support, the availability of a sponsor, or ambivalent feelings is important, the priority action is to assess for suicide. Lisko, Susan (2013-10-01). NCLEX-RN Questions and Answers Made Incredibly Easy (Nclexrn Questions & Answers Made Incredibly Easy) (Kindle Locations 16024-16026). Lippincott Williams & Wilkins. Kindle Edition.

The nurse is providing education to a client who wants to quit smoking. Which statements are appropriate for the nurse to include in the teaching session? Select all that apply. A) "There is no adverse risk if you choose to smoke while wearing a nicotine patch." B) "Bupropion (Zyban) is used to suppress the craving for tobacco." C) "A piece of nicotine gum should be chewed for 5 minutes of every waking hour, then held in the cheek." D) "Most people quit smoking several times before they are successful." E) "Alternative therapies can help reduce the stress that accompanies smoking cessation."

B D E

51. A client with chronic alcoholism says to the nurse, "I'm tired of using and I want to stop. Is there a medication that can help me maintain sobriety?" About which medication would the nurse provide information? 1) Carbamazepine (Tegretol) 2) Clonidine (Catapres) 3) Disulfiram (Antabuse) 4) Folic acid (Folvite)

3) Disulfiram (Antabuse) Ingestion of alcohol while disulfiram is in the body results in a syndrome of symptoms that can cause varying degrees of discomfort. It can even result in death if blood alcohol levels are high enough. It is important that the client understands that all alcohol, oral or topical, and medications that contain alcohol are strictly prohibited when taking this drug.

50. A client diagnosed with chronic alcohol use disorder complains of feeling tremulous. The client's BP is now 170/110, P 116, R 30, T 97°F. The nurse anticipates which medication would give the client the most immediate relief from these symptoms? 1) Benztropine (Cogentin), 2 mg PO 2) Oxazepam (Serax), 30 mg PO 3) Lorazepam (Ativan), 1 mg IM 4) Meperidine (Demerol), 100 mg IM

3) Lorazepam (Ativan), 1 mg IM Ativan is frequently used to treat the symptoms of alcohol withdrawal. Because Ativan is ordered parenterally, this medication would give the client the most immediate relief of symptoms.

36. A client who is going through alcohol detoxification states, "I see bugs crawling on the wall." Which is the best nursing response? 1) "I'll remove the bugs from the wall." 2) "You are confused because of your alcoholism." 3) "There are no bugs on the wall. I'll stay with you until you feel less anxious." 4) "You do not see any bugs on the wall."

3) "There are no bugs on the wall. I'll stay with you until you feel less anxious." This response presents objective reality and may help decrease the client's anxiety by the nurse's therapeutic offering of self.

Which therapy should the nurse recommend that demonstrates success in smoking cessation? SATA A) Phototherapy B) Acupuncture C) Nicotine gum D) Massage E) Yoga

B, C, D, E Rationale: Yoga and massage have been shown to reduce anxiety when withdrawing from nicotine. Acupuncture has been shown to improve the likelihood of quitting.​ Gum, with or without​ nicotine, can be used to give oral stimulation without cigarettes. No studies have been done to show phototherapy improves smoking cessation.

A client who abuses alcohol is admitted to an outpatient drug and alcohol treatment facility. The nurse determines that which of the following is the most objective way to determine if the client is still using alcohol? 1. Having the client walk a straight line 2. Smelling the client's breath 3. Giving the client a breath alcohol test 4. Asking the client if he has been drinking

3. A breath alcohol test is the most objective way to determine if the client is still using alcohol. Having him walk a straight line and smelling his breath aren't objective tests. Asking him if he has been drinking may not elicit an honest answer (many clients who abuse alcohol deny alcohol use). Lisko, Susan (2013-10-01). NCLEX-RN Questions and Answers Made Incredibly Easy (Nclexrn Questions & Answers Made Incredibly Easy) (Kindle Locations 16584-16586). Lippincott Williams & Wilkins. Kindle Edition.

A client who abuses alcohol is admitted to an outpatient drug and alcohol treatment facility. The nurse determines that which of the following is the most objective way to determine if the client is still using alcohol? 1. Having the client walk a straight line 2. Smelling the client's breath 3. Giving the client a breath alcohol test 4. Asking the client if he has been drinking

3. A breath alcohol test is the most objective way to determine if the client is still using alcohol. Having him walk a straight line and smelling his breath aren't objective tests. Asking him if he has been drinking may not elicit an honest answer (many clients who abuse alcohol deny alcohol use). Lisko, Susan (2013-10-01). NCLEX-RN Questions and Answers Made Incredibly Easy (Nclexrn Questions & Answers Made Incredibly Easy) (Kindle Locations 16584-16586). Lippincott Williams & Wilkins. Kindle Edition.

3. A nurse is planning care for a client who is experiencing benzodiazepine withdrawal. Which of the following is the priority nursing intervention? A. Orient the client frequently to time, place, and person. B. Offer fluids and nourishing diet as tolerated. C. Implement seizure precautions. D. Encourage participation in group therapy sessions.

3. A. INCORRECT: Reorienting the client is an appropriate intervention. However, it is not the priority. B. INCORRECT: Providing hydration and nourishment is an appropriate intervention. However, it is not the priority. C. CORRECT: The greatest risk to the client is injury. Therefore, implementing seizure precautions is the priority intervention. D. INCORRECT: Encouraging participation in therapy is an appropriate intervention. However, it is not the priority.

The nurse prepares a presentation to promote smoking cessation at an assisted living facility. Which condition should the nurse present as having an association with smoking? SATA A) Cataracts B) Retinal detachment C) Bone disease D) Stomach ulcers E) Diabetes complications

A, C, D, E Rationale: Smoking increases the risk of many types of​ cancers, especially cancers of the respiratory and gastrointestinal​ (GI) tracts. Smoking has also been linked to diseases other than cancer in older​ adults, including pulmonary and cardiovascular​ diseases, diabetes​ complications, bone​ disease, bone density​ loss, cataracts, and stomach ulcers. Smokers are up to 10 times more likely to get cancer than a person who has never smoked.​ However, it has not been linked to retinal detachment.

The nurse at a clinic prepares a class on the effects of secondhand smoke on children. Which condition shoult the nurse include? SATA A) Cancer B) Yellowing of the fingers C) Wrinkles in the skin D) Upper respiratory tract infections E) Otitis Media

A, D, E Rationale: Children exposed to secondhand smoke are at an increased risk of otitis​ media, upper respiratory tract​ infections, and cancer. Individuals who actually smoke are at risk for developing wrinkles in the skin and yellowing of the fingers. Next Question

Which body change should the nurse recognize as related to nicotine addiction? SATA A) Increase in BP B) Decrease in HR C) Decrease in BP D) Bronchitis E) Increase in HR

A, D, E Rationale: Use of nicotine causes​ vasoconstriction, which results in increased heart rate and blood pressure as well as peripheral vascular​ resistance, thereby increasing the workload of the heart.​ Long-term effects include​ hypertension, bronchitis, and possible lung cancer.

A pregnant client is thinking about stopping cocaine use. The nurse determines that teaching about drug use and pregnancy has been effective when the client makes which statement? 1. " Right after birth, I'll give the baby up for adoption." 2. " I'll help the baby get through the withdrawal period." 3. " I don't want the baby to have withdrawal symptoms." 4. " It's scary to think the baby may have Down syndrome."

3. Neonates born to mothers addicted to cocaine have withdrawal symptoms at birth. If the client says she'll give the baby up for adoption after birth or help the baby get through the withdrawal period, the teaching was ineffective because the mother doesn't see the impact of her drug use on the child. Use of cocaine Lisko, Susan (2013-10-01). NCLEX-RN Questions and Answers Made Incredibly Easy (Nclexrn Questions & Answers Made Incredibly Easy) (Kindle Locations 16328-16330). Lippincott Williams & Wilkins. Kindle Edition.

A client asks the nurse, "Why does it matter if I talk to my peers in group therapy?" What is the most appropriate response by the nurse? 1. " Group therapy lets you see what you're doing wrong in your life." 2. " Group therapy acts as a defense against your disorganized behavior." 3. " Group therapy provides a way to ask for support as well as to support others." 4. " In group therapy, you can vent your frustrations and others will listen."

3. The best response addresses how group therapy provides opportunities to communicate, learn, and give and get support. Group members will give a client feedback, not just point out what a client is doing wrong. Group therapy isn't a defense against disorganized behavior. People can express all kinds of feelings and discuss a variety of topics in group therapy. Interactions are goal oriented and not just vehicles to vent one's frustrations. Lisko, Susan (2013-10-01). NCLEX-RN Questions and Answers Made Incredibly Easy (Nclexrn Questions & Answers Made Incredibly Easy) (Kindle Locations 16161-16164). Lippincott Williams & Wilkins. Kindle Edition.

The difference between Twelve-Step Facilitation (TSF) and Alcoholics Anonymous (AA) is that: TSF groups follow 12 steps to recovery and AA groups do not. TSF groups are smaller than AA groups. AA groups are leaderless and TSF groups are led by mental health professionals. AA groups are always made up of anonymous individuals, whereas TSF groups meet regularly with the same members, and identity disclosure is part of the process.

AA groups are leaderless and TSF groups are led by mental health professionals

25. A nurse is working with a client on recognizing the relationship between alcohol abuse and interpersonal problem. Which of the following is the priority intervention? 1. Help the client identify personal strengths. 2. Help the client decrease compulsive behaviors. 3. Examine the client's use of defense mechanisms. 4. Have the client work with peers who can serve as role models.

3. Defense mechanisms can impede the development of healthy relationships and cause the client pain. After identifying barriers to relationship problems, it would be appropriate to identify or clarify personal strengths. Compulsive behavior doesn't tend to be a problem for alcoholic clients who struggle with interpersonal problems. Working with peers who are role models would be useful after the client recognizes and gains some insight into the problems. It isn't the priority intervention. Lisko, Susan (2013-10-01). NCLEX-RN Questions and Answers Made Incredibly Easy (Nclexrn Questions & Answers Made Incredibly Easy) (Kindle Locations 16085-16089). Lippincott Williams & Wilkins. Kindle Edition.

25. A nurse is working with a client on recognizing the relationship between alcohol abuse and interpersonal problem. Which of the following is the priority intervention? 1. Help the client identify personal strengths. 2. Help the client decrease compulsive behaviors. 3. Examine the client's use of defense mechanisms. 4. Have the client work with peers who can serve as role models.

3. Defense mechanisms can impede the development of healthy relationships and cause the client pain. After identifying barriers to relationship problems, it would be appropriate to identify or clarify personal strengths. Compulsive behavior doesn't tend to be a problem for alcoholic clients who struggle with interpersonal problems. Working with peers who are role models would be useful after the client recognizes and gains some insight into the problems. It isn't the priority intervention. Lisko, Susan (2013-10-01). NCLEX-RN Questions and Answers Made Incredibly Easy (Nclexrn Questions & Answers Made Incredibly Easy) (Kindle Locations 16085-16089). Lippincott Williams & Wilkins. Kindle Edition.

A group of teenagers tell the school nurse they used cocaine because they were bored. What is the most important goal for the nurse? 1. Prepare a drug lecture. 2. Restrict school privileges. 3. Establish an activity schedule. 4. Report the incident to their parents. Lisko, Susan (2013-10-01). NCLEX-RN Questions and Answers Made Incredibly Easy (Nclexrn Questions & Answers Made Incredibly Easy) (Kindle Locations 16376-16380). Lippincott Williams & Wilkins. Kindle Edition.

3. Having an activity schedule enables the adolescents to develop coping skills to make better choices about what to do with their free time. Preparing a drug lecture or restricting school privileges won't be seen as useful by the adolescents and may inadvertently contribute to their inappropriate behavior. As the nurse works with the adolescents, it would be more effective to have the children talk to their parents about their drug use. Lisko, Susan (2013-10-01). NCLEX-RN Questions and Answers Made Incredibly Easy (Nclexrn Questions & Answers Made Incredibly Easy) (Kindle Locations 16381-16384). Lippincott Williams & Wilkins. Kindle Edition.

A 20-year-old client is admitted with bone marrow depression. He tells the nurse he's been abusing drugs since age 13. The nurse reviews the client's history for use of which drug? 1. Amphetamines 2. Cocaine 3. Inhalants 4. Marijuana

3. Inhalants cause severe bone marrow depression. Marijuana, cocaine, and amphetamines don't cause bone marrow depression. Lisko, Susan (2013-10-01). NCLEX-RN Questions and Answers Made Incredibly Easy (Nclexrn Questions & Answers Made Incredibly Easy) (Kindle Locations 16272-16273). Lippincott Williams & Wilkins. Kindle Edition.

The family of an adolescent who smokes marijuana asks a nurse if the use of marijuana leads to abuse of other drugs. What is the most appropriate response by the nurse? 1. " Use of marijuana is a stage your child will go through." 2. " Many people use marijuana and don't use other street drugs." 3. " Use of marijuana can lead to abuse of more potent substances." 4. " It's difficult to answer that question as I don't know your child."

3. Marijuana is considered a "gateway drug" because it tends to lead to the abuse of more potent drugs. People who use marijuana tend to use or at least experiment with more potent substances. Marijuana isn't a part of a developmental stage that adolescents go through. It isn't important that the nurse knows the child. Lisko, Susan (2013-10-01). NCLEX-RN Questions and Answers Made Incredibly Easy (Nclexrn Questions & Answers Made Incredibly Easy) (Kindle Locations 16318-16320). Lippincott Williams & Wilkins. Kindle Edition.

A client asks the nurse, "Why does it matter if I talk to my peers in group therapy?" What is the most appropriate response by the nurse? 1. " Group therapy lets you see what you're doing wrong in your life." 2. " Group therapy acts as a defense against your disorganized behavior." 3. " Group therapy provides a way to ask for support as well as to support others." 4. " In group therapy, you can vent your frustrations and others will listen." Lisko, Susan (2013-10-01). NCLEX-RN Questions and Answers Made Incredibly Easy (Nclexrn Questions & Answers Made Incredibly Easy) (Kindle Locations 16154-16159). Lippincott Williams & Wilkins. Kindle Edition.

3. The best response addresses how group therapy provides opportunities to communicate, learn, and give and get support. Group members will give a client feedback, not just point out what a client is doing wrong. Group therapy isn't a defense against disorganized behavior. People can express all kinds of feelings and discuss a variety of topics in group therapy. Interactions are goal oriented and not just vehicles to vent one's frustrations. Lisko, Susan (2013-10-01). NCLEX-RN Questions and Answers Made Incredibly Easy (Nclexrn Questions & Answers Made Incredibly Easy) (Kindle Locations 16161-16164). Lippincott Williams & Wilkins. Kindle Edition.

A client recovering from alcohol abuse needs to develop effective coping skills to handle daily stressors. What is the most appropriate nursing intervention for this client? 1. Determine the client's level of verbal skills. 2. Help the client avoid areas that cause conflict. 3. Discuss examples of successful coping behavior. 4. Teach the client to accept uncomfortable situations.

3. The client needs help to identify successful coping behavior and develop ways to incorporate that behavior into daily functioning. There are many skills for coping with stress, and determining the client's level of verbal skills may not be important. Encouraging the client to avoid conflict prevents him from learning skills to handle daily stressors. Lisko, Susan (2013-10-01). NCLEX-RN Questions and Answers Made Incredibly Easy (Nclexrn Questions & Answers Made Incredibly Easy) (Kindle Locations 16065-16068). Lippincott Williams & Wilkins. Kindle Edition.

A client recovering from alcohol abuse needs to develop effective coping skills to handle daily stressors. What is the most appropriate nursing intervention for this client? 1. Determine the client's level of verbal skills. 2. Help the client avoid areas that cause conflict. 3. Discuss examples of successful coping behavior. 4. Teach the client to accept uncomfortable situations. Lisko, Susan (2013-10-01). NCLEX-RN Questions and Answers Made Incredibly Easy (Nclexrn Questions & Answers Made Incredibly Easy) (Kindle Locations 16060-16064). Lippincott Williams & Wilkins. Kindle Edition.

3. The client needs help to identify successful coping behavior and develop ways to incorporate that behavior into daily functioning. There are many skills for coping with stress, and determining the client's level of verbal skills may not be important. Encouraging the client to avoid conflict prevents him from learning skills to handle daily stressors. Lisko, Susan (2013-10-01). NCLEX-RN Questions and Answers Made Incredibly Easy (Nclexrn Questions & Answers Made Incredibly Easy) (Kindle Locations 16065-16068). Lippincott Williams & Wilkins. Kindle Edition.

The nurse is facilitating a family meeting for a client who abuses alcohol. During the meeting, the nurse observes the communication and determines an unhealthy pattern of: 1. use of descriptive jargon. 2. disapproval of behaviors. 3. avoidance of conflicting issues. 4. unlimited expression of nonverbal communication.

3. The interaction pattern of a family with a member who abuses alcohol often revolves around denying the problem, avoiding conflict, or rationalizing the addiction. Health care providers are more likely to use jargon. The family might have problems setting limits and expressing disapproval of the client's behavior. Nonverbal communication often gives the nurse insight into family dynamics. CN: Psychosocial integrity; CNS: Lisko, Susan (2013-10-01). NCLEX-RN Questions and Answers Made Incredibly Easy (Nclexrn Questions & Answers Made Incredibly Easy) (Kindle Locations 16171-16175). Lippincott Williams & Wilkins. Kindle Edition.

The nurse is facilitating a family meeting for a client who abuses alcohol. During the meeting, the nurse observes the communication and determines an unhealthy pattern of: 1. use of descriptive jargon. 2. disapproval of behaviors. 3. avoidance of conflicting issues. 4. unlimited expression of nonverbal communication. Lisko, Susan (2013-10-01). NCLEX-RN Questions and Answers Made Incredibly Easy (Nclexrn Questions & Answers Made Incredibly Easy) (Kindle Locations 16166-16170). Lippincott Williams & Wilkins. Kindle Edition.

3. The interaction pattern of a family with a member who abuses alcohol often revolves around denying the problem, avoiding conflict, or rationalizing the addiction. Health care providers are more likely to use jargon. The family might have problems setting limits and expressing disapproval of the client's behavior. Nonverbal communication often gives the nurse insight into family dynamics. CN: Psychosocial integrity; CNS: Lisko, Susan (2013-10-01). NCLEX-RN Questions and Answers Made Incredibly Easy (Nclexrn Questions & Answers Made Incredibly Easy) (Kindle Locations 16171-16175). Lippincott Williams & Wilkins. Kindle Edition.

A client recovering from cocaine abuse is participating in group therapy. The nurse determines that the client has benefited from the therapy when the client makes which statement? 1. " I think the laws about drug possession are too strict in this country." 2. " I'll be more careful about talking about my drug use to my children." 3. " I finally realize the short high from cocaine isn't worth the depression." 4. " I can't understand how I could get all these problems that we talked about in group." Lisko, Susan (2013-10-01). NCLEX-RN Questions and Answers Made Incredibly Easy (Nclexrn Questions & Answers Made Incredibly Easy) (Kindle Locations 16484-16489). Lippincott Williams & Wilkins. Kindle Edition.

3. This is a realistic appraisal of a client's experience with cocaine and how harmful the experience is. The first option indicates the client was distracting self from personal issues and isn't working on goals in the group setting. Talking about drugs to children must be reinforced with nonverbal behavior, and not talking about drugs may give children the wrong message about drug use. The fourth option indicates the client is in denial about the consequences of cocaine use. Lisko, Susan (2013-10-01). NCLEX-RN Questions and Answers Made Incredibly Easy (Nclexrn Questions & Answers Made Incredibly Easy) (Kindle Locations 16491-16494). Lippincott Williams & Wilkins. Kindle Edition.

The nurse is trying to determine if a client who abuses heroin has any drug-related problems. What is the most appropriate question for the nurse to ask? 1. " When did your spouse become aware of your use of heroin?" 2. " Do you have a probation officer that you report to periodically?" 3. " Have you experienced any legal violations while being intoxicated?" 4. " Do you have a history of frequent visits with the employee assistance program manager?"

3. This question focuses on obtaining direct information about drug-related legal problems. When a spouse becomes aware of a partner's substance abuse, the first action isn't necessarily to institute legal action. Even if the client reports to a probation officer, the offense isn't necessarily a drug-related problem. Asking if the client has a history of frequent visits with the employee assistance program manager isn't useful. It assumes any visit to the employee assistance program manager is related to drug issues.

A client who abuses alcohol tells a nurse, "I'm sure I can become a social drinker." What is the most appropriate response by the nurse? 1. " When do you think you can become a social drinker?" 2. " What makes you think you'll learn to drink normally?" 3. " Does your alcohol use cause major problems in your life?" 4. " How many alcoholic beverages can a social drinker consume?"

3. This question may help the client recall the problematic results of using alcohol and the reasons the client began treatment. Asking when he believes he can become a social drinker will only encourage the addicted person to deny the problem and develop an unrealistic, self-defeating goal. Asking how many alcoholic beverages a social drinker can consume and why the client thinks he can drink normally will encourage the addicted person to defend himself and deny the problem. Lisko, Susan (2013-10-01). NCLEX-RN Questions and Answers Made Incredibly Easy (Nclexrn Questions & Answers Made Incredibly Easy) (Kindle Locations 15852-15855). Lippincott Williams & Wilkins. Kindle Edition.

A client who abuses alcohol tells a nurse, "I'm sure I can become a social drinker." What is the most appropriate response by the nurse? 1. " When do you think you can become a social drinker?" 2. " What makes you think you'll learn to drink normally?" 3. " Does your alcohol use cause major problems in your life?" 4. " How many alcoholic beverages can a social drinker consume?"

3. This question may help the client recall the problematic results of using alcohol and the reasons the client began treatment. Asking when he believes he can become a social drinker will only encourage the addicted person to deny the problem and develop an unrealistic, self-defeating goal. Asking how many alcoholic beverages a social drinker can consume and why the client thinks he can drink normally will encourage the addicted person to defend himself and deny the problem. Lisko, Susan (2013-10-01). NCLEX-RN Questions and Answers Made Incredibly Easy (Nclexrn Questions & Answers Made Incredibly Easy) (Kindle Locations 15852-15855). Lippincott Williams & Wilkins. Kindle Edition.

The nurse has just completed an assessment of a client recovering from alcohol addiction who has limited coping skills. During the assessment, the nurse also identified that the client is experiencing relationship problems. This assessment is supported by which finding? 1. The client is prone to panic attacks. 2. The client doesn't pay attention to details. 3. The client has poor problem-solving skills. 4. The client ignores the need to relax and rest.

3. To have satisfying relationships, a person must be able to communicate and problem solve. Relationship problems don't predispose people to panic attacks more than other psychosocial stressors. Paying attention to details isn't a major concern when addressing the client's relationship difficulties. Although ignoring the need for rest and relaxation is unhealthy, it shouldn't pose a major relationship problem. Lisko, Susan (2013-10-01). NCLEX-RN Questions and Answers Made Incredibly Easy (Nclexrn Questions & Answers Made Incredibly Easy) (Kindle Locations 16097-16100). Lippincott Williams & Wilkins. Kindle Edition.

The nurse has just completed an assessment of a client recovering from alcohol addiction who has limited coping skills. During the assessment, the nurse also identified that the client is experiencing relationship problems. This assessment is supported by which finding? 1. The client is prone to panic attacks. 2. The client doesn't pay attention to details. 3. The client has poor problem-solving skills. 4. The client ignores the need to relax and rest. Lisko, Susan (2013-10-01). NCLEX-RN Questions and Answers Made Incredibly Easy (Nclexrn Questions & Answers Made Incredibly Easy) (Kindle Locations 16091-16096). Lippincott Williams & Wilkins. Kindle Edition.

3. To have satisfying relationships, a person must be able to communicate and problem solve. Relationship problems don't predispose people to panic attacks more than other psychosocial stressors. Paying attention to details isn't a major concern when addressing the client's relationship difficulties. Although ignoring the need for rest and relaxation is unhealthy, it shouldn't pose a major relationship problem. Lisko, Susan (2013-10-01). NCLEX-RN Questions and Answers Made Incredibly Easy (Nclexrn Questions & Answers Made Incredibly Easy) (Kindle Locations 16097-16100). Lippincott Williams & Wilkins. Kindle Edition.

A client has been admitted to a detoxification unit for treatment of alcohol withdrawal. He is estranged from his wife, who has taken their two young children to her parents' house. Because of his alcohol use, he has also recently lost his job, which was the sole source of income for the family. His wife refuses to return home until he commits to sobriety. The nurse concludes that which independent nursing action(s) would be appropriate for this client? (Select all that apply.) A) Assist the client to perform activities of daily living (ADLs). B) Educate the wife about signs and symptoms of alcohol dependence and withdrawal. C) Encourage the client to discuss triggers for relapse. D) Educate the client about signs and symptoms of detoxification. E) Encourage the wife to discuss triggers for relapse.

ACE

39. Which nursing intervention relates to rehabilitative care for a recovering alcoholic? 1) Providing a safe and supportive environment during alcohol withdrawal 2) Teaching about physical symptoms 3) Providing client and family education and assistance during treatment 4) Encouraging continued participation in AA

4) Encouraging continued participation in AA Because recovery is a long-term process, it is critical that the nurse encourage continuous participation in outpatient support systems such as AA.

34. Which symptom would the nurse expect to observe in a client experiencing opioid intoxication? 1) Insomnia 2) Abdominal cramps 3) Muscle aches 4) Impaired judgment

4) Impaired judgment Impaired judgment; initial euphoria followed by apathy; dysphoria; and psychomotor agitation or retardation are all symptoms of opioid intoxication.

47. On admission, a client experienced severe alcohol withdrawal symptoms. Four days later, the nurse notes a decrease in withdrawal symptoms. Which nursing intervention is most appropriate? 1) Withhold potentially addictive as needed (prn) medications. 2) Increase prn medications because potentially fatal complications can still occur. 3) Ask the doctor to prescribe a less addictive medication to reduce potential for dependence. 4) Monitor for withdrawal complications and administer medications on the basis of client symptoms.

4) Monitor for withdrawal complications and administer medications on the basis of client symptoms. The nurse must remain vigilant because withdrawal complications can occur days after initial withdrawal symptoms appear. Medication dosages for withdrawal should be based on an objective assessment of symptoms. This is usually done by the use of an assessment tool such as Clinical Institute Withdrawal Assessment (CIWA).

42. Which is the most serious symptom experienced during alcohol withdrawal? 1) Blackout 2) Acute withdrawal delirium 3) Hypotension 4) Seizure

4) Seizure During alcohol withdrawal, the central nervous system (CNS) rebounds from the effects of suppression caused by alcohol intake. This excitation of the CNS can lead to grand mal seizures and other complications, which are life threatening. This is the most serious complication of alcohol withdrawal syndrome.

53. A client is brought to the ED. The client is aggressive, has slurred speech, and exhibits impaired motor coordination. Blood alcohol level is 347 mg/dL. Among the physician's orders is thiamine. Which is the rationale for this intervention? 1) To prevent nutritional deficits 2) To prevent pancreatitis 3) To prevent alcoholic hepatitis 4) To prevent Wernicke's encephalopathy

4) To prevent Wernicke's encephalopathy Wernicke's encephalopathy is the most serious form of thiamine deficiency in clients diagnosed with alcoholism. If thiamine replacement therapy is not undertaken quickly, death will ensue.

The nurse is preparing a teaching plan for a client who abused alcohol. What is the most important information for the nurse to include? 1. Personal needs 2. Illness exacerbation 3. Cognitive distortions 4. Communication skills Lisko, Susan (2013-10-01). NCLEX-RN Questions and Answers Made Incredibly Easy (Nclexrn Questions & Answers Made Incredibly Easy) (Kindle Locations 16113-16116). Lippincott Williams & Wilkins. Kindle Edition.

4. Addicted clients typically have difficulty communicating their needs in an appropriate way. Learning appropriate communication skills is a major goal of treatment. Next, behavior that focuses on the self and meeting personal needs will be addressed. The identification of cognitive distortions would be difficult if the client has poor communication skills. Teaching about illness exacerbation isn't a skill, but it is essential for relaying information about relapse. Lisko, Susan (2013-10-01). NCLEX-RN Questions and Answers Made Incredibly Easy (Nclexrn Questions & Answers Made Incredibly Easy) (Kindle Locations 16117-16121). Lippincott Williams & Wilkins. Kindle Edition.

A client with an alcohol addiction requests a prescription for disulfiram (Antabuse). To determine the client's ability to take the drug appropriately, the nurse should assess which of the following? 1. Whether the client will take a prescription drug 2. Whether the client's family accepts the use of this treatment strategy 3. Whether the client is willing to follow the necessary dietary restrictions 4. Whether the client is motivated to stay sober

4. A client with a strong craving for alcohol (and a lack of impulse control) isn't a good candidate for disulfiram therapy. Disulfiram is a prescription drug. Accepting the treatment strategy is a decision that the client and health care provider make; although family input may be welcome, family members don't make the final decision. Significant dietary restrictions aren't necessary during disulfiram therapy (except for alcohol and foods prepared or cooked in it).

78. A client with an alcohol addiction requests a prescription for disulfiram (Antabuse). To determine the client's ability to take the drug appropriately, the nurse should assess which of the following? 1. Whether the client will take a prescription drug 2. Whether the client's family accepts the use of this treatment strategy 3. Whether the client is willing to follow the necessary dietary restrictions 4. Whether the client is motivated to stay sober

4. A client with a strong craving for alcohol (and a lack of impulse control) isn't a good candidate for disulfiram therapy. Disulfiram is a prescription drug. Accepting the treatment strategy is a decision that the client and health care provider make; although family input may be welcome, family members don't make the final decision. Significant dietary restrictions aren't necessary during disulfiram therapy (except for alcohol and foods prepared or cooked in it). Lisko, Susan (2013-10-01). NCLEX-RN Questions and Answers Made Incredibly Easy (Nclexrn Questions & Answers Made Incredibly Easy) (Kindle Locations 16615-16618). Lippincott Williams & Wilkins. Kindle Edition.

78. A client with an alcohol addiction requests a prescription for disulfiram (Antabuse). To determine the client's ability to take the drug appropriately, the nurse should assess which of the following? 1. Whether the client will take a prescription drug 2. Whether the client's family accepts the use of this treatment strategy 3. Whether the client is willing to follow the necessary dietary restrictions 4. Whether the client is motivated to stay sober .

4. A client with a strong craving for alcohol (and a lack of impulse control) isn't a good candidate for disulfiram therapy. Disulfiram is a prescription drug. Accepting the treatment strategy is a decision that the client and health care provider make; although family input may be welcome, family members don't make the final decision. Significant dietary restrictions aren't necessary during disulfiram therapy (except for alcohol and foods prepared or cooked in it). Lisko, Susan (2013-10-01). NCLEX-RN Questions and Answers Made Incredibly Easy (Nclexrn Questions & Answers Made Incredibly Easy) (Kindle Locations 16615-16618). Lippincott Williams & Wilkins. Kindle Edition.

A client with a history of cocaine abuse exhibits behavior changes following return from an inpatient treatment facility. The nurse anticipates that the physician will order which test? 1. Antibody screen 2. Glucose screen 3. Hepatic screen 4. Urine screen

4. A urine toxicology screen would show the presence of cocaine in the body. Glucose, hepatic, or antibody screening wouldn't show the presence of cocaine in the body. Lisko, Susan (2013-10-01). NCLEX-RN Questions and Answers Made Incredibly Easy (Nclexrn Questions & Answers Made Incredibly Easy) (Kindle Locations 16337-16338). Lippincott Williams & Wilkins. Kindle Edition.

25. Which is the priority nursing intervention for a client admitted for acute alcohol intoxication? A. Darken the room to reduce stimuli in order to prevent seizures. B. Assess aggressive behaviors in order to intervene to prevent injury to self or others. C. Administer lorazepam (Ativan) to reduce the rebound effects on the central nervous system. D. Teach the negative effects of alcohol on the body.

ANS: B Symptoms associated with the syndrome of alcohol intoxication include but are not limited to aggressiveness, impaired judgment, impaired attention, and irritability. Safety is a nursing priority in this situation.

A client asks a nurse not to tell his parents about his alcohol problem. What is the most appropriate response by the nurse? 1. " How can you not tell them? Is that being honest?" 2. " Don't you think you'll need to tell them someday?" 3. " Do alcohol problems run in either side of your family?" 4. " What do you think will happen if you tell your parents?"

4. Clients who struggle with addiction problems often believe people will be judgmental, rejecting, and uncaring if they are told that the client is recovering from alcohol abuse. The first option challenges the client and will put him on the defensive. The second option will make the client defensive and construct rationalizations as to why his parents don't need to know. The third option is a good assessment question, but it isn't an appropriate question to ask a client who's afraid to tell others about his addiction. Lisko, Susan (2013-10-01). NCLEX-RN Questions and Answers Made Incredibly Easy (Nclexrn Questions & Answers Made Incredibly Easy) (Kindle Locations 15863-15866). Lippincott Williams & Wilkins. Kindle Edition.

The nurse is developing interventions to prevent a client who abused alcohol from relapsing. What is the most important intervention for the client? 1. Avoid taking over-the-counter medications. 2. Limit monthly contact with the family of origin. 3. Refrain from becoming involved in group activities. 4. Avoid people, places, and activities from the former lifestyle.

4. Changing the client's old habits is essential for sustaining a sober lifestyle. Certain over-the-counter medications that don't contain alcohol will probably need to be used by the client at certain times. It's unrealistic to have the client abstain from all such medications. Contact with the client's family of origin may not be a trigger to relapse, so limiting contact wouldn't be useful. Refraining from group activities isn't a good strategy to prevent relapse. Going to Alcoholics Anonymous and other support groups will help prevent relapse. Lisko, Susan (2013-10-01). NCLEX-RN Questions and Answers Made Incredibly Easy (Nclexrn Questions & Answers Made Incredibly Easy) (Kindle Locations 16137-16140). Lippincott Williams & Wilkins. Kindle Edition.

The nurse is developing interventions to prevent a client who abused alcohol from relapsing. What is the most important intervention for the client? 1. Avoid taking over-the-counter medications. 2. Limit monthly contact with the family of origin. 3. Refrain from becoming involved in group activities. 4. Avoid people, places, and activities from the former lifestyle. Lisko, Susan (2013-10-01). NCLEX-RN Questions and Answers Made Incredibly Easy (Nclexrn Questions & Answers Made Incredibly Easy) (Kindle Locations 16131-16136). Lippincott Williams & Wilkins. Kindle Edition.

4. Changing the client's old habits is essential for sustaining a sober lifestyle. Certain over-the-counter medications that don't contain alcohol will probably need to be used by the client at certain times. It's unrealistic to have the client abstain from all such medications. Contact with the client's family of origin may not be a trigger to relapse, so limiting contact wouldn't be useful. Refraining from group activities isn't a good strategy to prevent relapse. Going to Alcoholics Anonymous and other support groups will help prevent relapse. Lisko, Susan (2013-10-01). NCLEX-RN Questions and Answers Made Incredibly Easy (Nclexrn Questions & Answers Made Incredibly Easy) (Kindle Locations 16137-16140). Lippincott Williams & Wilkins. Kindle Edition.

A client asks a nurse not to tell his parents about his alcohol problem. What is the most appropriate response by the nurse? 1. " How can you not tell them? Is that being honest?" 2. " Don't you think you'll need to tell them someday?" 3. " Do alcohol problems run in either side of your family?" 4. " What do you think will happen if you tell your parents?"

4. Clients who struggle with addiction problems often believe people will be judgmental, rejecting, and uncaring if they are told that the client is recovering from alcohol abuse. The first option challenges the client and will put him on the defensive. The second option will make the client defensive and construct rationalizations as to why his parents don't need to know. The third option is a good assessment question, but it isn't an appropriate question to ask a client who's afraid to tell others about his addiction. Lisko, Susan (2013-10-01). NCLEX-RN Questions and Answers Made Incredibly Easy (Nclexrn Questions & Answers Made Incredibly Easy) (Kindle Locations 15863-15866). Lippincott Williams & Wilkins. Kindle Edition.

A client says, "I started using cocaine as a recreational drug, but now I can't seem to control the use." The nurse interprets the client's statement as most consistent with which drug behavior? 1. Toxic dose 2. Dual diagnosis 3. Cross-tolerance 4. Compulsive use Lisko, Susan (2013-10-01). NCLEX-RN Questions and Answers Made Incredibly Easy (Nclexrn Questions & Answers Made Incredibly Easy) (Kindle Locations 16250-16253). Lippincott Williams & Wilkins. Kindle Edition.

4. Compulsive drug use involves taking a substance for a period of time significantly longer than intended. A toxic dose is the amount of a drug that causes a poisonous effect. Dual diagnosis is the coexistence of a drug problem and a mental health problem. Cross-tolerance occurs when the effects of a drug are decreased and the client takes larger amounts to achieve the desired drug effect.

Which is the priority nursing intervention for a client admitted for acute alcohol intoxication? A. Darken the room to reduce stimuli in order to prevent seizures. B. Assess aggressive behaviors in order to intervene to prevent injury to self or others. C. Administer lorazepam (Ativan) to reduce the rebound effects on the central nervous system. D. Teach the negative effects of alcohol on the body.

ANS: B Symptoms associated with the syndrome of alcohol intoxication include but are not limited to aggressiveness, impaired judgment, impaired attention, and irritability. Safety is a nursing priority in this situation.

The nurse believes that a client being admitted for a surgical procedure may have a drinking problem. How should the nurse further evaluate this possibility? A. By asking directly if the client has ever had a problem with alcohol B. By holistically assessing the client using the CIWA scale C. By using a screening tool such as the CAGE questionnaire D. By referring the client for physician evaluation

ANS: C The CAGE questionnaire is a screening tool used to determine the diagnosis of alcoholism. This questionnaire is composed of four simple questions. Scoring two or three "yes" answers strongly suggests a problem with alcohol.

28. The nurse believes that a client being admitted for a surgical procedure may have a drinking problem. How should the nurse further evaluate this possibility? A. By asking directly if the client has ever had a problem with alcohol B. By holistically assessing the client, using the CIWA scale C. By using a screening tool such as the CAGE questionnaire D. By referring the client for physician evaluation

ANS: C The CAGE questionnaire is a screening tool used to determine whether the individual has a problem with alcohol. This questionnaire is composed of four simple questions. Scoring two or three yes answers strongly suggests a problem with alcohol.

13. A clients wife has been making excuses for her alcoholic husbands work absences. In family therapy, she states, I just need to work harder to get him there on time. Which is the appropriate nursing response? A. Why do you assume responsibility for his behaviors? B. Codependency is a typical behavior of spouses of alcoholics. C. Your husband needs to deal with the consequences of his drinking. D. Do you understand what the term enabler means?

ANS: C The appropriate nursing response is to use confrontation with caring. In Stage One (The Survival Stage) of recovery from codependency, the codependent person must begin to let go of the denial that problems exist or that his or her personal capabilities are unlimited.

What is the most important assessment for a nurse to implement before starting a teaching session for a client who abuses alcohol? 1. Sleep patterns 2. Decision making 3. Note-taking skills 4. Readiness to learn Lisko, Susan (2013-10-01). NCLEX-RN Questions and Answers Made Incredibly Easy (Nclexrn Questions & Answers Made Incredibly Easy) (Kindle Locations 16123-16126). Lippincott Williams & Wilkins. Kindle Edition.

4. It's important to know if the client's current situation helps or hinders the potential to learn. Decision making and sleep patterns aren't factors that must be assessed before teaching about addiction. Note-taking skills aren't a factor in determining whether the client will be receptive to teaching. Lisko, Susan (2013-10-01). NCLEX-RN Questions and Answers Made Incredibly Easy (Nclexrn Questions & Answers Made Incredibly Easy) (Kindle Locations 16127-16129). Lippincott Williams & Wilkins. Kindle Edition.

A young, depressed adult woman with a history of alcohol abuse is admitted to the hospital after a motor vehicle accident. The nurse performs the admission assessment of the client and anticipates that the history will include which of the following? 1. Defiant responses 2. Infertility 3. Memory loss 4. Sexual abuse

4. Many women diagnosed with substance abuse problems also have a history of physical or sexual abuse. Alcohol abuse isn't a common finding in a young woman showing defiant behavior or experiencing infertility. Memory loss isn't a common finding in a young woman experiencing alcohol abuse. Lisko, Susan (2013-10-01). NCLEX-RN Questions and Answers Made Incredibly Easy (Nclexrn Questions & Answers Made Incredibly Easy) (Kindle Locations 15941-15943). Lippincott Williams & Wilkins. Kindle Edition.

The nurse is caring for a client who uses cocaine and has been admitted to an intensive outpatient rehabilitation program. It is most important for the nurse to assess the client for which finding? 1. GI distress 2. Blurred vision 3. Perceptual distortions 4. Increased appetite .

4. Increased appetite is typical during cocaine or nicotine withdrawal. GI distress (especially nausea and vomiting) occurs during alcohol or opioid withdrawal. Blurred vision isn't typical in cocaine withdrawal. Perceptual distortions are common during withdrawal from phencyclidine (PCP, or "angel dust"), amphetamines, and hallucinogens. Lisko, Susan (2013-10-01). NCLEX-RN Questions and Answers Made Incredibly Easy (Nclexrn Questions & Answers Made Incredibly Easy) (Kindle Locations 16574-16576). Lippincott Williams & Wilkins. Kindle Edition.

What is the most important assessment for a nurse to implement before starting a teaching session for a client who abuses alcohol? 1. Sleep patterns 2. Decision making 3. Note-taking skills 4. Readiness to learn

4. It's important to know if the client's current situation helps or hinders the potential to learn. Decision making and sleep patterns aren't factors that must be assessed before teaching about addiction. Note-taking skills aren't a factor in determining whether the client will be receptive to teaching. Lisko, Susan (2013-10-01). NCLEX-RN Questions and Answers Made Incredibly Easy (Nclexrn Questions & Answers Made Incredibly Easy) (Kindle Locations 16127-16129). Lippincott Williams & Wilkins. Kindle Edition.

A client recovering from alcohol addiction asks the nurse how to talk to his children about the impact of addiction on them. What is the best response by the nurse? 1. " Try to limit references to the addiction and focus on the present." 2. " Talk about all the hardships you've had in working to remain sober." 3. " Tell them you're sorry and emphasize that you're doing so much better now." 4. " Talk to them by acknowledging the difficulties and pain your drinking caused."

4. Part of the healing process for the family is to acknowledge the pain, embarrassment, and overall difficulties the client's drinking problem caused family members. The first option facilitates the client's ability to deny the problem. The second option prevents the client from acknowledging the difficulties the children endured. The third option leads the client to believe only a simple apology is needed. The addiction must be addressed and the children's pain acknowledged. Lisko, Susan (2013-10-01). NCLEX-RN Questions and Answers Made Incredibly Easy (Nclexrn Questions & Answers Made Incredibly Easy) (Kindle Locations 16192-16196). Lippincott Williams & Wilkins. Kindle Edition.

A client recovering from alcohol addiction asks the nurse how to talk to his children about the impact of addiction on them. What is the best response by the nurse? 1. " Try to limit references to the addiction and focus on the present." 2. " Talk about all the hardships you've had in working to remain sober." 3. " Tell them you're sorry and emphasize that you're doing so much better now." 4. " Talk to them by acknowledging the difficulties and pain your drinking caused." Lisko, Susan (2013-10-01). NCLEX-RN Questions and Answers Made Incredibly Easy (Nclexrn Questions & Answers Made Incredibly Easy) (Kindle Locations 16186-16191). Lippincott Williams & Wilkins. Kindle Edition.

4. Part of the healing process for the family is to acknowledge the pain, embarrassment, and overall difficulties the client's drinking problem caused family members. The first option facilitates the client's ability to deny the problem. The second option prevents the client from acknowledging the difficulties the children endured. The third option leads the client to believe only a simple apology is needed. The addiction must be addressed and the children's pain acknowledged. Lisko, Susan (2013-10-01). NCLEX-RN Questions and Answers Made Incredibly Easy (Nclexrn Questions & Answers Made Incredibly Easy) (Kindle Locations 16192-16196). Lippincott Williams & Wilkins. Kindle Edition.

The nurse is preparing a client with the diagnosis of alcohol dependency for discharge from the hospital. What is the most important goal for the client? 1. Find a way to drink socially. 2. Allow self to grieve recent losses. 3. Work to bring others into treatment. 4. Develop relapse-prevention strategies.

4. The primary goal for a client in outpatient treatment is to focus on strategies that prevent relapse. Finding ways to drink socially and working to bring others into treatment aren't goals of outpatient therapy. Allowing self to grieve the losses the addiction caused is a part of the early work of inpatient therapy and may be continued in outpatient therapy. CN: Safe, effective care environment; CNS: Management of care; CL: Analysis Lisko, Susan (2013-10-01). NCLEX-RN Questions and Answers Made Incredibly Easy (Nclexrn Questions & Answers Made Incredibly Easy) (Kindle Locations 16203-16207). Lippincott Williams & Wilkins. Kindle Edition.

The nurse is preparing a client with the diagnosis of alcohol dependency for discharge from the hospital. What is the most important goal for the client? 1. Find a way to drink socially. 2. Allow self to grieve recent losses. 3. Work to bring others into treatment. 4. Develop relapse-prevention strategies. Lisko, Susan (2013-10-01). NCLEX-RN Questions and Answers Made Incredibly Easy (Nclexrn Questions & Answers Made Incredibly Easy) (Kindle Locations 16198-16202). Lippincott Williams & Wilkins. Kindle Edition.

4. The primary goal for a client in outpatient treatment is to focus on strategies that prevent relapse. Finding ways to drink socially and working to bring others into treatment aren't goals of outpatient therapy. Allowing self to grieve the losses the addiction caused is a part of the early work of inpatient therapy and may be continued in outpatient therapy. CN: Safe, effective care environment; CNS: Management of care; CL: Analysis Lisko, Susan (2013-10-01). NCLEX-RN Questions and Answers Made Incredibly Easy (Nclexrn Questions & Answers Made Incredibly Easy) (Kindle Locations 16203-16207). Lippincott Williams & Wilkins. Kindle Edition.

A client who uses cocaine denies that drug use is a problem. What is the best intervention by the nurse? 1. State ways to cope with stress. 2. Repeat the drug facts as needed. 3. Identify the client's ambivalence. 4. Use open-ended, factual questions.

4. The use of open-ended, factual questions will help the client acknowledge that a drug problem is present. Stating ways to cope with stress and identifying the client's ambivalence won't be effective for breaking through a client's denial. Repeating drug facts won't be effective, as the client will perceive it as preaching or nagging. Lisko, Susan (2013-10-01). NCLEX-RN Questions and Answers Made Incredibly Easy (Nclexrn Questions & Answers Made Incredibly Easy) (Kindle Locations 16533-16535). Lippincott Williams & Wilkins. Kindle Edition.

A client who abuses alcohol tells the nurse that everyone in his family has an alcohol problem and nothing can be done about it. What is the most appropriate response by the nurse? 1. " You're right; it's much harder to become a recovering person." 2. " This is just an excuse for you so you don't have to work on becoming sober." 3. " Sometimes, nothing can be done, but you may be the exception in this family." 4. " Alcohol problems can occur in families, but you can decide to take the steps to become and stay sober."

4. This statement challenges the client to become proactive and take the steps necessary to maintain a sober lifestyle. The first option agrees with the client's denial and isn't a useful response. The second option confronts the client and may make him more adamant in defense of this position. The third option agrees with the client's denial and isn't a useful response. Lisko, Susan (2013-10-01). NCLEX-RN Questions and Answers Made Incredibly Easy (Nclexrn Questions & Answers Made Incredibly Easy) (Kindle Locations 15923-15926). Lippincott Williams & Wilkins. Kindle Edition.

A client is brought to the emergency department by the police because of aggressive behavior; slurred speech; and impaired motor coordination; his blood alcohol level is 347 mg/dL. He has been drinking 1 pint of bourbon per day. His wife reports, "He starts drinking in the early afternoon and drinks continuously into the night." It is now 12 AM. When would the nurse expect withdrawal symptoms to begin? A) Between 4 AM and noon B) Around midnight C) In 2 to 3 days D) Around 4 to 6 PM.

A

A mental health nurse has referred a client with an alcohol addiction to a 12-step Alcoholics Anonymous program. Which of the following is a basic concept of a 12-step program? a) Admit life is unmanageable. b) Detoxify from the addictive substance. c) Identify stimuli that promote drinking. d) Include family in counseling sessions.

A

A client who abuses alcohol tells the nurse that everyone in his family has an alcohol problem and nothing can be done about it. What is the most appropriate response by the nurse? 1. " You're right; it's much harder to become a recovering person." 2. " This is just an excuse for you so you don't have to work on becoming sober." 3. " Sometimes, nothing can be done, but you may be the exception in this family." 4. " Alcohol problems can occur in families, but you can decide to take the steps to become and stay sober."

4. This statement challenges the client to become proactive and take the steps necessary to maintain a sober lifestyle. The first option agrees with the client's denial and isn't a useful response. The second option confronts the client and may make him more adamant in defense of this position. The third option agrees with the client's denial and isn't a useful response. Lisko, Susan (2013-10-01). NCLEX-RN Questions and Answers Made Incredibly Easy (Nclexrn Questions & Answers Made Incredibly Easy) (Kindle Locations 15923-15926). Lippincott Williams & Wilkins. Kindle Edition.

A nurse is assessing a client with a history of cocaine abuse. The nurse is aware that the assessment may include which finding? 1. Glossitis 2. Pharyngitis 3. Bilateral ear infections 4. Perforated nasal septum

4. When cocaine is snorted frequently, the client often develops a perforated nasal septum. Bilateral ear infections, pharyngitis, and glossitis aren't common physical findings for a client with a history of cocaine abuse.

5. A nurse is providing teaching to the family of a client who has a substance use disorder. Which of the following statements by a family member indicates a need for further teaching? A. "We need to understand that she is not responsible for her disorder." B. "Eliminating any codependent behavior will promote her recovery." C. "She should participate in an Al-Anon group to help her recover." D. "The primary goal of her treatment is abstinence from substance use."

5. A. INCORRECT: Clients are not responsible for their disease but are responsible for their recovery. B. INCORRECT: Families should be aware of codependent behavior, such as enabling, that can promote substance use rather than recovery. C. CORRECT: Al-Anon is a recovery group for the family of a client, rather than the client who has a substance use disorder. D. INCORRECT: Abstinence is the primary treatment goal for a client who has a substance use disorder.

A nurse is performing a physical assessment on a client who uses heroin. It is most important for the nurse to assess the client for which of the following? 1. Hepatitis 2. Peptic ulcers 3. Hypertension 4. Chronic pharyngitis Lisko, Susan (2013-10-01). NCLEX-RN Questions and Answers Made Incredibly Easy (Nclexrn Questions & Answers Made Incredibly Easy) (Kindle Locations 16425-16428). Lippincott Williams & Wilkins. Kindle Edition.

60. 1. Hepatitis is the most common medical complication of heroin abuse. Peptic ulcers are more likely to be a complication of caffeine use, hypertension is a complication of amphetamine use, and chronic pharyngitis is a complication of marijuana use. Lisko, Susan (2013-10-01). NCLEX-RN Questions and Answers Made Incredibly Easy (Nclexrn Questions & Answers Made Incredibly Easy) (Kindle Locations 16429-16431). Lippincott Williams & Wilkins. Kindle Edition.

61. The family of a client in rehabilitation following heroin withdrawal asks a nurse why the client is receiving naltrexone (ReVia). What is the best response by the nurse? 1. To help reverse withdrawal symptoms 2. To keep the client sedated during withdrawal 3. To take the place of detoxification with methadone 4. To decrease the client's memory of the withdrawal experience Lisko, Susan (2013-10-01). NCLEX-RN Questions and Answers Made Incredibly Easy (Nclexrn Questions & Answers Made Incredibly Easy) (Kindle Locations 16433-16438). Lippincott Williams & Wilkins. Kindle Edition.

61. 1. Naltrexone is an opioid antagonist and helps the client stay drug free. Keeping the client sedated during withdrawal isn't the reason for giving this drug. The drug doesn't decrease the client's memory of the withdrawal experience and isn't used in place of detoxification with methadone. Lisko, Susan (2013-10-01). NCLEX-RN Questions and Answers Made Incredibly Easy (Nclexrn Questions & Answers Made Incredibly Easy) (Kindle Locations 16439-16441). Lippincott Williams & Wilkins. Kindle Edition.

64. A client discusses with the nurse how drug addiction has made life unmanageable. The nurse determines that information to assist the client with coping would include: 1. how peers have committed to sobriety. 2. how to accomplish family of origin work. 3. the addiction process and tools for recovery. 4. how environmental stimuli serve as drug triggers.

64. 3. When the client admits life has become unmanageable, the best strategy is to teach about the addiction, how to obtain support, and how to develop new coping skills. Information about how peers committed to sobriety would be shared with the client as the treatment process begins. Identification of how environmental stimuli serve as drug triggers would be a later part of the treatment process and family of origin work. Initially, the client must commit to sobriety and learn skills for recovery.

The nurse is assessing a pregnant client who states that she smokes a pack of cigarettes each day. Which action by the nurse is the most appropriate? A) Assess the client's readiness to stop B) Ask if she would like a referral to a smoking cessation support group C) Do nothing; smoking is a personal decision D) Encourage the client to cut down to half a pack a day

A) Assess the client's readiness to stop Rationale: Before planning any intervention with a client who​ smokes, it is essential to determine whether or not the client is willing or ready to stop smoking. Smoking can be detrimental to fetal development and is related to increased rate of spontaneous​ abortion, increased incidence of placental​ abruption, being small for gestational​ age, and sudden infant death syndrome​ (SIDS). Informing the client of the risk factors and offering help with a referral to a support group provides​ encouragement, but it is the​ client's decision to quit.​ Commonly, women will stop smoking for the duration of the​ pregnancy, which gives the nurse an opportunity to work with the client to help with permanent smoking cessation. No action results in not providing adequate health promotion information.

The nurse is teaching clients about the health risks associated with nicotine addiction. Which condition should the nurse include? SATA A) Premature aging B) Degenerative disc disease C) Heart disease D) Graves disease E) Kaposi sarcoma

A, B, C, D Rationale: Nicotine addiction is associated with multiple health risks including heart​ disease, lung​ cancer, and chronic obstructive pulmonary disease. Cancers of the​ bladder, colon, and cervix have been linked to smoking. Graves​ disease, infertility, early​ menopause, dysmenorrhea,​ impotence, osteoporosis, and degenerative disc disease have also been associated with smoking. Other less serious consequences include discolored teeth and​ fingernails, premature aging and​ wrinkling, bad​ breath, reduced sense of smell and​ taste, strong smell of smoke clinging to hair and​ clothing, and gum disease. Recent studies have indicated a negative relationship between Kaposi sarcoma and nicotine use.

The community nurse is creating a resource tool kit for a public health website. Which of these organizations should the nurse link to for info on smoking cessation? SATA A) The American Lung Association B) The U.S. Surgeon General's website C) The American Heart Association D) The CDC E) The American Red Cross

A, B, C, D Rationale: The American Red Cross deals with disaster​ mitigation, not smoking cessation. The other organizations are good sources of information about smoking cessation.

The nurse is caring for a client who uses nicotine. Which clinical manifestation should the nurse anticipate when assessing the client? SATA A) Nervousness B) Burns on fingers C) Chronic cough D) Dry, chapped lips E) Shortness of breath on exertion

A, B, C, E Rationale: Assessment findings for nicotine use can include chronic​ cough, nervousness, burns on​ fingers, and shortness of breath on exertion. Lips are not dry and chapped as a result of nicotine​ use, but they can be discolored.

The school nurse is addressing parents about risk factors for children becoming addicted to nicotine. Which factor should be included? SATA A) Alcohol use B) Social Pressure C) Age D) Higher Education E) Emotions

A, B, C, E Rationale: Emotions, social​ pressure, alcohol​ use, and age are all risk factors associated with nicotine use. A lack of​ education, not higher​ education, is also associated with nicotine use.

The nurse develops a discharge plan for a client with COPD and a nicotine addiction. Which nursing intervention is appropriate for promoting smoking abstinence? SATA A) Teaching relaxation techniques B) Providing community resource information C) Assessing airway patency D) Assessing for injury E) Requesting a prescription order for smoking cessation medications

A, B, E Rationale: Consulting with the healthcare provider regarding the possibility of ordering smoking cessation​ medications, teaching relaxation​ techniques, and providing community resource information are all appropriate nursing interventions when promoting smoking abstinence. Assessing for injury and for airway patency does not promote abstinence.

The school nurse is seeking to reduce the rate of smoking among teens in the community. Which method should the nurse recognize as proving effective? SATA A) Increased taxes on cigarettes B) School-based tobacco-use prevention policies C) Celebrities shown smoking in movies D) Parent-led programs E) Counter-advertising mass media commercials

A, B, E Rationale: National,​ state, and local program activities that have reduced and prevented youth tobacco use in the past have included combinations of the​ following: counteradvertising mass media campaigns​ (such as TV and radio​ commercials); comprehensive​ school-based tobacco-use prevention policies and​ programs; and higher costs of tobacco products through increased excise taxes.​ Parent-based programs and showing celebrities glamorizing smoking in movies do not stop teens from smoking.

A nurse is caring for a client with congestive heart failure (CHF) who currently smokes cigarettes and had a 50 pack year smoking history. When providing smoking cessation education to this client, whcih statements regarding the pathophysiology of nicotine use are appropriate? SATA A) "In low doses, nicotine stimulates nicotinic receptors in the brain to release dopamine." B) "In high doses nicotine stimulates the parasympathetic system to release epinephrine, causing vasoconstriction." C) "Initially, nicotine increases mental alertness and cognitive ability." D) "Nicotine is a non-psychoactive substance found in tobacco." E) "Gradual reduction of nicotine intake appears to be the best method of cessation."

A, C Rationale: In low doses, nicotine stimulates nicotinic receptors in the brain to release dopamine and epinephrine, causing vasoconstriction. This response is associated with the sympathetic nervous system, not the parasympathetic nervous system. Initially, nicotine increases mental alertness and cognitive ability, but eventually it depresses those responses. Nicotine is a psychoactive substance found in tobacco. Gradual reduction in nicotine use seems to prolong the suffering of withdrawal and is not a recommended method of cessation.

Which drug and/or behavior should the nurse anticipate as a risk associated with the use of tobacco? SATA A) Cannabis B) Safe sex practices C) Cocaine D) Fighting with others E) Alcohol

A, C, D, E Rationale: Tobacco use has been associated with​ cannabis, alcohol, and cocaine use and with unprotected sexual activity and fighting with​ others, as well as other risky behaviors.

The clinic nurse assesses a pregnant client who has delayed seeking prenatal care. The client states she is addicted to nicotine and is inquiring about therapy. Which therapy should the nurse recommend as options for this client to consider? SATA A) Narcotics anonymous B) Medication therapy C) Group supportive therapy D) Detoxification E) Behavioral therapy

B, C, E Rationale: Behavioral​ therapy, group supportive​ therapy, and medication therapy are beneficial therapies in a client addicted to nicotine. Detoxification is not beneficial and would not be appropriate for the client addicted to nicotine. Nicotine is not a​ narcotic; therefore, a referral to Narcotics Anonymous would not benefit this client.

23. A client is questioning the nurse about a newly prescribed medication, acamprosate calcium (Campral). Which is the most appropriate reply by the nurse? A. This medication will help you maintain your abstinence. B. This medication will cause uncomfortable symptoms if you combine it with alcohol. C. This medication will decrease the effect alcohol has on your body. D. This medication will lower your risk of experiencing a complicated withdrawal.

ANS: A Campral has been approved by the U.S. Food and Drug Administration (FDA) for the maintenance of abstinence from alcohol in clients diagnosed with alcohol dependence who are abstinent at treatment initiation

A client is questioning the nurse about a newly prescribed medication, acamprosate calcium (Campral). Which is the most appropriate reply by the nurse? A. "This medication will help you maintain your abstinence." B. "This medication will cause uncomfortable symptoms if you combine it with alcohol." C. "This medication will decrease the effect alcohol has on your body." D. "This medication will lower your risk of experiencing a complicated withdrawal."

ANS: A Campral has been approved by the U.S. Food and Drug Administration (FDA) for the maintenance of abstinence from alcohol in clients diagnosed with alcohol dependence who are abstinent at treatment initiation.

7. A client diagnosed with chronic alcohol use disorder is being discharged from an inpatient treatment facility after detoxification. Which client outcome related to Alcoholics Anonymous (AA) would be most appropriate for a nurse to discuss with the client during discharge teaching? A. After discharge, the client will immediately attend 90 AA meetings in 90 days. B. After discharge, the client will rely on an AA sponsor to help control alcohol cravings. C. After discharge, the client will incorporate family in AA attendance. D. After discharge, the client will seek appropriate deterrent medications through AA.

ANS: A The most appropriate client outcome for the nurse to discuss during discharge teaching is attending 90 AA meetings in 90 days after discharge. AA is a major self-help organization for the treatment of alcoholism. It accepts alcoholism as an illness and promotes total abstinence as the only cure.

A client diagnosed with chronic alcohol dependency is being discharged from an inpatient treatment facility after detoxification. Which client outcome related to Alcoholics Anonymous (AA) would be most appropriate for a nurse to discuss with the client during discharge teaching? A. After discharge, the client will immediately attend 90 AA meetings in 90 days. B. After discharge, the client will rely on an AA sponsor to help control alcohol cravings. C. After discharge, the client will incorporate family in AA attendance. D. After discharge, the client will seek appropriate deterrent medications through AA.

ANS: A The most appropriate client outcome for the nurse to discuss during discharge teaching is attending 90 AA meetings in 90 days after discharge. AA is a major self-help organization for the treatment of alcoholism. It accepts alcoholism as an illness and promotes total abstinence as the only cure.

18. A client has a history of daily bourbon drinking for the past 6 months. He is brought to an emergency department by family, who report that his last drink was 1 hour ago. It is now 12 midnight. When should a nurse expect this client to exhibit withdrawal symptoms? A. Between 3 a.m. and 11 a.m. B. Shortly after a 24-hour period C. At the beginning of the third day D. Withdrawal is individualized and cannot be predicted.

ANS: A The nurse should expect that this client will begin experiencing withdrawal symptoms from alcohol between 3 a.m. and 11 a.m. Symptoms of alcohol withdrawal usually occur within 4 to 12 hours of cessation or reduction in heavy and prolonged alcohol use.

A client has a history of daily bourbon drinking for the past 6 months. He is brought to an emergency department by family who report that his last drink was 1 hour ago. It is now 12 midnight. When should a nurse expect this client to exhibit withdrawal symptoms? A. Between 3 a.m. and 11 a.m. B. Shortly after a 24-hour period C. At the beginning of the third day D. Withdrawal is individualized and cannot be predicted.

ANS: A The nurse should expect that this client will begin experiencing withdrawal symptoms from alcohol between 3 a.m. and 11 a.m. Symptoms of alcohol withdrawal usually occur within 4 to 12 hours of cessation or reduction in heavy and prolonged alcohol use.

19. A client diagnosed with depression and substance use disorder has an altered sleep pattern and demands that a psychiatrist prescribe a sedative. Which rationale explains why a nurse should encourage the client to first try nonpharmacological interventions? A. Sedative-hypnotics are potentially addictive and will lose their effectiveness due to tolerance. B. Sedative-hypnotics are expensive and have numerous side effects. C. Sedative-hypnotics interfere with necessary REM (rapid eye movement) sleep. D. Sedative-hypnotics are not as effective to promote sleep as antidepressant medications.

ANS: A The nurse should recommend nonpharmacological interventions to this client because sedative-hypnotics are potentially addictive and will lose their effectiveness due to tolerance. The effects of central nervous system depressants are additive with one another and are capable of producing physiological and psychological dependence.

A client diagnosed with depression and substance abuse has an altered sleep pattern and demands that a psychiatrist prescribe a sedative. Which rationale explains why a nurse should encourage the client to first try nonpharmacological interventions? A. Sedative-hypnotics are potentially addictive and will lose their effectiveness due to tolerance. B. Sedative-hypnotics are expensive and have numerous side effects. C. Sedative-hypnotics interfere with necessary REM (rapid eye movement) sleep. D. Sedative-hypnotics are not as effective to promote sleep as antidepressant medications.

ANS: A The nurse should recommend nonpharmacological interventions to this client because sedative-hypnotics are potentially addictive and will lose their effectiveness due to tolerance. The effects of central nervous system depressants are additive with one another and are capable of producing physiological and psychological dependence.

A lonely, depressed divorcée has been self-medicating with cocaine for the past year. Which term should a nurse use to best describe this individual's situation? A. The individual is experiencing psychological dependency. B. The individual is experiencing physical dependency. C. The individual is experiencing substance dependency. D. The individual is experiencing social dependency.

ANS: A The nurse should use the term "psychological dependency" to best describe this client's situation. A client is considered to be psychologically dependent on a substance when there is an overwhelming desire to use a substance in order to produce pleasure or avoid discomfort.

5. A lonely, depressed divorce has been self-medicating with cocaine for the past year. Which term should a nurse use to best describe this individuals situation? A. The individual is experiencing psychological addiction. B. The individual is experiencing physical addiction. C. The individual is experiencing substance addiction. D. The individual is experiencing social addiction.

ANS: A The nurse should use the term psychological addiction to best describe this clients situation. A client is considered to be psychologically addicted to a substance when there is an overwhelming desire to use a substance in order to produce pleasure or avoid discomfort.

1. What should be the priority nursing diagnosis for a client experiencing alcohol withdrawal? A. Risk for injury R/T central nervous system stimulation B. Disturbed thought processes R/T tactile hallucinations C. Ineffective coping R/T powerlessness over alcohol use D. Ineffective denial R/T continued alcohol use despite negative consequences

ANS: A The priority nursing diagnosis for a client experiencing alcohol withdrawal should be risk for injury R/T central nervous system stimulation. Alcohol withdrawal may include the following symptoms: course tremors of hands, tongue, or eyelids; seizures; nausea or vomiting; malaise or weakness; tachycardia; sweating; elevated blood pressure; anxiety; depressed mood; hallucinations; headache; and insomnia.

What should be the priority nursing diagnosis for a client experiencing alcohol withdrawal? A. Risk for injury R/T central nervous system stimulation B. Disturbed thought processes R/T tactile hallucinations C. Ineffective coping R/T powerlessness over alcohol use D. Ineffective denial R/T continued alcohol use despite negative consequences

ANS: A The priority nursing diagnosis for a client experiencing alcohol withdrawal should be risk for injury R/T central nervous system stimulation. Alcohol withdrawal may include the following symptoms: course tremors of hands, tongue, or eyelids; seizures; nausea or vomiting; malaise or weakness; tachycardia; sweating; elevated blood pressure; anxiety; depressed mood; hallucinations; headache; and insomnia.

21. A client is admitted for alcohol detoxification. During detoxification, which symptoms should the nurse expect to assess? A. Gross tremors, delirium, hyperactivity, and hypertension B. Disorientation, peripheral neuropathy, and hypotension C. Oculogyric crisis, amnesia, ataxia, and hypertension D. Hallucinations, fine tremors, confabulation, and orthostatic hypotension

ANS: A Withdrawal is defined as the physiological and mental readjustment that accompanies the discontinuation of an addictive substance. Symptoms can include gross tremors, delirium, hyperactivity, hypertension, nausea, vomiting, tachycardia, hallucinations, and seizures.

A client is admitted for alcohol detoxification. During detoxification, which symptoms should the nurse expect to assess? A. Gross tremors, delirium, hyperactivity, and hypertension B. Disorientation, peripheral neuropathy, and hypotension C. Oculogyric crisis, amnesia, ataxia, and hypertension D. Hallucinations, fine tremors, confabulation, and orthostatic hypotension

ANS: A Withdrawal is defined as the physiological and mental readjustment that accompanies the discontinuation of an addictive substance. Symptoms can include gross tremors, delirium, hyperactivity, hypertension, nausea, vomiting, tachycardia, hallucinations, and seizures.

Which of the following drugs is considered a central nervous system (CNS) depressant? Possible Answers: a. All of these b. Cyclobenzaprine c.Marijuana d. Atenolol

ANS: A All of these All of the drugs listed are considered to be CNS depressants. While generally classified as a psychedelic, marijuana has multiple central depressant effects such as sedation and muscle relaxation. Also, alcohol is another commonly used depressant.

A client's wife has been making excuses for her alcoholic husband's work absences. In family therapy, she states, "His problems at work are my fault." Which is the appropriate nursing response? A. "Why do you assume responsibility for his behaviors?" B. "Codependency is a typical behavior of spouses of alcoholics." C. "Your husband needs to deal with the consequences of his drinking." D. "Do you understand what the term 'enabler' means?"

ANS: C The appropriate nursing response is to use confrontation with caring. The nurse should understand that the client's wife may be in denial and enabling the husband's behavior. Partners of clients with substance abuse must come to realize that the only behavior they can control is their own.

Which of the following nursing statements exemplify the cognitive process that must be completed by a nurse prior to caring for clients diagnosed with substance-abuse disorders? (Select all that apply.) A. "I am easily manipulated and need to work on this prior to caring for these clients." B. "Because of my father's alcoholism, I need to examine my attitude toward these clients." C. "I need to review the side effects of the medications used in the withdrawal process." D. "I'll need to set boundaries to maintain a therapeutic relationship." E. "I need to take charge when dealing with clients diagnosed with substance disorders."

ANS: A, B, D The nurse should examine personal bias and preconceived negative attitudes prior to caring for clients diagnosed with substance-abuse disorders. A deficit in this area may affect the nurse's ability to establish therapeutic relationships with these clients.

29. Which of the following nursing statements exemplify important insights that will promote effective intervention with clients diagnosed with substance use disorders? Select all that apply. A. I am easily manipulated and need to work on this prior to caring for these clients. B. Because of my fathers alcoholism, I need to examine my attitude toward these clients. C. Drinking is legal, so the diagnosis of substance use disorder is an infringement on client rights. D. Opiate addicts are typically uneducated, unrefined individuals who will need a lot of education and social skills training. E. I can fix clients diagnosed with substance use disorders as long as I truly care about them.

ANS: A, B, D The nurse should examine personal bias and preconceived negative attitudes prior to caring for clients diagnosed with substance-abuse disorders. A deficit in this area may affect the nurses ability to establish therapeutic relationships with these clients. A nurse who adopts the attitude that he or she can fix another person may be struggling with codependency issues.

A nursing instructor is teaching students about cirrhosis of the liver. Which of the following student statements about the complications of hepatic encephalopathy should indicate that further student teaching is needed? (Select all that apply.) A. "A diet rich in protein will promote hepatic healing." B. "This condition leads to a rise in serum ammonia resulting in impaired mental functioning." C. "In this condition, blood accumulates in the abdominal cavity." D. "Neomycin and lactulose are used in the treatment of this condition." E. "This condition is caused by the inability of the liver to convert ammonia to urea."

ANS: A, C The nursing instructor should understand that further teaching is needed if the nursing student states that a diet rich in protein will promote hepatic healing and that this condition causes blood to accumulate in the abdominal cavity (ascites), because these are incorrect statements. The treatment of hepatic encephalopathy requires abstention from alcohol, temporary elimination of protein from the diet, and reduction of intestinal ammonia using neomycin or lactulose. This condition occurs in response to the inability of the liver to convert ammonia to urea for excretion.

30. A nursing instructor is teaching students about cirrhosis of the liver. Which of the following student statements about the complications of hepatic encephalopathy should indicate that further student teaching is needed? Select all that apply. A. A diet rich in protein will promote hepatic healing. B. This condition leads to a rise in serum ammonia, resulting in impaired mental functioning. C. In this condition, blood accumulates in the abdominal cavity. D. Neomycin and lactulose are used in the treatment of this condition. E. This condition is caused by the inability of the liver to convert ammonia to urea.

ANS: A, C The nursing instructor should understand that further teaching is needed if the nursing student states that a diet rich in protein will promote hepatic healing or that this condition causes blood to accumulate in the abdominal cavity (ascites), because these are incorrect statements. The treatment of hepatic encephalopathy requires abstention from alcohol, temporary elimination of protein from the diet, and reduction of intestinal ammonia by means of neomycin or lactulose. This condition occurs in response to the inability of the liver to convert ammonia to urea for excretion

A mother who has a history of chronic heroin use has lost custody of her children due to abuse and neglect. She has been admitted to an inpatient substance-abuse program. Which client statement should a nurse associate with a positive prognosis for this client? A. "I'm not going to use heroin ever again. I know I've got the willpower to do it this time." B. "I cannot control my use of heroin. It's stronger than I am." C. "I'm going to get all my children back. They need their mother." D. "Once I deal with my childhood physical abuse, recovery should be easy."

ANS: B A positive prognosis is more likely when a client admits that he or she is addicted to a substance and has a loss of control. One of the first steps in accepting treatment is for the client to admit powerlessness over the substance.

12. A mother who has a history of chronic heroin use has lost custody of her children due to abuse and neglect. She has been admitted to an inpatient drug rehabilitation program. Which client statement should a nurse associate with a positive prognosis for this client? A. Im not going to use heroin ever again. I know Ive got the willpower to do it this time. B. I cannot control my use of heroin. Its stronger than I am. C. Im going to get all my children back. They need their mother. D. Once I deal with my childhood physical abuse, recovery should be easy.

ANS: B A positive prognosis is more likely when a client admits that he or she is addicted to a substance and has a loss of control. One of the first steps in the 12-step model for treatment is for the client to admit powerlessness over the substance.

27. In assessing a client with polysubstance abuse, the nurse should recognize that withdrawal from which substance may require a life-saving emergency intervention? A. Dextroamphetamine (Dexedrine) B. Diazepam (Valium) C. Morphine (Astramorph) D. Phencyclidine (PCP)

ANS: B If large doses of a central nervous system (CNS) depressant (such as Valium) are repeatedly administered over a prolonged duration, a period of CNS hyperexcitability occurs on withdrawal of the drug. The response can be quite severe, even leading to convulsions and death.

In assessing a client diagnosed with polysubstance abuse, the nurse should recognize that withdrawal from which substance may require a life-saving emergency intervention? A. Dextroamphetamine (Dexedrine) B. Diazepam (Valium) C. Morphine (Astramorph) D. Phencyclidine (PCP)

ANS: B If large doses of central nervous system (CNS) depressants (like Valium) are repeatedly administered over a prolonged duration, a period of CNS hyperexcitability occurs on withdrawal of the drug. The response can be quite severe, even leading to convulsions and death.

A nurse evaluates a client's patient-controlled analgesia (PCA) pump and notices 100 attempts within a 30-minute period. Which is the best rationale for assessing this client for substance dependence? A. Narcotic pain medication is contraindicated for all clients with active substance-abuse problems. B. Clients who are dependent on alcohol or benzodiazepines may have developed cross-tolerance to analgesics and require increased doses to achieve effective pain control. C. There is no need to assess the client for substance dependence. There is an obvious PCA malfunction. D. The client is experiencing symptoms of withdrawal and needs to be accurately assessed for lorazepam (Ativan) dosage.

ANS: B The nurse should assess the client for substance dependence because clients who are dependent on alcohol or benzodiazepines may have developed cross-tolerance to analgesics, and require increased doses to achieve effective pain control. Cross-tolerance occurs when one drug lessened the client's response to another drug.

2. A nurse evaluates a clients patient-controlled analgesia (PCA) pump and notices 100 attempts within a 30-minute period. Which is the best rationale for assessing this client for substance use disorder? A. Narcotic pain medication is contraindicated for all clients with active substance-use problems. B. Clients who are regularly using alcohol or benzodiazepines may have developed cross-tolerance to analgesics and require increased doses to achieve effective pain control. C. There is no need to assess the client for substance use disorder. There is an obvious PCA malfunction. D. The client is experiencing symptoms of withdrawal and needs to be accurately assessed for lorazepam (Ativan) dosage.

ANS: B The nurse should assess the client for substance use disorder because clients who are regularly using alcohol or benzodiazepines may have developed cross-tolerance to analgesics and require increased doses to achieve effective pain control. Cross-tolerance occurs when one drug lessens the clients response to another drug.

17. A nurse is reviewing STAT laboratory data of a client presenting in the emergency department. At what minimum blood alcohol level should a nurse expect intoxication to occur? A. 50 mg/dL B. 100 mg/dL C. 250 mg/dL D. 300 mg/dL

ANS: B The nurse should expect that 100 mg/dL is the minimum blood alcohol level at which intoxication occurs. Intoxication usually occurs between 100 and 200 mg/dL. Death has been reported at levels ranging from 400 to 700 mg/dL.

A nurse is reviewing STAT laboratory data of a client presenting in the emergency department. At what minimum blood alcohol level should a nurse expect intoxication to occur? A. 50 mg/dL B. 100 mg/dL C. 250 mg/dL D. 300 mg/dL

ANS: B The nurse should expect that 100 mg/dL is the minimum blood alcohol level at which intoxication occurs. Intoxication usually occurs between 100 and 200 mg/dL. Death has been reported at levels ranging from 400 to 700 mg/dL.

8. A client with a history of heavy alcohol use is brought to an emergency department (ED) by family members who state that the client has had nothing to drink in the last 24 hours. Which client symptom should the nurse immediately report to the ED physician? A. Antecubital bruising B. Blood pressure of 180/100 mm Hg C. Mood rating of 2/10 on numeric scale D. Dehydration

ANS: B The nurse should recognize that high blood pressure is a symptom of alcohol withdrawal and should promptly report this finding to the physician. Complications associated with alcohol withdrawal may progress to alcohol withdrawal delirium and possible seizure activity on about the second or third day following cessation of prolonged alcohol consumption.

A client with a history of heavy alcohol use is brought to an emergency department (ED) by family members who state that the client has had nothing to drink in the last 24 hours. Which client symptom should the nurse immediate report to the ED physician? A. Tactile hallucinations B. Blood pressure of 180/100 mm Hg C. Mood rating of 2/10 on numeric scale D. Dehydration

ANS: B The nurse should recognize that high blood pressure is a symptom of alcohol withdrawal and should promptly report this finding to the physician. Complications associated with alcohol withdrawal may progress to alcohol withdrawal delirium and possible seizure activity on about the second or third day following cessation of prolonged alcohol consumption.

A patient undergoing alcohol rehabilitation decides to begin disulfiram (Antabuse) therapy. Patient teaching should include the need to: (select all that apply) a. avoid aged cheeses. b. avoid alcohol-based skin products. c. read labels of all liquid medications. d. wear sunscreen and avoid bright sunlight. e. maintain an adequate dietary intake of sodium. f. avoid breathing fumes of paints, stains, and stripping compounds.

ANS: B, C, F The patient must avoid hidden sources of alcohol. Many liquid medications, such as cough syrups, contain small amounts of alcohol that could trigger an alcohol-disulfiram reaction. Using alcohol-based skin products such as aftershave or cologne, smelling alcohol-laden fumes, and eating foods prepared with wine, brandy, or beer may also trigger reactions. The other options do not relate to hidden sources of alcohol.

A recovering alcoholic relapses and drinks a glass of wine. The client presents in the emergency department (ED) experiencing severe throbbing headache, tachycardia, flushed face, dyspnea, and continuous vomiting. What may these symptoms indicate to the ED nurse? A. Alcohol poisoning B. Cardiovascular accident (CVA) C. A reaction to disulfiram (Antabuse) D. A reaction to tannins in the red wine

ANS: C Ingestion of alcohol while disulfiram is in the body results in a syndrome of symptoms that can produce a good deal of discomfort for the individual. Symptoms may include but are not limited to flushed skin, throbbing in the head and neck, respiratory difficulty, dizziness, nausea and vomiting, confusion, hypotension, and tachycardia.

22. A recovering alcoholic relapses and drinks a glass of wine. The client presents in the emergency department (ED) experiencing severe throbbing headache, tachycardia, flushed face, dyspnea, and continuous vomiting. What may these symptoms indicate to the ED nurse? A. Alcohol poisoning B. Cardiovascular accident (CVA) C. A reaction to disulfiram (Antabuse) D. A reaction to tannins in the red wine

ANS: C Ingestion of alcohol while disulfiram is in the body results in a syndrome of symptoms that can produce a good deal of discomfort for the individual. Symptoms may include flushed skin, throbbing in the head and neck, respiratory difficulty, dizziness, nausea and vomiting, confusion, hypotension, and tachycardia.

On the first day of a client's alcohol detoxification, which nursing intervention should take priority? A. Strongly encourage the client to attend 90 Alcoholics Anonymous meetings in 90 days. B. Educate the client about the biopsychosocial consequences of alcohol abuse. C. Administer ordered chlordiazepoxide (Librium) in a dosage according to protocol. D. Administer vitamin B1 to prevent Wernicke-Korsakoff syndrome.

ANS: C The priority nursing intervention for this client should be to administer ordered chlordiazepoxide (Librium) in a dosage according to protocol. Chlordiazepoxide (Librium) is a benzodiazepine and is often used for substitution therapy in alcohol withdrawal. Substitution therapy may be required to reduce life-threatening effects of the rebound stimulation of the central nervous system that occurs during withdrawal.

20. A client with a history of insomnia has been taking chlordiazepoxide (Librium), 15 mg, at night for the past year. The client currently reports that this dose is no longer helping him fall asleep. Which nursing diagnosis appropriately documents this problem? A. Ineffective coping R/T unresolved anxiety AEB substance abuse B. Anxiety R/T poor sleep AEB difficulty falling asleep C. Disturbed sleep pattern R/T Librium tolerance AEB difficulty falling asleep D. Risk for injury R/T addiction to Librium

ANS: C Tolerance is defined as the need for increasingly larger or more frequent doses of a substance in order to obtain the desired effects originally produced by a lower dose

A client with a history of insomnia has been taking chlordiazepoxide (Librium) 15 mg at night for the past year. The client currently reports getting to sleep. Which nursing diagnosis appropriately documents this problem? A. Ineffective coping R/T unresolved anxiety AEB substance abuse B. Anxiety R/T poor sleep AEB difficulty falling asleep C. Disturbed sleep pattern R/T Librium tolerance AEB difficulty falling asleep D. Risk for injury R/T addiction to Librium

ANS: C Tolerance is defined as the need for increasingly larger or more frequent doses of a substance in order to obtain the desired effects originally produced by a lower dose.

6. Which term should a nurse use to describe the administration of a central nervous system (CNS) depressant during alcohol withdrawal? A. Antagonist therapy B. Deterrent therapy C. Codependency therapy D. Substitution therapy

ANS: D A CNS depressant such as Ativan is used during alcohol withdrawal as substitution therapy to prevent life-threatening symptoms that occur because of the rebound reaction of the central nervous system.

Which term should a nurse use to describe the administration of a central nervous system (CNS) depressant during alcohol withdrawal? A. Antagonist therapy B. Deterrent therapy C. Codependency therapy D. Substitution therapy

ANS: D A CNS depressant such as Ativan is used during alcohol withdrawal as substitution therapy to prevent life-threatening symptoms that occur because of the rebound reaction of the central nervous system.

Which client statement demonstrates positive progress toward recovery from substance abuse? A. "I have completed detox and therefore am in control of my drug use." B. "I will faithfully attend Narcotic Anonymous (NA) when I can't control my carvings." C. "As a church deacon, my focus will now be on spiritual renewal." D. "Taking those pills got out of control. It cost me my job, marriage, and children."

ANS: D A client who takes responsibility for the consequences of substance abuse/dependence is making positive progress toward recovery. This client would most likely be in the working phase of the counseling process in which acceptance of the fact that substance abuse causes problems occurs.

9. Which client statement demonstrates positive progress toward recovery from a substance use disorder? A. I have completed detox and therefore am in control of my drug use. B. I will faithfully attend Narcotic Anonymous (NA) when I cant control my cravings. C. As a church deacon, my focus will now be on spiritual renewal. D. Taking those pills got out of control. It cost me my job, marriage, and children.

ANS: D A client who takes responsibility for the consequences of substance use is making positive progress toward recovery. This client would most likely be in the working phase of the counseling process, in which he or she accepts the fact that substance use causes problems

24. A nurse is caring for a client who has been prescribed disulfiram (Antabuse) as a deterrent to alcohol relapse. Which information should the nurse include when teaching the client about this medication? A. Only oral ingestion of alcohol will cause a reaction when taking this drug. B. It is safe to drink beverages that have only 12% alcohol content. C. This medication will decrease your cravings for alcohol. D. Reactions to combining Antabuse with alcohol can occur for as long as 2 weeks after stopping the drug.

ANS: D If Antabuse is discontinued, it is important for the client to understand that the sensitivity to alcohol may last for as long as 2 weeks.

A nurse is caring for a client who has been prescribed disulfiram (Antabuse) as a deterrent to alcohol relapse. Which information should the nurse include when teaching the client about this medication? A. "Only oral ingestion of alcohol will cause a reaction when taking this drug." B. "It is safe to drink beverages that have only 12% alcohol content." C. "This medication will decrease your cravings for alcohol." D. "Reactions to combining Antabuse with alcohol can occur 2 weeks after stopping the drug."

ANS: D If Antabuse is discontinued, it is important for the client to understand that the sensitivity to alcohol may last for as long as 2 weeks.

Which medication to maintain abstinence would most likely be prescribed for patients with an addiction to either alcohol or opioids? a. Bromocriptine (Parlodel) b. Disulfiram (Antabuse) c. Methadone (Dolophine) d. Naltrexone (ReVia)

ANS: D Naltrexone (ReVia) is useful for treating both opioid and alcohol addiction. An opioid antagonist blocks the action of opioids and the mechanism of reinforcement. It also reduces or eliminates alcohol craving.

10. A nurse holds the hand of a client who is withdrawing from alcohol. What is the nurses rationale for this intervention? A. To assess for emotional strength B. To assess for Wernicke-Korsakoff syndrome C. To assess for tachycardia D. To assess for fine tremors

ANS: D The nurse is most likely assessing the client for fine tremors secondary to alcohol withdrawal. Withdrawal from alcohol can also cause headache, insomnia, transient hallucinations, depression, irritability, anxiety, elevated blood pressure, sweating, tachycardia, malaise, coarse tremors, and seizure activity.

A nurse is reviewing the plan of care for a patient experiencing alcohol withdrawal syndrome. What does the nurse expect the plan to include?

Administer thiamine. Administer benzodiazepines. Prepare suction equipment and an Ambu-bag.

Which statement reflects a finding of researchers? The older a person is when he or she starts to use drugs or alcohol, the more likely he or she will be to abuse drugs. Adolescents who have dysfunctional family interactions (e.g., have experienced child abuse, violence in the household, or parental substance abuse) are more likely to use and abuse substances. Adolescents start to drink alcohol under peer pressure to try to fit in and be like their peers. Adolescents use drugs to get back at their parents or to escape their interpersonal problems.

Adolescents who have dysfunctional family interactions (e.g., have experienced child abuse, violence in the household, or parental substance abuse) are more likely to use and abuse substances

A patient's family inquires as to why the patient with abdominal pain received a dose of alprazolam. The nurse explains that this medication was given because the patient exhibited which sign?

Agitation

The nurse is assessing a patient in the emergency department, and would suspect cocaine overdose in the patient who is experiencing which of these?

Agitation Cardiac dysrhythmias

_____ appear(s) to play a role in the shift from drug use to drug abuse and dependence. Serotonin GABA Glutamate All of the answers are correct

All of the answers are correct

Blood alcohol concentration is affected by: the number of drinks consumed. the period over which the drinks are consumed. the individual's gender. All of the answers are correct.

All of the answers are correct.

When used as part of behavioral therapy for addictions, token economies function as a form of A) punishment. B) negative reinforcement. C) positive reinforcement. D) extinction.

Answer: C Token economies are formalized programs in which clients who meet desired outcomes accrue a number of token rewards to exchange for privileges or activities. As such, they represent a form of positive reinforcement, or consequences that increase the likelihood of a particular behavior. In contrast, negative reinforcement involves removing a negative stimulus to increase the chances that a desired behavior will occur. Punishment involves applying negative consequences to cause a decrease in undesirable behavior. Extinction refers to the progressive weakening of an undesirable behavior through repeated nonreinforcement of the behavior.

The nurse is providing care for a client who experienced several fractures as a result of intimate partner violence. Which intervention is the most appropriate to include when planning care for the client? A) Assist the client to devise a safety or escape plan. B) Encourage the client to take charge of the situation. C) Offer to contact outpatient services if the client promises not to return home after discharge. D) Make it clear to the spouse that the couple needs to see a therapist.

Answer: A A client who has been victimized by a partner should have a safety plan. This has the highest priority as the client's life is in danger. The client has no control over the partner, and suggesting that the couple needs to see a therapist may escalate the situation. Encouraging the client to take charge is too general a statement to be helpful; the client needs specific tools to develop a safety plan. It may not be safe and feasible for the client to leave the situation right away, and resources should not be withheld if a client is unable to promise not to return home.

Which diagnostic test might the healthcare team use to determine the full extent of an abuse victim's injuries if the victim complains of abdominal pain? A) Ultrasound B) X-ray C) MRI D) Blood test

Answer: A An ultrasound or CT scan of the abdomen can check for abdominal or organ injuries. An MRI of the spine will show spinal injuries. X-rays can detect fractured bones. Blood tests may be used to detect sexually transmitted diseases

A client with a long history of experiencing domestic violence tells the nurse, "There is no way out for me; this situation will never change." What nursing diagnosis would be most appropriate? A) Powerlessness B) Risk for Other-Directed Violence C) Ineffective Health Maintenance D) Chronic Low Self-Esteem

Answer: A Powerlessness is indicated when the client feels an inability to change the pattern or to leave the situation. The victim may experience health maintenance problems as a result of experiencing domestic violence; however, this is not the primary diagnosis. Some victims will experience self-esteem issues, which are secondary to their feeling of having little or no control over their lives. The client is not at high risk for other-directed violence but is rather at high risk to experience it.

Which theory states that individuals learn violent tendencies through association with others and a reinforcement of abusive behaviors? A) Social learning theory B) Psychopathology theory C) Neurobiology theory D) Environmental theory

Answer: A Social learning theory explains that individuals learn violent tendencies through association with others and a reinforcement of the abusive behavior. Psychopathology theory suggests that some individuals who experience personality disorders and mental illnesses participate in family violence as a result of these illnesses. Neurobiology theory asserts that genetics plays a role in anger modulation and emotion control. Environmental theory is not related to the etiology of abuse.

A client with a history of alcohol abuse is being discharged to a treatment facility. Which prescription does the nurse anticipate for this client? A) Disulfiram B) Naloxone C) Bupropion hydrochloride D) Varenicline

Answer: A Disulfiram (Antabuse) is an abstinence medication that would cause the client to become immediately and violently ill when consuming alcohol. Naloxone is administered to clients who overdose on opiates. Bupropion hydrochloride and varenicline are both medications that assist with smoking cessation.

A nurse is concerned about potential substance abuse by a coworker. Which of the coworker's behaviors would place the clients on the unit at risk for injury? A) The nurse in question frequently volunteers to give medications to clients. B) The nurse in question prefers not to be the "medication nurse" on the shift. C) The nurse in question declines to take scheduled breaks. D) The nurse in question frequently requests the largest client care assignment for the shift.

Answer: A Frequently volunteering to give medications or having excessive medication wasting could be a sign that a nurse is using or diverting drugs. The nurse who is unable or unwilling to manage a large client care assignment or who requests to administer medications could be a substance abuser. Taking frequent or lengthy breaks might signal substance abuse.

A college student attends a seminar on alcohol abuse. Which statement would alert the nurse that the student needs more education? A) "The children of alcoholics have a lower risk of becoming alcoholic." B) "Native Americans are at higher risk of becoming alcoholic." C) "Married college graduates are less likely to become alcoholics." D) "Childless people are more likely than parents to become alcoholics."

Answer: A Genetic and environmental factors put both Native Americans and children of alcoholics at greater risk for developing alcoholism. Married people, college graduates, and people with children are less likely to become alcoholics.

While practicing at an outpatient addiction clinic, the nurse summarizes a diagram in the orientation handbook for another nurse who is new to the clinic. That diagram is reproduced below. Which of the following statements on the part of the new nurse would reflect an appropriate understanding of this diagram? A) "Most abused substances either imitate or block the action of neurotransmitters." B) "In order to be addictive, a substance must cause the release of excess neurotransmitters." C) "Substances that exert antagonistic effects can be used to counteract the addictive tendencies of substances that exert agonistic effects." D) "People with addictive personalities process neurotransmitters differently than people who are less prone to addiction."

Answer: A Most abused substances either mimic or block neurotransmitters at critical receptor sites. These drugs exert agonistic effects if they boost neurotransmitter synthesis, increase neurotransmitter release, or activate receptors that normally respond to neurotransmitters (as illustrated in the top half of the diagram). Conversely, they exert antagonistic effects if they interfere with neurotransmitter release, occupy receptor sites that are normally sensitive to neurotransmitters, or cause leakage of neurotransmitters from synaptic vesicles (as shown in the bottom half of the diagram). Both drugs with agonistic effects and those with antagonistic effects can be addictive, and administering one class of drug will not counteract the addictive tendencies of the other class. Researchers have not identified an addictive personality type.

As compared to men, women are A) less likely to begin regularly using alcohol at an early age. B) likely to wait a greater number of years before entering treatment for alcohol abuse. C) less susceptible to alcohol-related organ damage. D) likely to experience greater cognitive impairment from alcohol consumption.

Answer: D No gender difference has been noted for age at onset of regular alcohol use, but women typically use alcohol for fewer years than men before entering treatment. Compared with men, women experience greater cognitive impairment by alcohol and are more susceptible to alcohol-related organ damage.

A nurse is talking with the adult daughter of an 80-year-old client who was recently discovered to be abusing prescription narcotics. The daughter expresses frustration that this substance abuse wasn't discovered sooner, and she asks the nurse how her father's previous healthcare providers could have overlooked this problem. Which of the following statements would not be appropriate for the nurse to include in her reply? A) "Substance abuse and addiction are almost unheard of among older adults, so few providers would consider the possibility of these diagnoses when working with clients like your father." B) "Older adults often choose not to tell their providers about substance use because they either don't recognize they have a problem or don't feel they need treatment." C) "Diagnosis of substance abuse can be difficult in older clients because their symptoms sometimes mimic those of other disorders." D) "There is currently little research data on substance abuse in the older adult population, which means many providers are unaware of the actual extent of this problem."

Answer: A Substance abuse in older adults is believed to be underestimated, underidentified, underdiagnosed, and undertreated. Until relatively recently, alcohol and prescription drug misuse was not discussed in either the substance abuse or the gerontologic literature. Insufficient knowledge, limited research data, and hurried office visits are cited as reasons why healthcare providers often overlook substance abuse and misuse in this population. Diagnosis is often difficult because symptoms of substance abuse in older individuals sometimes mimic symptoms of other medical and behavioral disorders common among older adults, such as diabetes, dementia, and depression. In addition, many older adults do not seek treatment for substance abuse because they do not feel they need it.

Which of the following tools is considered the best scale for assessing the severity of symptoms of acute alcohol withdrawal? A) CIWA-Ar B) MAST C) CAGE D) B-DAST

Answer: A The Clinical Institute Withdrawal Assessment for Alcohol-Revised (CIWA-Ar) scale is currently recommended as the best scale for assessing the severity of symptoms of acute alcohol withdrawal. The Michigan Alcohol Screening Test (MAST) and the Brief Drug Abuse Screening Test (B-DAST) are used to determine the degree of severity of alcohol abuse or dependence. The CAGE questionnaire is useful when the client may not recognize that he or she has an alcohol problem or is uncomfortable acknowledging it.

A client with alcoholism is receiving court-ordered care in a residential treatment facility. After alcohol is discovered in her room, the client states, "It is not mine." Which responses by the nurse are appropriate in this situation? Select all that apply. A) "You will lose your day pass privileges for this Sunday." B) "We have a video of you accepting the alcohol from your brother." C) "What do you think about sharing this at AA tonight?" D) "You won't be allowed to go to dinner tonight." E) "You have violated our behavior contract."

Answer: A, B, C, E Used with care and a calm attitude, confrontation interferes with the client's ability to use denial or rationalization. Losing privileges is an appropriate consequence for violating the behavior contract. Participation in AA will provide peer feedback and can help the client remain accountable for her behavior. Withholding food is inappropriate, particularly for a client with potential nutritional deficits.

The nurse is collecting data on clients at a clinic. One client has risk factors for substance abuse. What physical sign or signs did the nurse assess that suggest substance abuse in this client? Select all that apply. A) Dilated pupils B) Odor of alcohol on the breath C) Frequent accidents or falls D) Extremely low body weight E) Dressed in jeans and a t-shirt

Answer: A, B, D Physical signs of substance abuse include dilated or constricted pupils, inflamed nasal mucosa, evidence of needle "track marks" or abscesses, poor nutritional status, slurred speech or staggering gait, and an odor of alcohol on the breath. Frequent accidents or falls are behavioral signs of substance abuse. Wearing jeans and a t-shirt is not indicative of substance abuse.

An older adult client is brought into the emergency room after experiencing a fall. The nurse suspects elder abuse. Which assessment findings support the nurse's suspicions? Select all that apply. A) Poor hygiene B) Dehydration C) Intracranial trauma D) Fecal impaction E) Dislocations

Answer: A, B, D, E The nurse suspecting elder abuse would assess for clinical manifestations associated with elder abuse. Some of those clinical manifestations are constant hunger or malnutrition, poor hygiene, social isolation, contractures, dehydration, fecal impaction, fractures, sprains, or dislocations. Intracranial trauma is not a typical clinical manifestation of elder abuse; however, it is a clinical manifestation of child abuse.

A nurse is caring for a client with congestive heart failure (CHF) who currently smokes cigarettes and has a 50 pack-year smoking history. When providing smoking cessation education to this client, which statements regarding the pathophysiology of nicotine use are appropriate? Select all that apply. A) "In low doses, nicotine stimulates nicotinic receptors in the brain to release dopamine." B) "In high doses, nicotine stimulates the parasympathetic system to release epinephrine, causing vasoconstriction." C) "Initially, nicotine increases mental alertness and cognitive ability." D) "Nicotine is a nonpsychoactive substance found in tobacco." E) "Gradual reduction of nicotine intake appears to be the best method of cessation."

Answer: A, C In low doses, nicotine stimulates nicotinic receptors in the brain to release dopamine and epinephrine, causing vasoconstriction. This response is associated with the sympathetic nervous system, not the parasympathetic nervous system. Initially, nicotine increases mental alertness and cognitive ability, but eventually it depresses those responses. Nicotine is a psychoactive substance found in tobacco. Gradual reduction in nicotine use seems to prolong the suffering of withdrawal and is not a recommended method of cessation.

A client who has experienced domestic violence in the past has decided to stop participating in counseling. Which client statement would indicate that therapy has been effective? A) "Everyone knows what my problems are, and there is nothing I can do about it." B) "I am functioning fine now but I know that when problems come up again, I will ask for help." C) "My friends tell me that I have improved so this is a good time to stop." D) "It is so draining to deal with the same painful issues all of the time."

Answer: B The client acknowledging that future problems will come up indicates that the client has gained insight into problems. The client's willingness to ask for help shows that the client is prepared to continue with counseling when new problems arise. Stating that the process is draining and painful suggests that little progress has been made and that the client is looking to avoid the pain. Stating that there is nothing than can be done is fatalistic. Basing termination of treatment on the statements of others places emphasis on others and not on self-evaluation.

The nurse assesses a client with a history of alcoholism who is hospitalized with anorexia, dysphagia, odynophagia, and chest pressure after eating. Which nursing diagnosis is a priority for this client? A) Ineffective Coping B) Imbalanced Nutrition: Less than Body Requirements C) Disturbed Sensory Perception D) Disturbed Thought Processes

Answer: B A client with anorexia and a history of alcoholism is at risk for a diagnosis of Imbalanced Nutrition: Less than Body Requirements. The client's symptoms of dysphagia, odynophagia, and chest pressure after eating further support this diagnosis. Ineffective Coping is a potential diagnosis associated with substance abuse, but it is not suggested by the symptoms given here. Disturbed Thought Processes and Disturbed Sensory Perception are diagnoses appropriate for clients who are experiencing delusions, hallucinations, and illusions associated with delirium tremens, but again, these diagnoses are not suggested by the symptoms listed here.

During visitation on the unit, a nurse is observing the family dynamics of an adolescent client who has an addiction problem. In doing so, the nurse concludes that the family is exhibiting codependence. Which of the following behaviors on the part of the family supports the nurse's conclusion? A) The family is intolerant of any frustration on the part of the client. B) The family engages in actions that enable the client's self-destructive behavior. C) The family is argumentative about seemingly insignificant issues. D) The family exhibits high levels of impatience.

Answer: B Codependence is a cluster of maladaptive behaviors exhibited by the significant others of a substance-abusing individual that serve to protect and perpetuate the abuse. Codependence frequently involves enabling behavior, which is any action an individual takes that either consciously or unconsciously facilitates substance dependence. Although impatience, intolerance of frustration, and argumentative behaviors may also be present in this family, they are generally not related to the cycle of codependence and addiction.

Which of the following terms refers to a physiologic need for a substance that the client cannot control and that results in withdrawal symptoms if the substance is withheld? A) Tolerance B) Dependence C) Addiction D) Codependence

Answer: B Dependence is a physiologic need for a substance that the client cannot control and that results in withdrawal symptoms if the substance is stopped or withheld. Dependence causes the user to develop a physiologic tolerance for the substance, which means the user requires progressively greater quantities of the substance to achieve the same pleasurable effects. Addiction is a psychologic or physical need for a substance or process to the extent that the individual will risk negative consequences in an attempt to meet the need. Codependence is a cluster of maladaptive behaviors exhibited by the significant others of a substance-abusing individual that serves to enable and protect the abuse.

A nurse who runs an addiction treatment group at an inner-city clinic has noticed that many of the group's participants struggle with the stigma attached to their condition. Which of the following statements on the part of the nurse would best help participants overcome this stigma? A) "Relapse is a common feature of substance abuse." B) "Heredity and complex environmental influences predispose some people to substance dependence." C) "People who have an addiction problem cannot be held accountable for their actions." D) "Alcoholics Anonymous and Narcotics Anonymous are both accepted treatment approaches."

Answer: B Many clients with addiction struggle with the stigma associated with their condition; for instance, they may feel that addiction is a form of moral failure or a result of personal weakness. Reassuring clients that addiction is rooted in a combination of heredity and environmental factors may help them overcome some of these false beliefs and move toward recovery. Although many clients with a history of substance abuse do relapse, this information would not help overcome the stigma of addiction, nor would information regarding Alcoholics Anonymous or Narcotics Anonymous. Nurses should strive to hold clients with addiction accountable for their actions; absolving them of responsibility only serves to enable their substance abuse.

The nurse is conducting a health history and wants to determine the client's level of alcohol use. What question from the nurse will provide the greatest amount of information? A) "Are you a heavy drinker?" B) "How many alcoholic beverages do you drink each day?" C) "Is alcohol use a concern for you?" D) "Drinking doesn't cause any problems for you, does it?"

Answer: B Open-ended questions will elicit the greatest amount of information about the client's alcohol use. Asking closed questions that can be answered with a "yes" or "no" will limit the information obtained by the nurse.

Which of the following signs and symptoms would least likely be observed in a client who is experiencing alcohol withdrawal syndrome? A) Anorexia B) Hypotension C) Visual hallucinations D) Hyperthermia

Answer: B Signs and symptoms of alcohol withdrawal syndrome include anorexia, hallucinations, and hyperthermia. Hypotension is not associated with alcohol withdrawal syndrome, but hypertension is.

Why is smoking particularly dangerous for clients with atherosclerosis? A) Smoking causes a direct increase in HDL cholesterol, which further contributes to plaque buildup. B) Smoking causes vasoconstriction, which further impairs tissue oxygenation. C) Smoking causes a direct increase in total cholesterol, which further contributes to plaque buildup. D) Smoking causes a decrease in blood pressure, which further impairs tissue oxygenation.

Answer: B Smoking does not directly affect cholesterol levels. However, it does lead to vasoconstriction and an increase in blood pressure, which further impairs tissue oxygenation in clients with atherosclerosis.

The nurse is teaching a class of adolescents about smoking. Which of the following points is most important to convey to this audience? A) Teenagers and young adults are not strongly influenced by tobacco advertising. B) Smoking is a major cause of lung cancer. C) Increasing the cost of cigarettes is not an effective deterrent to smoking. D) Few young people who smoke regularly during their teenage years continue to use tobacco as adults.

Answer: B Smoking is a main cause of lung cancer, and it also directly contributes to the development of many other diseases, including heart disease and stroke. Teenagers and young adults tend to be strongly influenced by both tobacco advertising and counteradvertising mass media campaigns. Increasing the cost of cigarettes has been found to be an effective deterrent to smoking. Teenagers who smoke regularly tend to continue smoking during adulthood, which makes prevention particularly important among the adolescent population.

An 80-year-old client with heart disease tells the nurse, "I am sick because I sinned by smoking cigarettes." Which response by the nurse is appropriate? A) "Smoking cigarettes isn't a sin. There are many worse habits you could have." B) "Cigarette smoking was socially acceptable when you began smoking. People didn't fully understand the problems it could cause." C) "Why don't we call the hospital chaplain and you can pray about your sins?" D) "You are correct, but it is too late to do anything about it now."

Answer: B This client is in distress and seeking forgiveness. The nurse should offer this forgiveness and a reason why the forgiveness is valid. If the nurse tells the client that it is too late to do anything about the problem, there is a possibility that the client's distress will increase. Suggesting that the hospital chaplain be called for prayer reinforces the idea that smoking is a sin. Saying there are worse habits minimizes the client's concerns and does not offer forgiveness.

A child is admitted to the hospital with physical injuries. Which assessment findings would indicate that the child is a victim of abuse? Select all that apply. A) Confusion B) Missing teeth C) Apprehension when other children cry D) Abrasions to the mouth, lips, and genitalia E) Dehydration

Answer: B, C, D Clinical manifestations of child abuse include abrasions to the mouth, lips, and genitalia; missing teeth; and apprehension when other children cry. Dehydration and confusion are manifestations of elder abuse.

The nurse is providing care to a client who admits to smoking two packs of cigarettes per day for 34 years. The client also has a history of intermittent claudication, chronic bronchitis, and emphysema. After 6 weeks of smoking cessation, the client reports that he is frequently "yelling" at his spouse and "flying off the handle." Which effects of cigarette smoking are associated with the data the nurse has collected from the client? Select all that apply. A) Nicotine causes destruction of the alveoli. B) Smoking triggers the release of epinephrine, which causes vasoconstriction. C) Dopaminergic processes are implicated in nicotine withdrawal. D) The tar in cigarettes can cause the mucus production seen in chronic bronchitis. E) Tobacco use leads to atherosclerosis.

Answer: B, C, D, E Nicotine causes the release of dopamine (a precursor to norepinephrine) and epinephrine. These substances trigger vasoconstriction, which exacerbates intermittent claudication. Tobacco use also causes atherosclerosis, which is seen in intermittent claudication. Tar and other chemicals, not nicotine, cause the destruction of the alveoli seen in emphysema and the productive cough seen in chronic bronchitis. Dopaminergic processes have a role in regulating the reinforcing effects of nicotine, making cessation difficult and contributing to withdrawal symptoms including nervousness, restlessness, irritability, impatience, and increased hostility.

A nurse educator is teaching a group of students about the comprehensive theory of addiction proposed by George Engel. Which of the following student statements indicate proper understanding of this theory? Select all that apply. A) "Addiction occurs because of a lack of emotional attachment." B) "There is a biological factor involved in the development of addiction." C) "There are social factors that contribute to the development of addiction." D) "There is a moral factor involved in the development of addiction." E) "There are psychologic elements involved in the development of addiction."

Answer: B, C, E George Engel is credited with proposing the biopsychosocial model of addiction. The biopsychosocial model is supported by current research and takes a more holistic view, theorizing that biological, psychologic, and social factors all contribute to the development of addiction. The view of addiction as a moral disease is nontherapeutic. Viewing addiction as only a behavioral or emotional problem oversimplifies a complex issue.

The nurse is conducting a crisis assessment for a client who admits to cocaine use. Which questions are appropriate for the nurse to ask the client during this process? Select all that apply. A) "Do you have access to any recreational centers?" B) "What is the most significant problem affecting your life right now?" C) "How long has this been a problem?" D) "What are the living conditions in your neighborhood?" E) "What other stresses are you dealing with?"

Answer: B, C, E When conducting a crisis assessment for a client who admits to using a substance that is associated with addiction, the nurse should ask about the most significant problem in the client's life right now, how long this problem has been occurring, and what other stresses may be affecting the client. Questions about recreational centers and neighborhood living conditions are more appropriate for a community crisis assessment than for an individual crisis assessment.

The nurse is providing education to a client who wants to quit smoking. Which statements are appropriate for the nurse to include in the teaching session? Select all that apply. A) "There is no adverse risk if you choose to smoke while wearing a nicotine patch." B) "Bupropion (Zyban) is used to suppress the craving for tobacco." C) "A piece of nicotine gum should be chewed for 5 minutes of every waking hour, then held in the cheek." D) "Most people quit smoking several times before they are successful." E) "Alternative therapies can help reduce the stress that accompanies smoking cessation."

Answer: B, D, E When teaching clients about smoking cessation, the nurse should emphasize that most people who quit smoking try to quit several times before they are successful. Bupropion is used to suppress the craving for tobacco and may be a viable option for this client. The proper use of nicotine gum is to take one piece when the urge to smoke occurs, up to 9-12 times daily. The gum should be chewed several times to soften it, then held in the buccal space for at least 30 minutes to absorb the medication. A client who is wearing a nicotine patch must not smoke because of increased risk for cardiovascular problems, including myocardial infarction. The nurse should always consider alternative therapies in addition to traditional therapies, because they may help the client better deal with the stress that accompanies smoking cessation.

A pediatric nurse is caring for an 8-month-old client. While making rounds, the nurse enters the room and finds the infant's father violently shaking the infant. The father attempts to make it appear as though the infant was choking. Upon further assessment, the nurse notes bruised areas on the infant's arms and legs. What is a priority action for the nurse to take? A) Discuss what the nurse witnessed with the infant's mother. B) Discuss what the nurse witnessed with the other nurses. C) Report what the nurse witnessed and assessed to the authorities. D) Call security to remove the father from the room

Answer: C Because of mandatory reporting laws, nurses must report all suspected cases of child abuse to the appropriate authorities. It would not be appropriate at this time to discuss the findings with the infant's mother or with other nurses. The nurse should also not call security to remove the father from the room until after the abuse has been reported.

The nurse is completing a morning assessment on an older adult Asian client. Assessment findings reveal circular red welts over the client's upper back with several bruised areas. Which nursing action is the most appropriate? A) Contact adult protective services. B) Call the healthcare provider immediately. C) Assess the client's cultural traditions. D) Contact the client's family.

Answer: C The most appropriate action for the nurse at this time is to assess the client's cultural traditions. The practice of cupping is generally practiced by many Asian cultures, as well as individuals who participate in holistic healing. Cupping is the act of placing a glass cup on the skin, and then using heat to create suction; often this is performed to promote blood flow and overall healing. The result of the procedure can be circular red welts or even dark bruising, which are often found along the individual's back. This treatment is not abusive in nature, but rather a form of healing.

Which of the following characteristics would increase a client's risk of unpleasant side effects from nicotine replacement therapy (NRT)? A) Male gender B) Heavy smoking prior to beginning NRT C) Low body weight D) History of failed attempts at smoking cessation

Answer: C A potential issue with NRT is that some transdermal patches and gums are designed for heavy smokers and may contain too large of a dose of nicotine. If clients have low body weight or did not smoke heavily prior to starting NRT, they may experience unpleasant side effects from ingesting too much nicotine. Male gender and a history of failed attempts at smoking cessation do not increase the risk of side effects.

A pregnant client tells the nurse that she and her husband are going to a 50th wedding anniversary party for her grandparents this weekend. The client asks the nurse if it will be okay for her to have a few glasses of wine at the party. Which response by the nurse is appropriate? A) "Drinking a few glasses of wine will not be a problem." B) "Consuming alcohol during pregnancy can cause the baby to be born without limbs." C) "Drinking any alcoholic beverages of any type during pregnancy puts your baby at risk for injury." D) "Wine is acceptable but not hard liquor."

Answer: C Drinking any alcohol, no matter what type and what quantity, during pregnancy increases the risk for accidents and damage to the infant. Women should be encouraged to drink no alcohol at all during pregnancy. Wine can put the mother and fetus at risk as much as hard liquor.

Which of the following statements is true with regard to vaping? A) Vaping is a safe alternative to cigarette smoking. B) E-cigarettes are less popular among teenagers than cigarettes and other traditional tobacco products. C) Vaping has been linked to a devastating respiratory disease known as "popcorn lung." D) Throughout the United States, vaping is subject to the same regulations as cigarette smoking

Answer: C It remains to be proven whether e-cigarettes are actually safe or simply less harmful than tobacco. In many parts of the United States, vaping is unregulated. The popularity of e-cigarettes is exploding among teenagers and young adults; in 2014, e-cigarette use surpassed current use of every other tobacco product overall. Flavored e-cigarettes frequently contain diacetyl, a chemical that causes the debilitating respiratory disease bronchiolitis obliterans, also known as "popcorn lung."

A nurse is caring for a client who smokes cigarettes and wants information about nicotine replacement therapy (NRT). Which statement is appropriate for the nurse to include in the teaching session? A) "Over-the-counter NRT products include transdermal patches, gums, nicotine inhalers, and nasal sprays." B) "NRT helps relieve the psychologic and physiologic effects of nicotine withdrawal." C) "NRT does not address addictive behavior." D) "Using NRT in conjunction with a smoking cessation program is no more effective than using NRT alone."

Answer: C Nicotine replacement therapy (NRT) does not address addictive behavior. Although it helps relieve some physiologic effects of nicotine withdrawal, it does not address the psychologic effects. Over-the-counter NRT products include transdermal patches and gums; nicotine inhalers and nasal sprays are available by prescription only. Using NRT in conjunction with a smoking cessation program is more effective than using NRT alone.

A community health nurse is providing teaching to the faculty of a local high school about preventing, recognizing, and treating substance use and addiction in teenagers. Which of the following statements on the part of the faculty members suggests that further teaching is necessary? A) "The earlier a teenager begins using substances, the more likely it is that the teenager will develop an addiction problem." B) "Teenagers whose parents suffer from addiction are at greater risk of addiction themselves." C) "The brain stops developing during the teenage years, so any substance-related brain changes that occur during this period will likely affect a person for the rest of his or her life." D) "Group therapy can be beneficial for teenagers with addiction, but it comes with a risk of unintended adverse effects."

Answer: C The human brain continues developing well into a person's mid-20s. The relative immaturity of the adolescent brain helps explain why teenagers are more likely than adults to engage in thrill-seeking, high-risk behaviors such as substance use. Teenagers who start using substances early or who have parents who struggle with addiction are more likely to become addicted themselves. Group therapy can be a useful treatment method for teenage clients, but it must be led by a trained facilitator to ensure that group members don't steer conversation toward talk that glorifies or extols substance use.

A nurse works at a clinic that provides care to a community with a high population of smokers. The nurse is planning an educational session entitled "Tips for Quitting." Which action by the nurse is appropriate for inclusion in this session? A) Telling participants that smoking is an unacceptable behavior B) Making sure participants are aware of the increased risk of liver disease and esophageal cancer associated with smoking C) Reviewing available pharmacologic adjuncts to cessation with participants D) Recommending that participants seek hypnosis at a local dinner theater to aid in their attempts at smoking cessation

Answer: C The nurse should include information about available pharmacologic adjuncts to cessation in a "Tips for Quitting" session. Simply telling the participants that smoking is unacceptable is not effective in promoting wellness. Although it is true that smoking contributes to cancer and liver disease, this information will not help participants identify and apply effective strategies for tobacco cessation. Hypnosis can be useful for clients who are trying to quit smoking, but the nurse should recommend that clients seek hypnosis only from a trained healthcare professional.

Which of the following nursing interventions would be most appropriate when caring for clients who are experiencing acute alcohol withdrawal? A) Avoiding administration of benzodiazepines B) Reassuring clients who are experiencing delusions that you share their beliefs C) Reducing environmental stimuli to the greatest extent possible D) Arguing with clients who are experiencing visual hallucinations in an attempt to prove that what they are seeing is not real

Answer: C When caring for clients who are experiencing acute alcohol withdrawal, the nurse should minimize environmental stimuli to decrease the likelihood of agitation. The nurse should also administer benzodiazepines according to the detoxification protocol to help minimize the discomfort of withdrawal symptoms; express reasonable doubt if clients relay suspicious or paranoid beliefs; and avoid arguing with clients who are experiencing delusions or hallucinations.

The nurse is providing care to a client with a history of chronic obstructive pulmonary disease (COPD) who wants help and information regarding nicotine addiction and ways to quit smoking. After the nurse has provided education regarding smoking cessation, which client statement would indicate appropriate understanding of the information presented? A) "I will keep a pack of cigarettes in my closet in case I need it." B) "I will taper off smoking gradually." C) "I will chew sugar-free gum when I want a cigarette." D) "I will eat a snack when I am feeling nervous."

Answer: C When providing education regarding smoking cessation, it is important for the nurse to include adaptive coping mechanisms for the client to use during times of stress. Expressing the intention to use a healthy coping mechanism—such as chewing sugar-free gum—when the urge to smoke arises indicates appropriate understanding of the information presented. Tapering off smoking and keeping cigarettes close by are examples of behaviors that indicate the client is not wholly committed to cessation. Eating when stressed may lead the client to substitute eating for smoking, which is a form of denial.

The nurse is providing care for a 2-year-old client. When assessing the client's risk for abuse, which factors increase this client's risk? Select all that apply. A) The child has bruises on the knees and shins. B) The child's parents are married. C) The child is less than 3 years old. D) The child is deaf. E) The child's parents are unemployed and receive medical assistance.

Answer: C, D, E Risk factors for child abuse include poverty, age less than 3 years, and child disability or condition that requires a great deal of care. Marriage of the parents and bruises on the knees and shins are not risk factors for abuse.

The nurse is evaluating a list of outcome goals for a client with alcoholism who is being discharged from a detoxification program. The list was written by a nursing student who is being mentored by the nurse. Which of the following outcomes are appropriate for this client? Select all that apply. A) Follow a 2000-calorie high-carbohydrate diet. B) Sponsor a participant in Alcoholics Anonymous (AA) meetings. C) Obtain at least 6-8 hours of sleep per night. D) Acknowledge the blame that family members must take for codependent behavior. E) Enroll in the Employee Assistance Program (EAP) through the client's employer.

Answer: C, E Appropriate outcomes for a client who is being discharged from alcohol detoxification are to obtain at least 6-8 hours of sleep per night and to enroll in an Employee Assistance Program if one is offered through the client's employer. The client's calorie requirement should be individualized and may not be 2000 calories. New or returning AA members should be sponsored; they are not ready to sponsor another person. The client should accept responsibility for his or her behavior in the family unit instead of assigning blame for codependent behavior.

The nurse is providing care to a client diagnosed with alcoholism. The client's physical examination reveals a BMI of 18. Which prescription does the nurse anticipate to manage the client's nutritional status? A) Sertraline B) Methadone C) Naloxone D) Multivitamin with folic acid

Answer: D A client with alcohol dependence may suffer from numerous nutritional deficiencies, including deficiencies in thiamine, folic acid, vitamin A, magnesium, and zinc. A multivitamin may be prescribed to help with these deficiencies. Naloxone is used to manage an opiate overdose. Methadone is prescribed to manage heroin cravings. Sertraline is used to reduce anxiety and stabilize mood.

A nurse who works in an outpatient primary care clinic is caring for a client with asthma who has an 80 pack-year smoking history. When assessing the client's current use of nicotine, which question is most appropriate? A) "Have you tried a nicotine patch for quitting smoking?" B) "Do you smoke cigarettes with filters or without?" C) "Do you smoke immediately after waking up?" D) "What prior attempts have you made to quit using nicotine?"

Answer: D Appropriate assessment questions should be open-ended and allow the client to elaborate on the answers. Of the choices provided, only "What prior attempts have you made to quit using nicotine?" meets these criteria.

The nurse is providing care for a client who admits to alcohol addiction. The client states she is able to hide the addiction from family and friends. Based on this information, which independent nursing intervention is appropriate for this client? A) Assertiveness training B) Milieu therapy C) Family therapy D) Communication training

Answer: D Many clients and families with addiction need training in communication skills. Verbal and nonverbal communication training is a vital independent nursing action. Cultural norms must be carefully considered prior to implementing assertiveness training. Milieu therapy and family therapy are interventions that involve collaboration with therapists.

A nurse is teaching a client about the effects of smoking during pregnancy. Upon conclusion of the teaching session, which of the following client statements would suggest that further education is necessary? A) "When a pregnant woman smokes, the concentration of nicotine in the fetus can be even higher than that in the woman's own body." B) "Smoking during pregnancy is associated with an increased risk of placental problems." C) "Prenatal nicotine exposure is linked to attention deficits and learning difficulties during childhood." D) "Although women who smoke during pregnancy are at higher risk for respiratory disease later in life, their children are not."

Answer: D Nicotine crosses the placenta, and fetal concentrations of nicotine can be 15% higher than maternal concentrations. Smoking during pregnancy is associated with increased risk for spontaneous abortion, preterm delivery, respiratory disease, immune system difficulties, and cancer later in life. Various studies link placental complications to prenatal exposure to cigarette smoke. Prenatal tobacco exposure has been associated with serious neurodevelopmental and behavioral consequences in children, including attention deficits and issues with learning and memory.

A client has been admitted to a healthcare facility for treatment for substance addiction. Shortly after entering the facility, the client received a prescription for phenytoin. Based on this data, which of the following statements is most likely true? A) The client is addicted to opioids. B) The client is experiencing cravings for nicotine. C) The client has high levels of anxiety. D) The client is experiencing withdrawal-related seizure activity.

Answer: D Phenytoin is an antiseizure medication that is used to reduce and control seizure activity resulting from withdrawal syndrome. Seizure activity may occur during withdrawal from several different categories of substances, including alcohol and benzodiazepines. Opioid withdrawal usually does not produce seizures. Clients who are experiencing nicotine cravings may receive nicotine replacement therapy and/or antidepressants, not phenytoin. Withdrawal-related anxiety may be treated with benzodiazepines, not antiseizure drugs.

The nurse is planning care for a client with esophageal cancer caused by years of nicotine abuse. Which of the following would be the priority nursing diagnosis for this client? A) Decisional Conflict B) Social Isolation C) Disturbed Body Image D) Ineffective Airway Clearance

Answer: D The nurse should anticipate that a client with esophageal cancer related to tobacco use may have issues with airway edema. Because proper breathing and oxygenation are crucial, Ineffective Airway Clearance would be the highest priority nursing diagnosis of the options listed. There is no evidence that the client has a disturbed body image or experiences decisional conflict or social isolation.

A public health nurse is presenting a teaching session about alcohol use to a group of college seniors. During the session, one of the students admits to frequent alcohol use. What is the nurse's priority action? A) Initiate a community assessment of the campus B) Contact the campus nurse and refer the student for services C) Notify campus security that the student may be driving while intoxicated D) Complete a crisis assessment with the student

Answer: D The student should be assessed to determine the extent of crisis he or she is facing as the result of frequent alcohol use. This crisis assessment will allow the nurse to determine the appropriate course of action. A community assessment is not necessary at this time because the issue appears to be limited to this particular student. Contacting the campus nurse is not advised without the student's permission. There is no evidence that the student is driving while intoxicated.

A nurse is providing preconception counseling and education to an adult client with no prior pregnancies. Which client statement indicates that the nurse's teaching has been effective? A) "I can continue to drink alcohol throughout my pregnancy." B) "One beer once per week will not harm the fetus." C) "I don't need to stop drinking alcohol until my pregnancy is confirmed." D) "I can't drink alcohol while breastfeeding, because it will pass into my breast milk."

Answer: D Women should stop drinking alcohol when they attempt to become pregnant. It is not known how much alcohol will cause fetal damage; therefore, any amount of alcohol, even one beer, during pregnancy is contraindicated. Alcohol passes readily into breast milk; therefore, women should abstain from alcohol while breastfeeding, or the milk should be pumped and dumped after alcohol consumption

A medication for treating alcohol use disorder that induces violent nausea and vomiting when it is mixed with alcohol is: Ritalin. Noproxin. methadone. Antabuse.

Antabuse

When caring for an alcoholic patient, the nurse recognizes that which drugs should be avoided or given with caution?

Antianginals Antihistamines Antihypertensives

An​ 80-year-old client states he drinks a​ six-pack of beer each evening. When reviewing the​ client's daily​ medications, which should concern the nurse the​ most? (Select all that​ apply.) Antidepressant Antihistamine Cough syrup Laxative Antihypertensives

Antidepressant Antihistamine Cough syrup Laxative Mixing prescription and​ over-the-counter medicines or herbal remedies with alcohol can be dangerous or even deadly for older​ adults, who routinely take medications for chronic health issues. The nurse reviewing the medications would be concerned about cough syrup and laxatives because some can contain alcohol and create an additive effect with the​ client's alcohol use. Alcohol combined with some antidepressants can cause respiratory depression. Cold and allergy medications containing antihistamines can cause​ drowsiness, and can cause problems when mixed with alcohol. Antihypertensives generally do not interact with alcohol.

The nurse is assessing an elderly patient who has been prescribed medication for insomnia. Which interventions should the nurse perform?

Asess the patient's knowledge of other medications taken. Explain complications related to mixing alcohol with the medication. Obtain a Short Michigan Alcoholism Screening Test-Geriatric Version (SMAST-G).

The nurse is providing care to a client who admits to smoking two packs of cigarettes per day for 34 years. The client also has a history of intermittent claudication, chronic bronchitis, and emphysema. After 6 weeks of smoking cessation, the client reports that he is frequently "yelling" at his spouse and "flying off the handle." Which effects of cigarette smoking are associated with the data the nurse has collected from the client? Select all that apply. A) Nicotine causes destruction of the alveoli. B) Smoking triggers the release of epinephrine, which causes vasoconstriction. C) Dopaminergic processes are implicated in nicotine withdrawal. D) The tar in cigarettes can cause the mucus production seen in chronic bronchitis. E) Tobacco use leads to atherosclerosis.

B C D E

A client admitted 3 days ago is now experiencing alcohol withdrawal delirium. Which nursing intervention should be implemented to promote safety for the​ client? (Select all that​ apply.) Encouraging the client to verbalize fears Assessing the​ client's level of orientation frequently Teaching the client healthy coping mechanisms Explaining all interventions before approaching the client Using simple​ step-by-step instructions with the client

Assessing the​ client's level of orientation frequently Teaching the client healthy coping mechanisms Using simple​ step-by-step instructions with the client Assessing the​ client's level of orientation​ frequently, explaining all interventions before approaching the​ client, and using simple​ step-by-step instructions with the client are all examples of promoting safety for the client during alcohol withdrawal. Teaching the client healthy coping mechanisms and encouraging the client to verbalize fears are examples of promoting healthy coping skills.

The nurse is completing health screening for alcohol abuse for clients in a primary care clinic. Which client should the nurse prioritize for immediate assessment based on the​ nurse's knowledge of risk factors for alcohol​ abuse? (Select all that​ apply.) A​ 32-year-old client living in a house without heat or water A​ 20-year-old client who recently experienced a sexual assault A​ 19-year-old client who has a​ 2-week-old newborn baby at home A​ 13-year-old girl who has started drinking at parties A​ 24-year-old college student in graduate studies

A​ 32-year-old client living in a house without heat or water A​ 20-year-old client who recently experienced a sexual assault A​ 13-year-old girl who has started drinking at parties

A heroin addict of 8 years lost custody of her first two children due to abuse and neglect secondary to her addiction. She is currently 4 months pregnant with her third child and has been admitted to the inpatient substance-abuse program for 3 days. Which statement best indicates that the client is working on her substance abuse issues? A) "I'm not going to use heroin ever again. I know I've got the will power to do it this time." B) "I cannot control my use of heroin. It's stronger than I am. I'm here to find out how to get started on the road to recovery." C) "I'm going to get all my children back. They need their mother." D) "My father abused me as a child, and my mother walked out on us. If anyone's got a right to use heroin, it's me."

B

An 80-year-old client with heart disease tells the nurse, "I am sick because I sinned by smoking cigarettes." Which response by the nurse is appropriate? A) "Smoking cigarettes isn't a sin. There are many worse habits you could have." B) "Cigarette smoking was socially acceptable when you began smoking. People didn't fully understand the problems it could cause." C) "Why don't we call the hospital chaplain and you can pray about your sins?" D) "You are correct, but it is too late to do anything about it now."

B

The nurse is obtaining an admission history from a patient who has been abusing drugs and alcohol for years, lost his job several months ago. His wife recently left him and is now living with their two children at her parents' house. He tells the nurse, "I wouldn't use drugs and booze if my wife hadn't been nagging me all the time to get a job. She never did think that I was good enough for her." How would the nurse interpret this statement? A) This is a dysfunctional family. B) The client is in denial about the problem. C) The client is under a lot of stress. D) The prognosis for recovery is slim.

B

The nurse is teaching a class of adolescents about smoking. Which of the following points is most important to convey to this audience? A) Teenagers and young adults are not strongly influenced by tobacco advertising. B) Smoking is a major cause of lung cancer. C) Increasing the cost of cigarettes is not an effective deterrent to smoking. D) Few young people who smoke regularly during their teenage years continue to use tobacco as adults.

B

Which process must be completed by the nurse before caring for clients with substance abuse disorders? A) Recognize that their personal potential for addiction is very low. B) Process his/her own attitudes and perceptions regarding substance abusers. C) Avoid being confrontational with clients. D) Encourage clients to identify the role that others play in creating and perpetuating substance abuse.

B

Which rationale explains why a client requiring intervention for pain management should be assessed for a history of substance abuse? A) Narcotic pain medication is not permitted for clients with active substance-abuse problems. B) Clients with certain substance addictions develop a higher tolerance to pain medications, thus may require increased doses to achieve effective pain control. C) Clients with an active substance-abuse disorder have a higher tolerance for pain, so they should be given less medication to achieve effective pain control. D) Clients who use substances should be encouraged to use non-pharmacologic alternative for pain management because they are not permitted to take narcotics.

B

Why is smoking particularly dangerous for clients with atherosclerosis? A) Smoking causes a direct increase in HDL cholesterol, which further contributes to plaque buildup. B) Smoking causes vasoconstriction, which further impairs tissue oxygenation. C) Smoking causes a direct increase in total cholesterol, which further contributes to plaque buildup. D) Smoking causes a decrease in blood pressure, which further impairs tissue oxygenation.

B

An 80-year-old client with heart disease tells the nurse, "I am sick because I sinned by smoking cigarettes." Which response by the nurse is appropriate? A) "Smoking cigarettes isn't a sin. There are many worse habits you could have." B) "Cigarette smoking was socially acceptable when you began smoking. People didn't fully understand the problems it could cause." C) "Why don't we call the hospital chaplain and you can pray about your sins?" D) "You are correct, but it's too late to do anything about it now."

B) "Cigarette smoking was socially acceptable when you began smoking. People didn't fully understand the problems it could cause." Rationale: This client is in distress and seeking forgiveness. The nurse should offer this forgiveness and a reason why the forgiveness is valid. If the nurse tells the client that it is too late to do anything about the problem, there is a possibility that the client's distress will increase. Suggesting that the hospital chaplain be called for prayer reinforces the idea that smoking is a sin. Saying there are worse habits minimizes the client's concerns and does not offer forgiveness.

A client presents with a burn on the right index finger caused by falling asleep with a lit cigarette in hand. Additionally the client reports an inability to relax and insomnia and denies that smoking has negative effects on the body. Which is the priority nursing diagnosis when developing the plan of care? A) Anxiety B) Injury, Risk for C) Knowledge, Deficient D) Sleep Pattern, Disturbed

B) Injury, Risk for Rationale: njury, Risk for is the priority diagnosis. The client has a burn on the right index finger from falling asleep with a lit cigarette in the hand.​ Therefore, safety is a priority concern. The assessment findings also show the client has Anxiety​ (inability to​ relax), Sleep​ Pattern, Disturbed ​(complaints of​ insomnia), and ​Knowledge, Deficient​ (denial that smoking has negative effects on the​ body), but these are not the priority.​ (NANDA-I ©​ 2014)

Why is smoking particularly dangerous for those with atherosclerosis? A) Smoking causes a direct increase in HDL cholesterol, which further contributes to plaque buildup B) Smoking causes vasoconstriction, which further impairs tissue oxygenation C) Smoking causes a direct increase in total cholesterol, which further contributes to plaque build up D) Smoking causes a decrease in blood pressure, which further impairs tissue oxygenation

B) Smoking causes vasoconstriction, which further impairs tissue oxygenation Rationale: Smoking does not directly affect cholesterol levels. However, it does lead to vasoconstriction and an increase in blood pressure, which further impairs tissue oxygenation in clients with atherosclerosis.

The nurse is teaching a class of adolescents about smoking. Which of the following points is most important to convey to this audience? A) Teens and young adults are not strongly influenced by tobacco advertising B) Smoking is a major cause of lung cancer C) Increasing the cost of cigarettes is not an effective deterrent to smoking D) Few young people who smoke regularly during their teen years continue to use tobacco as adults.

B) Smoking is a major cause of lung cancer Rationale: Smoking is a main cause of lung cancer, and it also directly contributes to the development of many other diseases, including heart disease and stroke. Teenagers and young adults tend to be strongly influenced by both tobacco advertising and counteradvertising mass media campaigns. Increasing the cost of cigarettes has been found to be an effective deterrent to smoking. Teenagers who smoke regularly tend to continue smoking during adulthood, which makes prevention particularly important among the adolescent population.

The nurse is incorporating yoga into a smoking cessation class. Which rationale explains the value of yoga for this class? A) Yoga influences nicotinic receptors in the brain B) Yoga reduces anxiety C) Yoga builds flexibility in the muscles D) Yoga increases awareness of spirtuality

B) Yoga reduces anxiety Rationale: Yoga reduces anxiety caused by the craving for nicotine during cessation. While it may build flexibility and raise spiritual​ awareness, that is not the purpose in this class. Yoga does not influence nicotinic receptors in the brain. OK

The public health nurse is preparing a presentation on nicotine addiction. Which nicotine replacement therapies (NRTs) should the nurse state are available over the counter? SATA A) Vaper B) Nicotine patch C) Nicotine gum D) Nicotine inhaler E) Nicotine spray

B, C Rationale: Nicotine patches and nicotine gum are available over the counter. Nicotine spray and nicotine inhalers require a prescription. Vapers are not NRTs. Researchers have yet to determine if vapers are safe or are simply less harmful than tobacco.

The nurse is creating a plan of care for a client undergoing treatment for nicotine addiction. Which expected outcome should the nurse consider appropriate? SATA A) The client denies the negative effects of nicotine on the body B) The client remains free from injury C) The client describes methods of coping without the use of nicotine D) The client voices benefits of starting a smoking cessation program E) The client maintains a patent airway

B, C, D, E Rationale: Expected outcomes for clients with nicotine addiction include a patent​ airway, freedom from​ injury, the ability to describe methods of coping without nicotine​ use, and the ability to voice benefits of smoking cessation programs. The client should also​ verbalize, not​ deny, the negative effects of nicotine on the body.

The nurse is providing care to a client who admits to smoking two packs of cigarettes per day for 34 years. The client also has a history of intermittent claudication, chronic bronchitis, and emphysema. After 6 weeks of smoking cessation, the client reports that he's frequently "yelling" at his spouse and "flying off the handle." Which effects of cigarette smoking are associated with the data the nurse has collected from the client? SATA A) Nicotine causes destruction of the alveoli B) Smoking triggers the release of epinephrine, which causes vasoconstriction C) Dopaminergic processes are implicated in nicotine withdrawal D) The tar in cigarettes can cause the mucus production seen in chronic bronchitis E) Tobacco use leads to atherosclerosis.

B, C, D, E Rationale: Nicotine causes the release of dopamine (a precursor to norepinephrine) and epinephrine. These substances trigger vasoconstriction, which exacerbates intermittent claudication. Tobacco use also causes atherosclerosis, which is seen in intermittent claudication. Tar and other chemicals, not nicotine, cause the destruction of the alveoli seen in emphysema and the productive cough seen in chronic bronchitis. Dopaminergic processes have a role in regulating the reinforcing effects of nicotine, making cessation difficult and contributing to withdrawal symptoms including nervousness, restlessness, irritability, impatience, and increased hostility.

The nurse is providing education to a client who wants to quit smoking. Which statements are appropriate for the nurse to include in the teaching session? SATA A) "There is no adverse risk if you choose to smoke while wearing a nicotine patch." B) "Buproprion (Zyban) is used to suppress the craving for tobacco." C) "A piece of nicotine gum should be chewed for 5 minutes of every waking hour, then held in the cheek." D) "Most people quit smoking several times before they're successful." E) "Alternative therapies can help reduce the stress that accompanies smoking cessation."

B, D, E Rationale: When teaching clients about smoking cessation, the nurse should emphasize that most people who quit smoking try to quit several times before they are successful. Bupropion is used to suppress the craving for tobacco and may be a viable option for this client. The proper use of nicotine gum is to take one piece when the urge to smoke occurs, up to 9-12 times daily. The gum should be chewed several times to soften it, then held in the buccal space for at least 30 minutes to absorb the medication. A client who is wearing a nicotine patch must not smoke because of increased risk for cardiovascular problems, including myocardial infarction. The nurse should always consider alternative therapies in addition to traditional therapies, because they may help the client better deal with the stress that accompanies smoking cessation.

Which drug is most commonly used as a recreational drug to produce euphoric effects?

Barbiturates

_____ is/are used to treat sleep disorders and must be tapered off gradually when the individual wants to discontinue using it/them. Alcohol Marijuana Barbiturates and benzodiazepines Amphetamines

Barbiturates and benzodiazepines

_____ occurs when a person drinks until his or her blood alcohol concentration reaches at least 0.08 percent in a 2-hour period. Alcoholism Binge drinking Drunkenness Impairment

Binge drinking

A patient who is admitted to a primary health care center is being treated with methadone medication. The patient's social history reveals the use of "junk." Which medication does the nurse expect to be prescribed when the patient is discharged?

Buprenorphine

A patient who is prescribed a non-nicotine agent for tobacco withdrawal is demonstrating aggressive behavior and depression. Which medication does the nurse suspect is causing the behavioral changes?

Bupropion

A nurse is caring for a client who smokes cigarettes and wants information about nicotine replacement therapy (NRT). Which statement is appropriate for the nurse to include in the teaching session? A) "Over-the-counter NRT products include transdermal patches, gums, nicotine inhalers, and nasal sprays." B) "NRT helps relieve the psychologic and physiologic effects of nicotine withdrawal." C) "NRT does not address addictive behavior." D) "Using NRT in conjunction with a smoking cessation program is no more effective than using NRT alone."

C

A nurse is discussing alcohol tolerance with a newly licensed nurse. Which of the following statements by the newly licensed nurse indicates understanding? a) "A client who has alcohol tolerance develops physical changes when they haven't recently ingested alcohol." b) "Alcohol tolerance causes the client to have an increased effect when taking opiates." c) "A client who has alcohol tolerance has a decreased physical response to alcohol." d) "Alcohol tolerance is a medical emergency and can develop as a result of withdrawal."

C

A nurse is teaching a client, who currently abuses alcohol, about Alcoholics Anonymous (AA). Which of the following client statements indicates an understanding of the program's basic concepts? a) "I am responsible for my alcoholism." b) "I need to identify things that cause me to be an alcoholic." c) "I am powerless about my addiction to alcohol." d) "I need to see a counselor who will be responsible for my recovery."

C

A nurse manager is preparing to report disciplinary action of a staff nurse for substance abuse. Which of the following has the authority to revoke a professional nurse's license? a) Civil judicial process b) Chief nursing officer of a hospital c) State board of nursing d) American Nurses Association

C

A nurse works at a clinic that provides care to a community with a high population of smokers. The nurse is planning an educational session entitled "Tips for Quitting." Which action by the nurse is appropriate for inclusion in this session? A) Telling participants that smoking is an unacceptable behavior B) Making sure participants are aware of the increased risk of liver disease and esophageal cancer associated with smoking C) Reviewing available pharmacologic adjuncts to cessation with participants D) Recommending that participants seek hypnosis at a local dinner theater to aid in their attempts at smoking cessation

C

In addition to diaphoresis and tremors, which symptoms would the nurse expect to see as a client undergoes alcohol withdrawal? A) Bradycardia and hypertension B) Bradycardia and hypotension C) Tachycardia and hypertension D) Tachycardia and hypotension

C

The nurse is providing care to a client with a history of chronic obstructive pulmonary disease (COPD) who wants help and information regarding nicotine addiction and ways to quit smoking. After the nurse has provided education regarding smoking cessation, which client statement would indicate appropriate understanding of the information presented? A) "I will keep a pack of cigarettes in my closet in case I need it." B) "I will taper off smoking gradually." C) "I will chew sugar-free gum when I want a cigarette." D) "I will eat a snack when I am feeling nervous."

C

The wife of a man with alcoholism says to the nurse: "His problems at work are my fault. I should have called to say he was sick instead of insisting that he go to work, but I was tired of always covering for him." Which is the appropriate nursing response? A) "Why didn't you call in this time?'' B) "Let's deal with the present rather than the past." C) "Your husband needs to deal with the results of his drinking." D) "Do you know you're an enabler when you protect your husband like that?"

C

Which nursing intervention is most appropriate on the first day of alcohol detoxification? A) Strongly encourage the client to attend two AA meetings. B) Educate the client about the biopsychosocial consequences of alcohol abuse. C) Administer Librium for withdrawal symptoms. D) Ensure that the client consumes 95% of his or her meals.

C

Which of the following characteristics would increase a client's risk of unpleasant side effects from nicotine replacement therapy (NRT)? A) Male gender B) Heavy smoking prior to beginning NRT C) Low body weight D) History of failed attempts at smoking cessation

C

The nurse is providing care to a client with a history of COPD who wants help and info regarding nicotine addiction and ways to quit smoking. After the nurse has provided education regarding smoking cessation, whch client statement would indicate appropriate understanding of the information presented? A) "I will keep a pack of cigarettes in my closet in case I need it." B) "I will taper off smoking gradually." C) "I will chew sugar-free gum when I want a cigarette." D) "I will eat a snack when I'm feeling nervous."

C) "I will chew sugar-free gum when I want a cigarette." Rationale: When providing education regarding smoking cessation, it is important for the nurse to include adaptive coping mechanisms for the client to use during times of stress. Expressing the intention to use a healthy coping mechanism—such as chewing sugar-free gum—when the urge to smoke arises indicates appropriate understanding of the information presented. Tapering off smoking and keeping cigarettes close by are examples of behaviors that indicate the client is not wholly committed to cessation. Eating when stressed may lead the client to substitute eating for smoking, which is a form of denial.

A nurse is caring for a client who smokes cigarettes and wants information about nicotine replacement therapy (NRT). Which statement is appropriate for the nurse to include in the teaching session? A) "OTC NRT products include transdermal patches, gums, nicotine inhalers, and nasal sprays." B) "NRT helps relieve the psychologic and physiologic effects of nicotine withdrawal." C) "NRT does not address addictive behavior." D) "Using NRT in conjunction with a smoking cessation program is no more effective than using NRT alone."

C) "NRT does not address addictive behavior." Rationale: Nicotine replacement therapy (NRT) does not address addictive behavior. Although it helps relieve some physiologic effects of nicotine withdrawal, it does not address the psychologic effects. Over-the-counter NRT products include transdermal patches and gums; nicotine inhalers and nasal sprays are available by prescription only. Using NRT in conjunction with a smoking cessation program is more effective than using NRT alone.

Which of the following characteristics would increase a client's risk of unpleasant side effects from nicotine replacement therapy (NRT)? A) Male gender B) Heavy smoking prior to starting NRT C) Low body weight D) History of failed attempts at smoking cessation

C) Low body weight Rationale: A potential issue with NRT is that some transdermal patches and gums are designed for heavy smokers and may contain too large of a dose of nicotine. If clients have low body weight or did not smoke heavily prior to starting NRT, they may experience unpleasant side effects from ingesting too much nicotine. Male gender and a history of failed attempts at smoking cessation do not increase the risk of side effects. B) A potential issue with NRT is that some transdermal patches

A nurse works at a clinic that provides care to a community with a high population of smokers. The nurse is planning an educational session entitled, "Tips for Quitting." Which action by the nurse is appropriate for inclusion in this session? A) Telling participants that smoking is an unacceptable behavior B) Making sure participants are aware of the increased risk of liver disease and esophageal cancer associated with smoking C) Reviewing available pharmacologic adjuncts to cessation with participants D) Recommending that participants seek hypnosis at a local dinner theater to aid in their attempts at smoking cessation

C) Reviewing available pharmacologic adjuncts to cessation with participants Rationale: The nurse should include information about available pharmacologic adjuncts to cessation in a "Tips for Quitting" session. Simply telling the participants that smoking is unacceptable is not effective in promoting wellness. Although it is true that smoking contributes to cancer and liver disease, this information will not help participants identify and apply effective strategies for tobacco cessation. Hypnosis can be useful for clients who are trying to quit smoking, but the nurse should recommend that clients seek hypnosis only from a trained healthcare professional.

Which of the following statements is true with regard to vaping? A) Vaping is a safe alternative to cigarette smoking B) E-cigarettes are less popular among teens than cigarettes and other traditional tobacco products C) Vaping has been linked to a devastating respiratory illness known as "popcorn lung" D) Throughout the United States, vaping is subject to the same regulations as cigarette smoking

C) Vaping has been linked to a devastating respiratory illness known as "popcorn lung." Rationale: It remains to be proven whether e-cigarettes are actually safe or simply less harmful than tobacco. In many parts of the United States, vaping is unregulated. The popularity of e-cigarettes is exploding among teenagers and young adults; in 2014, e-cigarette use surpassed current use of every other tobacco product overall. Flavored e-cigarettes frequently contain diacetyl, a chemical that causes the debilitating respiratory disease bronchiolitis obliterans, also known as "popcorn lung."

●● Standardized Screening Tools ◯◯ MAST (Michigan Alcohol Screening Test) or MAST-G (Michigan Alcohol Screening Test - Geriatric) ◯◯ Addiction Severity Index ◯◯ Recovery Attitude and Treatment Evaluator ◯◯ Drug Abuse Screening Test (DAST) ◯◯ __-__ - This screening tool asks questions of clients to determine how the clients perceives their current substance use.

CAGE-AID

While conducting research studies on the effects of cigarette smoking, a nurse identifies that which conditions can be caused by cigarette smoking?

Cataracts Periodontitis Chronic obstructive pulmonary disease

The nurse is caring for a patient who was admitted for using cocaine. A person who snorts cocaine may experience which of the following complications?

Chronic sinusitis

The nurse notes that a​ client's blood alcohol level​ (BAL) is​ 0.10%. Which symptom supports the​ client's current​ BAL? (Select all that​ apply.) Clumsy behavior Lack of coordination Frequent seizures Difficulty with speech Inability to remain upright

Clumsy behavior Lack of coordination Difficulty with speech Ataxia​ (lack of​ coordination), clumsy​ behavior, and dysarthria​ (difficulty with​ speech) are all symptoms associated with BALs of​ 0.10%. Inability to remain upright is associated with BALs of​ 0.20% to​ 0.25%. Seizures are associated with withdrawal symptoms and are not correlated with a​ client's BAL.

Which statement about incorporating the social factors of substance abuse into treatment is true? Residential treatment is not helpful in learning to change how an individual thinks, feels, and behaves. Cognitive-behavior therapy is only used in individual therapy and not in group therapy. Long-term participation in Alcoholics Anonymous is associated with better outcomes for substance abusers. Outpatient family therapy has not been shown to contribute to achieving abstinence.

Long-term participation in Alcoholics Anonymous is associated with better outcomes for substance abusers.

37. Paula is attending an education class on addictive disorders. She suspects that her husband may be abusing opiates since he has been taking pills given to him by his brother and she knows the brother had been taking oxycodone for back pain. She asks the nurse how to interpret her husband's behaviors. Which of the following observations by Paula are consistent with opioid intoxication? Select all that apply. 1) "Sometimes he seems euphoric and other times he acts like he doesn't care about anything." 2) "Last night he went out without a coat on and it was 15 degrees outside." 3) "While we were talking at dinner his speech was rapid and he seemed hyperalert to everything in the environment." 4) "He's been having trouble remembering things." 5) "Sometimes it looks like his pupils are very small."

Correct 1: One manifestation of opioid intoxication is an initial period of euphoria followed by apathy, which is indicated by her statement that "sometimes he acts like he doesn't care about anything." Correct 2: Going outside without a coat in subfreezing weather could be inferred as impaired judgment, which is consistent with opioid intoxication. Feedback 3: Rapid speech and hypervigilance are more consistent with stimulant intoxication. In opioid intoxication one would expect to see mental cloudiness. Correct 4: Impairment in attention and memory is consistent with opioid intoxication. Correct 5: Paula is describing pupillary constriction, which is consistent with opioid intoxication.

40. Which issues influence an individual's predisposition to substance-related disorders? Select all that apply. 1) Genetic history 2) Fixation at the oral stage of psychosexual development 3) Punitive ego 4) Personality traits 5) Behavior modeling

Correct 1: Research has indicated that an apparent hereditary factor is involved in the development of substance-use disorders. This is especially evident with alcoholism. Correct 2: Theories of psychosexual development state that anxiety in people fixated at the oral stage may be reduced by their consumption of substances such as alcohol. Feedback 3: A psychodynamic approach to the etiology of substance abuse focuses on a punitive superego, not ego. According to psychodynamic theory, individuals with punitive superegos turn to alcohol to diminish unconscious anxiety. Correct 4: Certain personality traits, such as low self-esteem, depression, and passivity, are thought to increase a tendency toward addictive behavior. Correct 5: Studies have shown that children and adolescents are more likely to use substances if they have parents who provide a model for substance use.

41. Paul, a 65-year-old Caucasian, is being seen at the health clinic for hypertension and has a history of alcohol use disorder. Which of the following observations by the nurse are consistent with physical complications associated with chronic alcohol use disorders? Select all that apply. 1) His skin is yellow. ) He has a butterfly-shaped rash on his cheeks and nose. 3) His abdomen is distended. 4) He is coughing up blood. 5) He complains of acute pain in his left eye.

Correct 1: Yellowish skin is evidence of jaundice, which is secondary to cirrhosis of the liver. Cirrhosis of the liver is a common manifestation of end-stage alcoholic liver disease. Feedback 2: Although facial flushing is a common manifestation in chronic alcohol use disorders, a distinctly butterfly-shaped rash may be indicative of other autoimmune conditions such as lupus erythematosis. Further assessment is warranted. Correct 3: Abdominal distention can be a manifestation of alcoholic hepatitis, cirrhosis of the liver, and pancreatitis, all of which are complications of alcohol use disorder. Further assessment is warranted. Correct 4: Coughing up blood may be evidence of several complications of alcoholism, including esophageal varices, which can culminate in potentially fatal hemorrhage. Further assessment is warranted to evaluate for these as well as other potential causes of coughing up blood. Feedback 5: A complaint of pain or pressure in or behind one's eyes is not directly associated with alcoholism but suggests a potentially emergent concern that requires further assessment.

The nurse is completing health teaching for a client who has been prescribed an alcohol deterrent. Which food or medication should the nurse instruct the client to avoid while taking this medication​ therapy? Grapefruit juice Nonsteroidal​ anti-inflammatory Cough syrup Analgesics

Cough syrup

A client is brought to the emergency department because of aggressive behavior; slurred speech; and impaired motor coordination; his blood alcohol level is 347 mg/dL. Although he denies that he is an alcoholic, the nurse encourages him to seek rehabilitative treatment. Which step does the nurse anticipate that the client must accomplish first to be successful in treatment? A) Identify someone to whom he can go for support. B) Give up all his old drinking buddies. C) Understand the effects of alcohol on the body. D) Correlate the problems in his life to his use of alcohol.

D

A nurse is teaching a client about the effects of smoking during pregnancy. Upon conclusion of the teaching session, which of the following client statements would suggest that further education is necessary? A) "When a pregnant woman smokes, the concentration of nicotine in the fetus can be even higher than that in the woman's own body." B) "Smoking during pregnancy is associated with an increased risk of placental problems." C) "Prenatal nicotine exposure is linked to attention deficits and learning difficulties during childhood." D) "Although women who smoke during pregnancy are at higher risk for respiratory disease later in life, their children are not."

D

A nurse who works in an outpatient primary care clinic is caring for a client with asthma who has an 80 pack-year smoking history. When assessing the client's current use of nicotine, which question is most appropriate? A) "Have you tried a nicotine patch for quitting smoking?" B) "Do you smoke cigarettes with filters or without?" C) "Do you smoke immediately after waking up?" D) "What prior attempts have you made to quit using nicotine?"

D

A nurse working with substance-abuse clients evaluates which statement as a knowledge deficit? A) "Although it's legal, alcohol is one of the most widely abused drugs in our society." B) "Tolerance to heroin develops quickly, leaving the addict searching to achieve that first-time high." C) "The effects of LSD, including flashbacks and hallucinations, may recur spontaneously weeks or months afterward." D) "Marijuana is like smoking cigarettes. Everyone does it. It's essentially harmless."

D

A recently divorced 43-year-old career woman suffering emotional .upheaval began drinking daily. After losing her job due to her drinking problem, she received treatment for the acute stage of her illness and is being discharged from the hospital. Which outpatient therapy should the nurse plan to discuss in the discharge teaching? A) Aversion therapy B) Controlled drinking C) Detoxification D) Alcoholics Anonymous

D

A nurse is teaching a client about the effects of smoking during pregnancy. Upon conclusion of the teaching session, which of the following client statements would suggest that further education is necessary? A) "When a pregnant woman smokes, the concentration of nicotine in the fetus can be even higher than that in the woman's own body." B) "Smoking during pregnancy is associated with an increased risk of placental problems." C) "Prenatal nicotine exposure is linked to attention deficits and learning difficulties during childhood." D) "Although women who smoke during pregnancy are at a higher risk for respiratory disease later in life, their children are not."

D) "Although women who smoke during pregnancy are at a higher risk for respiratory disease later in life, their children are not." Rationale: Nicotine crosses the placenta, and fetal concentrations of nicotine can be 15% higher than maternal concentrations. Smoking during pregnancy is associated with increased risk for spontaneous abortion, preterm delivery, respiratory disease, immune system difficulties, and cancer later in life. Various studies link placental complications to prenatal exposure to cigarette smoke. Prenatal tobacco exposure has been associated with serious neurodevelopmental and behavioral consequences in children, including attention deficits and issues with learning and memory.

A nurse who works in an outpatient primary care clinic is caring for a client with asthma who has an 80 pack year history. When assessing the client's current use of nicotine, which question is most appropriate? A) "Have you tried a nicotine patch for quitting smoking?" B) "Do you smoke cigarettes with or without filters?" C) "Do you smoke immediately after waking up?" D)" What prior attempts have you made to quit using nicotine?"

D) "What prior attempts have you made to quit using nicotine?" Rationale: Appropriate assessment questions should be open-ended and allow the client to elaborate on the answers. Of the choices provided, only "What prior attempts have you made to quit using nicotine?" meets these criteria.

The nurse is planning crae for a client with esophageal cancer caused by years of nicotine abuse. Which of the following would be the priority nursing diagnosis for this client? A) Decisional Conflict B) Social Isolation C) Disturbed Body Image D) Ineffective Airway Clearance

D) Ineffective Airway Clearance Rationale: The nurse should anticipate that a client with esophageal cancer related to tobacco use may have issues with airway edema. Because proper breathing and oxygenation are crucial, Ineffective Airway Clearance would be the highest priority nursing diagnosis of the options listed. There is no evidence that the client has a disturbed body image or experiences decisional conflict or social isolation.

The nurse assess a client with a nicotine addiction who is irritable, has not slept much lately, and has burns on his clothes. When developing a plan of care, the nurse identifies, Injury, Risk for as the priority nursing diagnosis. Which is the appropriate goal for this client based on the assigned diagnosis? A) The client will not experience complications B) The client will verbalize strategies for smoking cessation C) The client will demonstrate relaxation D) The client will remain free from injury

D) The client will remain free from injury Rationale: Remaining free of injury is a goal appropriate for the diagnosis of ​Injury, Risk for. Demonstrating​ relaxation, verbalizing strategies for smoking​ cessation, and avoiding complications are all appropriate goals based on the assessment​ findings, but not related to the priority diagnosis identified by the nurse.​ (NANDA-I ©​ 2014)

A patient with a history of methaqualone abuse is receiving phenobarbital therapy to manage withdrawal. After completion of the therapy, the nurse expects that the patient will no longer experience what conditions that are associated with withdrawal?

Delirium Convulsions

Which statement about depressant drugs is NOT true? When sedative-hypnotic drugs are mixed with alcohol, the combined effect can be lethal. Depressant drugs can cause memory problems, confusion, poor concentration, fatigue, and respiratory distress. Chronic use of depressants can lead to tolerance and withdrawal symptoms. Depressant drugs reduce aggressive behavior and increase sexual arousal and behavior.

Depressant drugs reduce aggressive behavior and increase sexual arousal and behavior.

A patient who experienced an overdose of methylphenidate develops central nervous system effects. The nurse recognizes that which treatments are beneficial to the patient?

Diazepam Chlorpromazine

A client is hospitalized for a substance abuse disorder. Which intervention should the nurse identify to promote safety for this​ client? (Select all that​ apply.) Discuss coping skills. Obtain a drug history. Obtain samples for drug analysis. Set limits to behavior. Encourage to verbalize fears.

Discuss coping skills. Encourage to verbalize fears.

The nurse suspects that a patient is experiencing alcohol-withdrawal delirium based on what assessment findings?

Disorientation Visual hallucinations Increased hyperactivity without seizures

The nurse is caring for a client with an addiction to cocaine. Which medication should the nurse expect to be​ prescribed? (Select all that​ apply.) Opioids Opioid antagonists Dopamine agonists Antidepressants Antipsychotics

Dopamine agonists Antidepressants

The nurse notes that a​ client's score on the Clinical Institute Withdrawal Assessment of Alcohol Scale​ (CIWA-Ar) has continued to increase. Which assessment would provide the nurse more information about the​ client's condition? Ensuring that the​ CIWA-Ar score has been administered correctly Assessing for tremors and vomiting Assessing for delusions or hallucinations Drawing blood alcohol level​ (BAL) and obtaining vital signs

Drawing blood alcohol level​ (BAL) and obtaining vital signs

The nurse recalls that which drug is prescribed to increase appetite in a patient with acquired immunodeficiency syndrome (AIDS)?

Dronabinol

The nurse recalls that which drug is prescribed to increase appetite in a patient with acquired immunodeficiency syndrome (AIDS)?

Dronabinol is a medication used to stimulate appetite. Thiamine is used to treat Wernicke encephalopathy. Buprenorphine is used to treat opioid withdrawal symptoms. Loperamide is used to reduce diarrhea.

A patient admitted to the emergency department with opioid overdose has severe diarrhea. Which medication does the nurse anticipate being prescribed to the patient to manage the diarrhea?

Loperamide

The nurse is preparing a presentation on substance use disorders for a community group. Which risk factor should the nurse include that predisposes the development of a substance use​ disorder? (Select all that​ apply.) Family history Mental illness Low income Loneliness Divorce

Family history Mental illness Loneliness

The nurse is teaching high school students about substance abuse. Which should the nurse include about the risk factors for becoming addicted to a​ substance? (Select all that​ apply.) Family history of drug abuse Depression Peer pressure Family involvement in​ child's life Poor social skill development in the child

Family history of drug abuse Depression Peer pressure Poor social skill development in the child

46. Pamela has sought treatment for ongoing substance use disorder. She asks the nurse what treatment options are available to help her combat this problem. Which of these options would be accurate for the nurse to include in patient education? Select all that apply. 1) ECT 2) Self-help groups 3) Deterrent therapy 4) Substitution pharmacotherapy 5) Vitamin supplements

Feedback 1: ECT is primarily indicated for the treatment of depression. There is no evidence of its benefit in preventing relapse in substance use disorders. Correct 2: Self-help groups such as Alcoholics Anonymous are commonly recommended as a treatment option for substance use disorders. Correct 3: Deterrent therapy, such as Antabuse to deter alcohol use, is a recognized option for some substance use disorders. Correct 4: Substitution therapy, such as methadone for heroin users, is a recognized option for some substance use disorders. Feedback 5: Vitamin supplements are beneficial in reversing nutritional deficiencies in alcoholism and other substance use disorders but do not combat the problem of substance use disorder itself.

44. Janice is a nurse whose husband is in rehab for alcohol use disorder. While attending a family group, Janice makes several statements about their relationship. Which of these statements would suggest Janice is exhibiting codependent behavior? Select all that apply. 1) "My husband has to accept responsibility for his behavior and the consequences of his drinking." 2) "I know I shouldn't go out drinking with him, but I'm afraid he'll leave me if I don't." 3) "My father was the same way and I learned its better just to keep your mouth shut so you don't get hit." 4) "If he didn't have me monitoring his every move he'd probably be dead already." 5) "I need to make sure I'm protecting myself and my children."

Feedback 1: This statement is an example of healthy boundaries rather than codependent behavior. Correct 2: People-pleasing, fear of abandonment, and neediness, as evidenced in this statement, are all characteristic codependent behaviors. Correct 3: The sense of helplessness and a history of abuse or neglect as a child are consistent with codependency. Correct 4: This statement suggests an unrealistic need to be in control and may also suggest that Janice's self-worth is rooted in her need to be needed. Both of these are evidence of codependency. Feedback 5: Janice's expression of concern for her own safety and her clear identification of her responsibilities as a parent are examples of healthy rather than codependent behaviors.

The nurse has given the patient a dose of lorazepam. The patient is exhibiting respiratory depression. Which medication should the nurse plan to administer?

Flumazenil

The nurse in the employee health clinic knows that nurses are at high risk of developing substance abuse problems. Which should the nurse identify as a reason for this​ risk? (Select all that​ apply.) Frequent contact with drugs Pressures in the workplace Spending time with coworkers outside work Working​ 12-hour shifts Easy access to drugs

Frequent contact with drugs Pressures in the workplace Easy access to drugs

Depressants directly affect the _____ and indirectly affect the _____. dopamine reward system; glutamate receptors nucleus accumbens; limbic regions of the brain serotonin receptors; GABAnergic system GABAnergic system; dopamine reward system

GABAnergic system; dopamine reward system

A nurse is caring for a patient injured in a car crash. The patient is manifesting signs of alcohol withdrawal delirium. Which does the nurse identify as signs of alcohol withdrawal delirium?

Gross tremors Visual hallucinations Seizures

The nurse is caring for a client withdrawing from the hallucinogen LSD. Which nonpharmacologic treatment should the nurse anticipate being​ used? (Select all that​ apply.) Having one person reassure and​ "talk the client​ down" Ensuring low stimuli with minimal​ light, sound, and activity Conducting group therapy Recommending individual therapy Speaking slowly and clearly to the client

Having one person reassure and​ "talk the client​ down" Ensuring low stimuli with minimal​ light, sound, and activity Speaking slowly and clearly to the client During withdrawal from​ LSD, nonpharmacologic treatment includes speaking slowly and clearly to the​ client; ensuring low stimuli with minimal​ light, sound, and​ activity; and having one person reassure and​ "talk the client​ down." Individual and group therapy would be beneficial after the withdrawal of the substance.

The nurse is teaching a client who uses intravenous drugs about the potential for health problems. Which infectious bloodborne disease should the nurse include in this​ teaching? (Select all that​ apply.) Tuberculosis Hepatitis B AIDS Hepatitis C HIV

Hepatitis B AIDS Hepatitis C HIV

A patient reports joint pain and inability to sleep. Assessment findings include watery eyes, dilated pupils, and increased blood pressure. The nurse suspects that the patient is experiencing withdrawal from which substance?

Heroin

_____ affect(s) the part of the brain involved in breathing and coughing and indirectly activates the dopamine reward system. Dissociative anesthetics Hallucinogens Heroin Stimulants

Heroin

The nurse is talking to a patient who is being treated for an acute exacerbation of chronic obstructive pulmonary disease. The patient states that he or she is not interested in quitting smoking at this time. Which of these statements by the nurse is appropriate at this time?

If you continue to smoke, your breathing problems will get worse. Let me describe how that will happen."

During a prenatal​ visit, a pregnant client admits to using cocaine at least once per​ day, and that getting cocaine is her highest priority. Which problem should the nurse make a priority for this​ client? Imbalanced​ Nutrition: Less than Body Requirements related to limited food intake Impaired Gas Exchange related to respiratory effects of substance abuse Activity Intolerance related to decreased tissue oxygenation Risk for Infection related to drug use

Imbalanced​ Nutrition: Less than Body Requirements related to limited food intake A person who abuses substances will spend money on drugs rather than food and other basic​ needs, which will lead to Imbalanced​ Nutrition: Less than Body Requirements related to limited food intake. Activity Intolerance related to decreased tissue oxygenation does not relate to cocaine use. Clients may have trouble sleeping or getting adequate rest from using cocaine. Risk for Infection related to drug use might be appropriate for cocaine​ use, but the question does not specify how cocaine is being used. Impaired Gas Exchange related to respiratory effects of substance abuse is inappropriate because it is a​ risk, not a current problem.​ (NANDA-I ©​ 2014)

Which outcome is expected for the client who has been treated for alcohol​ abuse? (Select all that​ apply.) Absence of anxiety Improved nutritional status Ability to verbalize negative aspects of alcohol use Minimal alcohol use Regular attendance at a​ self-help group

Improved nutritional status Ability to verbalize negative aspects of alcohol use Regular attendance at a​ self-help group Expected outcomes for the client who has been treated for alcohol abuse include improved nutritional​ status, ability to verbalize negative aspects of alcohol​ use, and regular attendance at a​ self-help group like Alcoholics Anonymous​ (AA). The client is expected to remain free of alcohol​ use, not keep it to a minimum. Anxiety is expected to be controlled at an acceptable and manageable​ level, not to be absent.

A patient is admitted to the emergency department with a suspected history of cocaine use. The nurse expects to find which physiologic changes?

Increased heart rate Increased blood pressure Decreased respiratory rate

Diaphoretic

Inducing perspiration, excessive sweating.

A patient is withdrawing from a central nervous system depressant. The nurse expects what symptoms?

Insomnia Disorientation Orthostatic hypotension

_____ refers to the direct results of using a substance, while _____ focuses more on the indirect effects of repeated use. Substance abuse; substance dependence Addiction; substance abuse Intoxication; substance abuse disorder Intoxication; addiction

Intoxication; substance abuse disorder

When caring for a patient with acute intoxication and a history of chronic alcoholism, the nurse will anticipate administrating which drug?

Intravenous (IV) thiamine and glucose

The nurse is caring for a client experiencing withdrawal from a central nervous system​ (CNS) depressant. Which collaborative treatment should the nurse expect to ​implement? (Select all that​ apply.) Keeping the client awake Inducing vomiting Recommending group therapy Taking vital signs every hour Using activated charcoal to absorb the drug

Keeping the client awake Inducing vomiting Using activated charcoal to absorb the drug During withdrawal from CNS​ depressants, treatment includes keeping the client​ awake, inducing vomiting while employing aspiration​ precautions, and using activated charcoal to absorb the drug. Vital signs are taken more frequently than hourly and should be monitored every 15 minutes. Group therapy would be beneficial after the withdrawal of the substance.

Which statement about ketamine is true? It will cause mild heart and breathing problems, but these are seldom lethal. Ketamine induces anesthesia and hallucinations. Ketamine is only snorted. It is longer acting than PCP and is less intense in its effects.

Ketamine induces anesthesia and hallucinations.

The nurse is assessing an alcoholic patient who has an irreversible form of amnesia characterized by short-term memory loss. Which associated condition does the nurse suspect in the patient?

Korsakoff's psychosis

31. The ED nurse assesses a confused client diagnosed with alcohol use disorder and notes the use of confabulation. Which complication of alcohol use disorder would the nurse suspect? 1) Korsakoff's psychosis 2) Vascular neurocognitive disorder 3) Wernicke's encephalopathy 4) Esophageal varices

Korsakoff's psychosis is identified by a syndrome of confusion, loss of memory, and confabulation. Confabulation is the creating of imaginary events to fill in memory gaps.

A client is being treated for glue inhalation. Which independent intervention should the nurse expect to​ implement? (Select all that​ apply.) Administering an opioid antagonist intravenously Maintaining a quiet environment Assessing the​ client's heart​ rate, respirations, and blood pressure Maintaining an accurate record of fluid intake and output Assigning the client to one nurse for therapeutic intervention

Maintaining a quiet environment Assessing the​ client's heart​ rate, respirations, and blood pressure Maintaining an accurate record of fluid intake and output Assigning the client to one nurse for therapeutic intervention Inhalants can displace oxygen and cause tachycardia and respiratory depression. Assess for renal function because chemicals found in many glues can cause chronic renal damage. It is best to have a single nurse who talks​ to, calms, and reassures the client until the effects of the substance use subside. To calm the​ client, external stimuli should be​ reduced, with minimal​ sound, light, and activity. An opioid antagonist is not effective with the use of inhalants.

A nurse is caring for a patient with cancer. To control nausea and vomiting after chemotherapy, the patient has received a prescription for nabilone. Which commonly abused substance has a similar composition to that of nabilone?

Marijuana

The nurse is teaching a class about stress reduction techniques in relation to substance abuse. Which should the nurse include in the​ lesson? (Select all that​ apply.) Meditation Social drinking Effective coping skills Progressive muscle relaxation Abdominal breathing techniques

Meditation Effective coping skills Progressive muscle relaxation Abdominal breathing techniques Important teaching regarding substance abuse includes stress reduction. This may include a variety of techniques including​ imagery, meditation, muscle​ relaxation, breathing​ techniques, and effective coping skills. In substance​ abuse, alcohol should be avoided​ entirely, even in medications containing alcohol such as cough medicine.

The nurse is planning care for a pregnant client with a substance abuse disorder. Which intervention should the nurse identify to address imbalanced nutrition in this​ client? (Select all that​ apply.) Monitor meal intake. Educate on negative effects of substances on fetal health. Educate on negative effects of substances on body. Assess for signs of infection. Obtain daily weight.

Monitor meal intake. Obtain daily weight.

_____ is a form of treatment specifically designed to boost a patient's motivation to decrease or stop substance use by highlighting discrepancies between stated personal goals related to substance use and current behavior. Contingency management Cognitive-behavior therapy Motivational enhancement therapy Stages of change

Motivational enhancement therapy

◯◯ Sociocultural theories ■■ Certain cultures within the United States, such as __ __ groups, have a high percentage of members who have alcohol use disorder. Other cultures, such as __ groups, have a low percentage of alcohol use disorder. ☐☐ Metabolism of alcohol and cultural views of alcohol use provide possible explanations for the incidence of alcohol use within a cultural group.

Native American Asian

The nurse is caring for a pregnant client with a substance use disorder. Which substance type should the nurse expect to be treated with medication​ therapy? (Select all that​ apply.) Nicotine Opioid Narcotic Cocaine Alcohol

Nicotine Opioid Narcotic Alcohol

The nurse is caring for a patient who has received counseling for smoking cessation. Which observation indicates that the patient has not smoked cigarettes for 12 hours?

O2 level is at 96%

The nurse prepares a teaching tool about substance abuse in older adults. Which information should the nurse​ include? (Select all that​ apply.) Older women are more likely to use prescription medicines. Alcohol and other substances can make it difficult to diagnose medical problems. Substance abuse is more likely to be recognized in older adults. Individuals can have substance abuse problems at any age. Depression and alcohol abuse are disorders frequently found in clients who completed suicide.

Older women are more likely to use prescription medicines. Alcohol and other substances can make it difficult to diagnose medical problems. Individuals can have substance abuse problems at any age. Depression and alcohol abuse are disorders frequently found in clients who completed suicide.

An overdose of which substance causes pinpoint pupils, nystagmus, and hypotension?

Opioid

Which drug is NOT a stimulant? crack nicotine Ritalin PCP

PCP

_____ alters the distribution of glutamate in the brain. PCP Marijuana LSD Heroin

PCP

A nurse is caring for a patient who has been admitted to the hospital with hepatic failure. On taking the history, the nurse finds that the patient is an alcoholic who has been consuming alcohol for the past 25 years. Which other health problems associated with alcoholism should the nurse assess the patient for?

Palmar erythema Esophageal varices Wernicke encephalopathy

A patient with a long-term history of antidepressant therapy reports sleeplessness, dizziness after waking up, and weakness. The nurse learns that the patient has suddenly stopped taking the antidepressants. To treat the patient's symptoms, the nurse expects that what medication will be prescribed?

Phenobarbital -long-acting barbiturate that helps relieve the symptoms of antidepressant withdrawal.

A patient with narcolepsy received a prescription for drug therapy. During a follow-up visit, the patient's assessment findings include excitement, confusion, a blood pressure of 140/90 mm Hg, a heart rate of 150 beats/minute, and ventricular dysrhythmia. The nurse identifies that which medications are appropriate to be included in the patient's treatment plan?

Phentolamine Procainamide

_____ incentives are more effective than _____ incentives in promoting continuing treatment and in decreasing substance abuse. Positive; negative Neutral; positive Negative; positive Neutral; negative

Positive; negative

After collecting data from a patient, the nurse concludes that the patient is experiencing the effects of cocaine withdrawal. Which set of clinical manifestations supports the nurse's conclusion?

Prolonged sleep, vivid dreams, apathy, and disorientation

The nurse is helping students form a​ peer-support team to implement a​ harm-reduction program for alcohol use at a high school. Which topic is of the highest priority in this​ population? Role of alcohol in increasing hyperactivity and aggression in adolescents Links to anxiety and panic associated with alcohol use Impact of social media and underage drinking Risk of sexual assault related to alcohol use

Risk of sexual assault related to alcohol use These answers are all correct because they outline the risk of alcohol use in adolescent populations.​ However, the question focuses on identifying the area of highest priority. In this​ case, the highest priority is safety. Educating adolescents on the risk of sexual assault with binge drinking is essential. Alcohol may also cause emotional disturbances such as anxiety and panic and hyperactivity and aggression. Adolescents may also be pressured by social media to engage in underage drinking. These remain important areas for​ education, but safety from sexual assault is the highest priority.

The nurse suspects that a patient is experiencing alcohol withdrawal delirium based on what assessment findings?

Seizures, hallucinations

_____ is a theory that categorizes readiness to change problematic behaviors into five stages. Contingency management Cognitive-behavior therapy Motivational enhancement therapy Stages of change

Stages of change

_____ is a pattern of the harmful use of a psychoactive substance. Substance use disorder Substance dependence Addiction Intoxication

Substance use disorder

_____ are characterized by the abuse of or dependence on psychoactive substances. Substance use disorders Psychoactive substances Psychotropic drugs Intoxicating substances

Substance use disorders

People can develop substance use disorders in which way? Substance use disorders can develop as a result of the intentional use of a substance for its psychoactive effect. Psychoactive substances are decriminalized. Subjects are more tolerant of larger doses. Society approves of increased drug use.

Substance use disorders can develop as a result of the intentional use of a substance for its psychoactive effect.

What is substance abuse?

THE HABITUAL USE OF A SUBSTANCE THAT FALLS OUTSIDE OF MEDICAL NECESSITY OR SOCIAL ACCEPTANC AND IS USED FOR THE SINGLE PURPOSE OF ALTERING ONES MOOD, EMOTION, OR STATE OF CONSCIOUSNESS.

A patient with a history of chronic alcohol use is hospitalized. The nurse should monitor the patient for what withdrawal symptoms?

Tachycardia Hyperreflexia Gross tremors

When aiding a patient to quit the use of tobacco, which should the nurse suggest to the patient as methods to deal directly with the physical urge to use tobacco?

Take a shower. Drink a lot of water. Walk, jog, or bicycle.

The nurse is assessing a client with a dependency on cocaine. Which cognitive manifestation should the nurse expect to assess in this​ client? (Select all that​ apply.) Talks incessantly with rambling thought patterns. Pulls at clothes while fidgeting in the chair. States sleeping all the time. Appears overly happy despite the hospital admission. Expresses feelings of anxiety.

Talks incessantly with rambling thought patterns. Pulls at clothes while fidgeting in the chair. Appears overly happy despite the hospital admission. Expresses feelings of anxiety. Individuals who routinely use cocaine have symptoms of​ anxiety; demonstrate excessive talking that indicates rambling thought​ processes, elation,​ euphoria, and sometimes tactile hallucinations.​ Insomnia, not excessive​ sleeping, occurs in those who routinely use cocaine.

When does tolerance occur? Compensatory mechanisms are in place, but the drug is no longer there to counterbalance the effects. The body adapts and tries to compensate for the repeated influx of a substance. The substance use is continued despite knowledge of having persistent or recurrent physical or psychological problems. The individual and the family have accepted the addiction and support its continuation.

The body adapts and tries to compensate for the repeated influx of a substance

The nurse is evaluating a​ client's progress in recovering from acute alcohol withdrawal. Which finding should the nurse evaluate as​ positive? (Select all that​ apply.) The client has adopted healthful eating habits. The client regularly attends Alcoholics Anonymous​ (AA) meetings. The client and spouse have moved into separate homes. The client has returned to a​ full-time work schedule. The client stays away from old drinking buddies.

The client has adopted healthful eating habits. The client regularly attends Alcoholics Anonymous​ (AA) meetings. The client has returned to a​ full-time work schedule. The client stays away from old drinking buddies.

The nurse is developing the plan of care for a​ 65-year-old man admitted with alcohol abuse and withdrawal symptoms who denies drinking. The family tells the nurse that the​ client's drinking is impacting his daily life. Which primary goal should the nurse include in the plan of​ care? The client will achieve optimal nutritional status. The client will admit that alcohol is controlling his life. The client will participate in support groups such as AA after discharge. The client will remain sober.

The client will admit that alcohol is controlling his life. Goals for client care depend on the​ client's needs. This client is denying a problem with alcohol. Even though the family has outlined that his alcohol abuse is impacting his daily​ activities, the nurse must first help the client in recognizing that alcohol is controlling his life. Remaining sober is an important​ goal, but it is not appropriate at this time because the client has not admitted to his problem. Nutritional status is important for the​ client, but the priority goal is recognition of the​ problem, so the nurse can implement other interventions. The client will only participate in AA if he believes that he has a problem with alcohol. OK

A​ 20-year-old woman discloses to the nurse that she has been drinking six to eight drinks per evening after breaking up with her partner 2 weeks ago. She states that it is only temporary and that she will not become an alcoholic. Which is a cause of concern based on the​ nurse's knowledge of alcohol abuse in​ women? Women have a higher​ first-pass metabolism of alcohol in the stomach and upper small intestine. Women develop alcohol abuse and dependence in less time than do men. Women who go through breakups are more likely to be pregnant. Women who are dependent on alcohol or consume heavier amounts are at a greater risk for hypotension.

Women develop alcohol abuse and dependence in less time than do men.

A client with a history of substance abuse denies using any narcotics. Which expected outcome should the nurse select for this​ client? (Select all that​ apply.) The client will admit to having a problem with substance abuse. The client will remain free from injury. The client will participate in group therapy. The client will verbalize the negative effects of alcohol on the body. The client will describe choices made that contributed to substance abuse

The client will admit to having a problem with substance abuse. The client will describe choices made that contributed to substance abuse The client admitting to a problem with substance abuse and the client describing choices made that contributed to substance abuse are both expected outcomes for a client diagnosed with ​Denial, Ineffective. These expected outcomes demonstrate the​ client's willingness and ability to recognize substance abuse as a problem. The client remaining free of​ injury, the client verbalizing the negative effects of alcohol on the​ body, and the client participating in group therapy are not expected outcomes for a diagnosis of ​Denial, Ineffective. ​(NANDA-I ©2014) OK

The nursing assessment findings for a client with alcohol abuse include a recent fall at​ home, decreased​ appetite, report of blurred​ vision, and a denial that alcohol has negative effects on the body. Which is an appropriate goal for this client at this​ time? (Select all that​ apply.) The client will participate in a support group. The client will verbalize the negative effects of alcohol on the body. The client will remain free of injury. The client will maintain adequate nutrition. The client will remain sober.

The client will verbalize the negative effects of alcohol on the body. The client will remain free of injury. The client will maintain adequate nutrition.

Why do cocaine and methamphetamines work? Glutamate receptors abound throughout the nucleus accumbens and limbic regions and produce a reinforcing effect of the drugs. Serotonin reuptake inhibitors are at work along with the dopamine reward system. The GABAnergic reward system works with glutamate receptors to produce a reinforcing effect of the drugs. The drugs bind to the molecules that transport excess dopamine back to the terminal buttons, preventing them from operating effectively and leaving more dopamine in the synapse.

The drugs bind to the molecules that transport excess dopamine back to the terminal buttons, preventing them from operating effectively and leaving more dopamine in the synapse

Which statement about LSD is NOT true? People who are alone when experiencing a bad trip may get hurt or kill themselves as they respond to the hallucinations that occur after taking LSD. The effects of an LSD high generally last up to 24 hours. Psychosis and flashbacks are two aftereffects that can occur from LSD use. LSD does not cause withdrawal symptoms.

The effects of an LSD high generally last up to 24 hours

The nurse is collaborating with an outpatient detoxification center for a client who is experiencing acute alcohol withdrawal. Which critical assessment should the nurse make prior to leaving the client at the outpatient​ center? The staff is aware of how to administer the medications in the detoxification protocol. The staff understands signs of severe withdrawal and knows when to notify the healthcare provider. The staff understands how to use the vital sign equipment. The staff understands how to keep a confused client safe.

The staff understands signs of severe withdrawal and knows when to notify the healthcare provider. The​ nurse's role is to promote safety in an outpatient or home setting with a client. In this​ case, the outpatient detoxification center is working from an established protocol. The staff will be educated in how to monitor the​ client's vital​ signs, administer needed​ medications, and keep the client safe. It is the​ nurse's responsibility to ensure that the outpatient center recognizes signs of severe alcohol withdraw and knows when to notify the healthcare provider or to transfer the client to a center with a higher level of acuity.

_____ occurs when more of a substance is required to obtain the same effect after repeated use. Habituation Maintenance Tolerance Adaptation

Tolerance

Which statement about heroin use is true? Heroin is one of the least addictive opioids. Withdrawal from heroin begins 24 hours after the drug was last used. Tolerance to heroin often causes irritability rather than euphoria. People who start out using heroin usually do so by injecting it.

Tolerance to heroin often causes irritability rather than euphoria.

The nurse suspects that a coworker has a substance use disorder. Which finding should the nurse use to confirm this​ suspicion? (Select all that​ apply.) Wears more makeup than other nurses. Wears long sleeves despite hot weather. Breath smells like mouthwash. Frequently asks peers for breath mints. Face is frequently flushed when indoors.

Wears long sleeves despite hot weather. Breath smells like mouthwash. Frequently asks peers for breath mints. Face is frequently flushed when indoors.

A nurse is creating a dietary plan for a patient who is a chronic alcoholic who has been diagnosed with Wernicke encephalopathy. Why should the nurse include food items rich in vitamin B 1 in the patient's diet plan?

Wernicke encephalopathy is caused by a deficiency of thiamine.

The symptoms of alcohol withdrawal marked by confusion, convulsions, visual hallucinations, and fever are called: Korsakoff's syndrome. delirium tremens. malnutrition due to alcohol consumption. cirrhosis.

delirium tremens

●● The defense mechanism of __ is commonly used by clients who have problems with a substance use or addictive disorder. For example, a person who has long-term tobacco use might say, "I can quit whenever I want to, but smoking really doesn't cause me any problems." Frequently, denial prevents a client from obtaining help with substance use or an addictive behavior.

denial

The _____ is known as the system of neurons that relies on dopamine and gives rise to pleasant feelings. nucleus accumbens and ventral tegmental area dopamine reward system dopaminergic hypothesis GABAnergic system

dopamine reward system

Which substance is similar to methamphetamine and the hallucinogen mescaline? crack Ritalin PCP Ecstasy

ectasy

◯◯ Safety is the primary focus of nursing care during acute intoxication or withdrawal. ■■ Maintain a safe environment to prevent __; implement __ precautions as necessary. ■■ Provide close observation for withdrawal symptoms, possibly one-on-one supervision. Physical restraint should be a last resort. ■■ Orient the client to time, place, and person. ■■ Maintain adequate nutrition and fluid balance. ■■ Create a __-__ environment. ■■ Administer medications as prescribed to treat the effects of intoxication or to prevent or manage withdrawal. ■■ Monitor for covert substance use during the detoxification period.

falls seizure low-stimulation

Substance abuse can develop in all of these ways EXCEPT: unintentionally, through environmental exposure. from wanting to escape troubles in life. when the psychoactive element is a side effect, and the substance is taken for medicinal reasons unrelated to the psychoactive effect. as a result of the intentional use of the substance for its psychoactive effect.

from wanting to escape troubles in life

Research has shown that people who begin abusing marijuana at an early age have: a smaller nucleus accumbens and basal ganglia. irreversible memory problems later in life. increased activation in their dopamine reward system. greater atrophy in the hippocampus and the cerebellum.

greater atrophy in the hippocampus and the cerebellum.

People who use Ecstasy for the first time report: heightened feelings of empathy toward others and greater sensitivity to touch. impaired cognitive functioning. poor concentration, depression, and fatigue. poor mood and difficulty regulating emotions.

heightened feelings of empathy toward others and greater sensitivity to touch

The nurse should monitor a patient who uses nicotine gum for what side effects?

hiccups and mouth sores

With repeated use, amphetamines can cause _____ when high. hostility toward others and a sense of grandiosity increased clarity in thinking depression and fatigue anxiety and irritability

hostility toward others and a sense of grandiosity

Research has shown that: stimulants affect the dopamine reward system indirectly. if animals are given a drug that blocks the effects of dopamine, they will not work as hard to receive electrical stimulation. people with excessive dopamine receptors may be more vulnerable to drug use. rats bred so that they consume high amounts of alcohol appear to have a more responsive dopamine reward system.

if animals are given a drug that blocks the effects of dopamine, they will not work as hard to receive electrical stimulation.

◯◯ Provide emotional support and reassurance to the client and family. ■■ Educate the client and family about codependent behaviors. ◯◯ Begin to educate the client and family about addiction and the initial treatment goal of abstinence. ◯◯ Educate the client and family regarding __ any prescription medications in the home that are not being used. Encourage the client not to share medication with someone for whom that medication is not prescribed. ◯◯ Begin to develop motivation and commitment for abstinence and recovery (abstinence plus working a program of personal growth and self-discovery). ◯◯ Encourage self-responsibility. ◯◯ Help the client develop an __ __ - a list of things the client would need to do and __ he would need to contact. ◯◯ Encourage __ at self-help groups.

removing emergency plan people attendance

The desire for the gratifying effects of using a substance is called a: reward craving. relief craving. drug cue. coping strategy.

reward craving

The effects of _____ are similar to but slower than the neurological effects of cocaine. Ecstasy crack Ritalin nicotine

ritalin

◯◯ Nursing Considerations ■■ Monitor the client's vital signs and neurological status. ■■ Provide for client safety by implementing __ __. ◯◯ Client Education ■■ Encourage the client to adhere to the treatment plan. ■■ Advise client taking disulfiram to avoid all __.

seizure precautions alcohol

The nurse is caring for a patient admitted to the hospital with pneumonia who has a history of misuse of alprazolam. Which clinical manifestations could be attributed to withdrawal from this medication?

seizures and delirium

●● Risk Factors ◯◯ Genetics - Predisposition to developing a substance use disorder due to family history ◯◯ Lowered __ __ ◯◯ Lowered tolerance for __ and __ ◯◯ Few meaningful __ __ ◯◯ Few __ __ ◯◯ __-__ tendencies

self-esteem pain frustration personal relationships life successes Risk-taking

Depressants tend to: not lead to tolerance but can lead to withdrawal. slow a person down, decreasing behavioral activity and level of awareness. work in a similar manner as stimulants. cause severe hallucinations and delusions.

slow a person down, decreasing behavioral activity and level of awareness

●● Subjective and Objective Data ◯◯ The nurse should use open-ended questions to obtain the following information for the nursing history. ■■ Type of __ or __ behavior ■■ __ and __ of substance use ■■ __ of substance used ■■ __ at onset of substance use ■■ Changes in __ or __ performance ■■ Changes in __ __ ■■ Periods of __ in history ■■ Previous __ manifestations ■■ __ of last substance use/addictive behavior

substance addictive Pattern frequency Amount Age occupational school use patterns abstinence withdrawal Date

Alcohol activates: the production of dopamine and stimulates the production of endogenous opioids. the neurotransmitters GABA and serotonin in the shifting from recreational use to abuse and dependence. the limbic system and serotonin receptors to develop alcohol dependence. the central nervous system

the production of dopamine and stimulates the production of endogenous opioids.

The DSM-5 diagnostic criteria for substance intoxication include: withdrawal. toleration. the substance causes specific (but reversible) biological, mental, and psychological changes. the substance is illegal or very difficult to obtain.

the substance causes specific (but reversible) biological, mental, and psychological changes

The client states that he has been increasing the amount and frequency of the drug he is using. The nurse determines that the client most likely has developed_________________ to the drug.

tolerance

What is the typical result of continued substance use? withdrawal intoxication tolerance dependence

tolerance

Most substance abuse treatment programs, especially those based on the Twelve-Step approach, insist that clients refrain from alcohol for the rest of their lives. This is known as: tolerance training. controlled dosing. abstinence. withdrawal.

tolerance training.

One of the effects of long-term alcohol use disorder—alcoholism—is enlarged _____ (the cavities in the brain filled with cerebrospinal fluid). medulla oblongata ventricles pons reticular formation

ventricles

Which of these has NOT been identified as a factor contributing to substance abuse disorders? unemployment peers' behavior family dysfunction wealth

wealth

Which of these is NOT an effect of methamphetamine abuse? adverse effect on the functioning of dopamine and serotonin impaired memory increased blood pressure weight gain

weight gain

On​ assessment, the nurse notes spider angiomata to the​ client's face and​ abdomen, jaundice, and ascites of a client with cirrhosis. The client tells the nurse that he plans to stop drinking to reverse his cirrhosis. Which is the most appropriate response by the​ nurse? ​"Avoid medications such as acetaminophen to improve your liver​ function." ​"The liver has the potential to regenerate if alcohol exposure is​ limited." ​"Abdominocentesis will help to reduce the​ ascites." ​"Cirrhosis cannot be​ reversed, but abstaining from alcohol can prevent further​ damage."

​"Cirrhosis cannot be​ reversed, but abstaining from alcohol can prevent further​ damage."

The nurse prepares a presentation to promote smoking cessation at an assisted living facility. Which condition should the nurse present as having an association with​ smoking? (Select all that​ apply.)

​Rationale: Smoking increases the risk of many types of​ cancers, especially cancers of the respiratory and gastrointestinal​ (GI) tracts. Smoking has also been linked to diseases other than cancer in older​ adults, including pulmonary and cardiovascular​ diseases, diabetes​ complications, bone​ disease, bone density​ loss, cataracts, and stomach ulcers. Smokers are up to 10 times more likely to get cancer than a person who has never smoked.​ However, it has not been linked to retinal detachment.

A client discloses that her father was an alcoholic. She is concerned that she will develop alcohol abuse in the​ future, even though she abstains from drinking. Which questions would be of the highest priority for the nurse to ask at this​ time? ​"Do you have a history of depression or mental health​ difficulties?" ​"Do you exercise on a regular​ basis?" ​"Are you currently​ employed?" ​"Are you able to limit the amount of alcohol you​ consume?"

​"Do you have a history of depression or mental health​ difficulties?" Children of alcoholics​ (COAs) have significantly more mental health difficulties and a higher rate of substance abuse issues. Their families had higher rates of unemployment and unhealthy lifestyle habits when compared with control groups. Asking about mental health demonstrates understanding of the most prominent risk factor for alcohol abuse. Unemployment is a risk​ factor, but mental health is a more significant indicator for future coping. The client states that she does not​ drink, so asking about the amount of alcohol consumed is not a priority question. Assessing for exercise will help determine lifestyle​ habits, but mental health is a higher priority.

A client of American Indian descent who states that he abstains from drinking alcohol because his father died from complications related to alcoholism is concerned that his children may be at risk. Which nursing statement is the most accurate in summarizing the genetic influences related to the potential for alcohol abuse in this​ population? ​"Research has shown that American Indians carry risk factors associated with drug sensitivity and​ tolerance." ​"Ensuring that children do not drink at a young age will help decrease the risk of alcoholism in​ future." ​"Social determinants of health are the greatest genetic influence in the development of alcohol​ abuse." ​"Genetic influences related to alcohol dependency are linked to exposure to​ trauma."

​"Ensuring that children do not drink at a young age will help decrease the risk of alcoholism in​ future." American Indians experience alcoholism at higher rates than do other populations. Genetics is a contributing factor. It has been found that American Indians do not have the genes that alter​ alcohol-metabolizing enzymes, and they lack protective variants found in other groups. This genetic variance can lead to drug sensitivity and tolerance. The incorrect answers focus on​ environmental, rather than genetic factors. Environmental factors that may contribute include early onset​ drinking, trauma​ exposure, and environmental hardship.

A client presents to the emergency department with signs of alcohol withdrawal. Which statement would be consistent for the client to make 6 to 8 hours after their last​ drink? (Select all that​ apply.) ​"I feel like I​ can't sit​ still, I​ don't want to stay in the​ bed." ​"My heart feels like it is going faster than​ normal." ​"Stop bothering​ me! You keep waking me​ up." ​"Where am​ I? Who are​ you?" ​"Can you see the spiders crawling on the​ wall?"

​"I feel like I​ can't sit​ still, I​ don't want to stay in the​ bed." ​"My heart feels like it is going faster than​ normal." ​"Stop bothering​ me! You keep waking me​ up." Signs of alcohol withdrawal syndrome occurring 6 to 8 hours after the last drink include​ irritability, increased anxiety and​ tremor, and mild tachycardia. Clients may feel their heart rate has increased​ (mild tachycardia). They may be irritable and ask not to be touched or left​ alone, and they may display signs of anxiety such as not sitting still or staying in the bed. Later signs of alcohol withdrawal such as confusion to​ person, place, and time and hallucinations typically occur 2 to 3 days after the last drink.

The nurse is caring for a client with a history of substance abuse. Which statement should indicate to the nurse that the client is progressing through an effective course of​ recovery? (Select all that​ apply.) ​"I haven't used drugs in over 2​ years." ​"I don't think I really was​ addicted; I just went through a bad​ patch." ​"I've been back in school for the last two​ semesters." ​"I still like hanging with my old​ friends." ​"I know I will always need to use the tools from the addiction​ program."

​"I haven't used drugs in over 2​ years." ​"I've been back in school for the last two​ semesters." ​"I know I will always need to use the tools from the addiction​ program."

A client tells the nurse about their increasing anxiety and use of alcohol to help reduce their anxiety symptoms. Which complementary health approach should the nurse suggest to better promote the​ client's health and reduce their use of​ alcohol? ​"It may be helpful to use an herbal supplement such as kava kava for your​ anxiety." ​"Acupuncture has been proven to cure anxiety and​ alcoholism." ​"It would be helpful to use neurotherapy and yoga as a substitute for drinking​ alcohol." ​"It is difficult to manage anxiety without the use of pharmacologic​ therapy."

​"It would be helpful to use neurotherapy and yoga as a substitute for drinking​ alcohol."

The nursing assessment findings for a client with known alcohol abuse include a recent fall at​ home, decreased​ appetite, complaints of blurred​ vision, and believes that alcohol does not have negative effects on the body. Which nursing diagnosis is a priority for the nurse when developing the plan of care for this​ client? ​Injury, Risk for ​Knowledge, Deficient ​Confusion, Chronic ​Nutrition, Imbalanced: Less than Body Requirements

​Injury, Risk for

A client presents with a burn on the right index finger caused by falling asleep with a lit cigarette in hand.​ Additionally, the client reports an inability to relax and insomnia and denies that smoking has negative effects on the body. Which is the priority nursing diagnosis when developing the plan of​ care?

​Injury, Risk for is the priority diagnosis. The client has a burn on the right index finger from falling asleep with a lit cigarette in the hand.​ Therefore, safety is a priority concern. The assessment findings also show the client has Anxiety​ (inability to​ relax), Sleep​ Pattern, Disturbed ​(complaints of​ insomnia), and ​Knowledge, Deficient​ (denial that smoking has negative effects on the​ body), but these are not the priority.​ (NANDA-I ©​ 2014)

The nurse is caring for a client who uses nicotine. Which clinical manifestation should the nurse anticipate when assessing the​ client? (Select all that​ apply.)

​Rationale: Assessment findings for nicotine use can include chronic​ cough, nervousness, burns on​ fingers, and shortness of breath on exertion. Lips are not dry and chapped as a result of nicotine​ use, but they can be discolored.

The community nurse is creating a resource tool kit for a public health website. Which of these organizations should the nurse link to for information on smoking​ cessation? (Select all that​ apply.)

​Rationale: The American Red Cross deals with disaster​ mitigation, not smoking cessation. The other organizations are good sources of information about smoking cessation.

The nurse is assessing a pregnant client who states that she smokes a pack of cigarettes each day. Which action by the nurse is the most​ appropriate?

​Rationale: Before planning any intervention with a client who​ smokes, it is essential to determine whether or not the client is willing or ready to stop smoking. Smoking can be detrimental to fetal development and is related to increased rate of spontaneous​ abortion, increased incidence of placental​ abruption, being small for gestational​ age, and sudden infant death syndrome​ (SIDS). Informing the client of the risk factors and offering help with a referral to a support group provides​ encouragement, but it is the​ client's decision to quit.​ Commonly, women will stop smoking for the duration of the​ pregnancy, which gives the nurse an opportunity to work with the client to help with permanent smoking cessation. No action results in not providing adequate health promotion information.

The clinic nurse assesses a pregnant client who has delayed seeking prenatal care. The client states she is addicted to nicotine and is inquiring about therapy. Which therapy should the nurse recommend as options for this client to​ consider? (Select all that​ apply.)

​Rationale: Behavioral​ therapy, group supportive​ therapy, and medication therapy are beneficial therapies in a client addicted to nicotine. Detoxification is not beneficial and would not be appropriate for the client addicted to nicotine. Nicotine is not a​ narcotic; therefore, a referral to Narcotics Anonymous would not benefit this client.

The nurse at a clinic prepares a class on the effects of secondhand smoke on children. Which condition should the nurse​ include? (Select all that​ apply.)

​Rationale: Children exposed to secondhand smoke are at an increased risk of otitis​ media, upper respiratory tract​ infections, and cancer. Individuals who actually smoke are at risk for developing wrinkles in the skin and yellowing of the fingers.

The nurse develops a discharge plan for a client with chronic obstructive pulmonary disease​ (COPD) and a nicotine addiction. Which nursing intervention is appropriate for promoting smoking​ abstinence? (Select all that​ apply.)

​Rationale: Consulting with the healthcare provider regarding the possibility of ordering smoking cessation​ medications, teaching relaxation​ techniques, and providing community resource information are all appropriate nursing interventions when promoting smoking abstinence. Assessing for injury and for airway patency does not promote abstinence.

The school nurse is addressing parents about risk factors for children becoming addicted to nicotine. Which factor should be​ included? (Select all that​ apply.)

​Rationale: Emotions, social​ pressure, alcohol​ use, and age are all risk factors associated with nicotine use. A lack of​ education, not higher​ education, is also associated with nicotine use.

The nurse is creating a plan of care for a client undergoing treatment for nicotine addiction. Which expected outcome should the nurse consider​ appropriate? (Select all that​ apply.)

​Rationale: Expected outcomes for clients with nicotine addiction include a patent​ airway, freedom from​ injury, the ability to describe methods of coping without nicotine​ use, and the ability to voice benefits of smoking cessation programs. The client should also​ verbalize, not​ deny, the negative effects of nicotine on the body.

A client asks the nurse about the effectiveness of hypnosis for smoking cessation. Which is the​ nurse's correct ​response?

​Rationale: Mixed results have been found in​ studies, perhaps due to the varying quality of the hypnotherapy.​ Therefore, it is recommended to pair hypnotherapy with other traditional smoking cessation programs.

The school nurse is seeking to reduce the rate of smoking among teenagers in the community. Which method should the nurse recognize as proving​ effective? (Select all that​ apply.)

​Rationale: National,​ state, and local program activities that have reduced and prevented youth tobacco use in the past have included combinations of the​ following: counteradvertising mass media campaigns​ (such as TV and radio​ commercials); comprehensive​ school-based tobacco-use prevention policies and​ programs; and higher costs of tobacco products through increased excise taxes.​ Parent-based programs and showing celebrities glamorizing smoking in movies do not stop teens from smoking.

The nurse is teaching clients about the health risks associated with nicotine addiction. Which condition should the nurse​ include? (Select all that​ apply.)

​Rationale: Nicotine addiction is associated with multiple health risks including heart​ disease, lung​ cancer, and chronic obstructive pulmonary disease. Cancers of the​ bladder, colon, and cervix have been linked to smoking. Graves​ disease, infertility, early​ menopause, dysmenorrhea,​ impotence, osteoporosis, and degenerative disc disease have also been associated with smoking. Other less serious consequences include discolored teeth and​ fingernails, premature aging and​ wrinkling, bad​ breath, reduced sense of smell and​ taste, strong smell of smoke clinging to hair and​ clothing, and gum disease. Recent studies have indicated a negative relationship between Kaposi sarcoma and nicotine use.

The public health nurse is preparing a presentation on nicotine addiction. Which nicotine replacement therapies​ (NRTs) should the nurse state are available over the​ counter? (Select all that​ apply.)

​Rationale: Nicotine patches and nicotine gum are available over the counter. Nicotine spray and nicotine inhalers require a prescription. Vapers are not NRTs. Researchers have yet to determine if vapers are safe or are simply less harmful than tobacco.

The nurse assesses a client with a nicotine addiction who is​ irritable, has not slept much​ lately, and has burns on his clothing. When developing the plan of​ care, the nurse identifies ​Injury, Risk for as the priority nursing diagnosis. Which is the appropriate goal for this client based on the assigned​ diagnosis?

​Rationale: Remaining free of injury is a goal appropriate for the diagnosis of ​Injury, Risk for. Demonstrating​ relaxation, verbalizing strategies for smoking​ cessation, and avoiding complications are all appropriate goals based on the assessment​ findings, but not related to the priority diagnosis identified by the nurse.​ (NANDA-I ©​ 2014)

Which drug​ and/or behavior should the nurse anticipate as a risk associated with the use of​ tobacco? (Select all that​ apply.)

​Rationale: Tobacco use has been associated with​ cannabis, alcohol, and cocaine use and with unprotected sexual activity and fighting with​ others, as well as other risky behaviors.


Set pelajaran terkait

SP Exam 1 Study Guide (chapter one)

View Set

Speech & Hearing Science: Quizzam #2 Vocab

View Set

Unit 20: Environmental Issues and the Real Estate Transaction

View Set

Nutrition Chapter 2: Tools for Healthy Eating

View Set

Planning Your Research end of course quiz

View Set

Annuities Part 1 FINAL EXAM (CT 109885)

View Set

Unit 6: Life Insurance Policy Options

View Set